По Π·Π°ΠΊΠΎΠ½Ρƒ ΠΎΠΌΠ° Ρ‚ΠΎΠΊ Π² Ρ†Π΅ΠΏΠΈ Ρ€Π°Π²Π΅Π½: ЭлСктричСский Ρ‚ΠΎΠΊ. Π—Π°ΠΊΠΎΠ½ Ома для участка Ρ†Π΅ΠΏΠΈ. Π’ΠΈΠ΄Ρ‹ соСдинСния ΠΏΡ€ΠΎΠ²ΠΎΠ΄Π½ΠΈΠΊΠΎΠ².

Π‘ΠΎΠ΄Π΅Ρ€ΠΆΠ°Π½ΠΈΠ΅

ЭлСктричСский Ρ‚ΠΎΠΊ. Π—Π°ΠΊΠΎΠ½ Ома для участка Ρ†Π΅ΠΏΠΈ. Π’ΠΈΠ΄Ρ‹ соСдинСния ΠΏΡ€ΠΎΠ²ΠΎΠ΄Π½ΠΈΠΊΠΎΠ².

ЭлСктричСский Ρ‚ΠΎΠΊ — упорядочСнноС Π΄Π²ΠΈΠΆΠ΅Π½ΠΈΠ΅ заряТСнных частиц ΠΏΠΎΠ΄ дСйствиСм сил элСктричСского поля ΠΈΠ»ΠΈ сторонних сил.
Π—Π° Π½Π°ΠΏΡ€Π°Π²Π»Π΅Π½ΠΈΠ΅ Ρ‚ΠΎΠΊΠ° Π²Ρ‹Π±Ρ€Π°Π½ΠΎ Π½Π°ΠΏΡ€Π°Π²Π»Π΅Π½ΠΈΠ΅ двиТСния ΠΏΠΎΠ»ΠΎΠΆΠΈΡ‚Π΅Π»ΡŒΠ½ΠΎ заряТСнных частиц.
ЭлСктричСский Ρ‚ΠΎΠΊ Π½Π°Π·Ρ‹Π²Π°ΡŽΡ‚ постоянным, Ссли сила Ρ‚ΠΎΠΊΠ° ΠΈ Π΅Π³ΠΎ Π½Π°ΠΏΡ€Π°Π²Π»Π΅Π½ΠΈΠ΅ Π½Π΅ ΠΌΠ΅Π½ΡΡŽΡ‚ΡΡ с Ρ‚Π΅Ρ‡Π΅Π½ΠΈΠ΅ΠΌ Π²Ρ€Π΅ΠΌΠ΅Π½ΠΈ.

ΠŸΡƒΡΡ‚ΡŒ цилиндричСский ΠΏΡ€ΠΎΠ²ΠΎΠ΄Π½ΠΈΠΊ ΠΈΠΌΠ΅Π΅Ρ‚ ΠΏΠΎΠΏΠ΅Ρ€Π΅Ρ‡Π½ΠΎΠ΅ сСчСниС ΠΏΠ»ΠΎΡ‰Π°Π΄ΡŒΡŽ S. Π—Π° ΠΏΠΎΠ»ΠΎΠΆΠΈΡ‚Π΅Π»ΡŒΠ½ΠΎΠ΅ Π½Π°ΠΏΡ€Π°Π²Π»Π΅Π½ΠΈΠ΅ Π² ΠΏΡ€ΠΎΠ²ΠΎΠ΄Π½ΠΈΠΊΠ΅ ΠΏΡ€ΠΈΠΌΠ΅ΠΌ Π½Π°ΠΏΡ€Π°Π²Π»Π΅Π½ΠΈΠ΅ слСва Π½Π°ΠΏΡ€Π°Π²ΠΎ. Заряд ΠΊΠ°ΠΆΠ΄ΠΎΠΉ частицы Π±ΡƒΠ΄Π΅ΠΌ ΡΡ‡ΠΈΡ‚Π°Ρ‚ΡŒ Ρ€Π°Π²Π½Ρ‹ΠΌ q0.

Π’ объСмС ΠΏΡ€ΠΎΠ²ΠΎΠ΄Π½ΠΈΠΊΠ°, ΠΎΠ³Ρ€Π°Π½ΠΈΡ‡Π΅Π½Π½ΠΎΠΌ ΠΏΠΎΠΏΠ΅Ρ€Π΅Ρ‡Π½Ρ‹ΠΌΠΈ сСчСниями 1 ΠΈ 2 с расстояниСм Ξ”l ΠΌΠ΅ΠΆΠ΄Ρƒ Π½ΠΈΠΌΠΈ, содСрТится ΠΎΠ±Ρ‰ΠΈΠΉ заряд q = q0nSΞ”l. Если частицы двиТутся слСва Π½Π°ΠΏΡ€Π°Π²ΠΎ со срСднСй ΡΠΊΠΎΡ€ΠΎΡΡ‚ΡŒΡŽ v, Ρ‚ΠΎ Π·Π° врСмя Ξ”t = Ξ”l / v всС частицы, Π·Π°ΠΊΠ»ΡŽΡ‡Π΅Π½Π½Ρ‹Π΅ Π² рассматриваСмом объСмС, ΠΏΡ€ΠΎΠΉΠ΄ΡƒΡ‚ Ρ‡Π΅Ρ€Π΅Π· ΠΏΠΎΠΏΠ΅Ρ€Π΅Ρ‡Π½ΠΎΠ΅ сСчСниС 2.
ΠŸΠΎΡΡ‚ΠΎΠΌΡƒ сила Ρ‚ΠΎΠΊΠ° Ρ€Π°Π²Π½Π°: Π’ БИ Π΅Π΄ΠΈΠ½ΠΈΡ†Π΅ΠΉ силы Ρ‚ΠΎΠΊΠ° являСтся Π°ΠΌΠΏΠ΅Ρ€ (А). Π­Ρ‚Ρƒ Π΅Π΄ΠΈΠ½ΠΈΡ†Ρƒ ΡƒΡΡ‚Π°Π½Π°Π²Π»ΠΈΠ²Π°ΡŽΡ‚ Π½Π° основС ΠΌΠ°Π³Π½ΠΈΡ‚Π½ΠΎΠ³ΠΎ взаимодСйствия Ρ‚ΠΎΠΊΠΎΠ².

Π˜Π·ΠΌΠ΅Ρ€ΡΡŽΡ‚ силу Ρ‚ΠΎΠΊΠ° Π°ΠΌΠΏΠ΅Ρ€ΠΌΠ΅Ρ‚Ρ€Π°ΠΌΠΈ. ΠŸΡ€ΠΈΠ½Ρ†ΠΈΠΏ устройства этих ΠΏΡ€ΠΈΠ±ΠΎΡ€ΠΎΠ² основан Π½Π° ΠΌΠ°Π³Π½ΠΈΡ‚Π½ΠΎΠΌ дСйствии Ρ‚ΠΎΠΊΠ°.

Π‘ΠΊΠΎΡ€ΠΎΡΡ‚ΡŒ упорядочСнного двиТСния элСктронов Π² ΠΏΡ€ΠΎΠ²ΠΎΠ΄Π½ΠΈΠΊΠ΅. НайдСм ΡΠΊΠΎΡ€ΠΎΡΡ‚ΡŒ упорядочСнного пСрСмСщСния элСктронов Π² мСталличСском ΠΏΡ€ΠΎΠ²ΠΎΠ΄Π½ΠΈΠΊΠ΅. Богласно Ρ„ΠΎΡ€ΠΌΡƒΠ»Π΅ v = I /enS, Π³Π΄Π΅ Π΅ — ΠΌΠΎΠ΄ΡƒΠ»ΡŒ заряда элСктрона. ΠŸΡƒΡΡ‚ΡŒ, Π½Π°ΠΏΡ€ΠΈΠΌΠ΅Ρ€, сила Ρ‚ΠΎΠΊΠ° I = 1 A, Π° ΠΏΠ»ΠΎΡ‰Π°Π΄ΡŒ ΠΏΠΎΠΏΠ΅Ρ€Π΅Ρ‡Π½ΠΎΠ³ΠΎ сСчСния ΠΏΡ€ΠΎΠ²ΠΎΠ΄Π½ΠΈΠΊΠ° S = 10-6 ΠΌ2. ΠœΠΎΠ΄ΡƒΠ»ΡŒ заряда элСктрона Π΅ = 1,6β€’10-19 Кл. Число элСктронов Π² 1 ΠΌ3 ΠΌΠ΅Π΄ΠΈ Ρ€Π°Π²Π½ΠΎ числу Π°Ρ‚ΠΎΠΌΠΎΠ² Π² этом объСмС, Ρ‚Π°ΠΊ ΠΊΠ°ΠΊ ΠΎΠ΄ΠΈΠ½ ΠΈΠ· Π²Π°Π»Π΅Π½Ρ‚Π½Ρ‹Ρ… элСктронов ΠΊΠ°ΠΆΠ΄ΠΎΠ³ΠΎ Π°Ρ‚ΠΎΠΌΠ° ΠΌΠ΅Π΄ΠΈ ΠΊΠΎΠ»Π»Π΅ΠΊΡ‚ΠΈΠ²ΠΈΠ·ΠΈΡ€ΠΎΠ²Π°Π½ ΠΈ являСтся свободным. Π­Ρ‚ΠΎ число Π΅ΡΡ‚ΡŒ n = 8,5β€’1028 ΠΌ3. Π‘Π»Π΅Π΄ΠΎΠ²Π°Ρ‚Π΅Π»ΡŒΠ½ΠΎ,

ΠžΡ‡Π΅Π²ΠΈΠ΄Π½ΠΎ, Ρ‡Ρ‚ΠΎ ΡΠΊΠΎΡ€ΠΎΡΡ‚ΡŒ упорядочСнного пСрСмСщСния элСктронов ΠΎΡ‡Π΅Π½ΡŒ ΠΌΠ°Π»Π°.

Основная количСствСнная характСристика элСктричСского Ρ‚ΠΎΠΊΠ° — сила Ρ‚ΠΎΠΊΠ°. Она опрСдСляСтся элСктричСским зарядом, пСрСносимым Ρ‡Π΅Ρ€Π΅Π· ΠΏΠΎΠΏΠ΅Ρ€Π΅Ρ‡Π½ΠΎΠ΅ сСчСниС ΠΏΡ€ΠΎΠ²ΠΎΠ΄Π½ΠΈΠΊΠ° Π·Π° Π΅Π΄ΠΈΠ½ΠΈΡ†Ρƒ Π²Ρ€Π΅ΠΌΠ΅Π½ΠΈ. Π‘ΠΊΠΎΡ€ΠΎΡΡ‚ΡŒ заряТСнных частиц (элСктронов) Π² ΠΏΡ€ΠΎΠ²ΠΎΠ΄Π½ΠΈΠΊΠ΅ ΠΎΡ‡Π΅Π½ΡŒ ΠΌΠ°Π»Π° — ΠΎΠΊΠΎΠ»ΠΎ 0,1 ΠΌΠΌ/с.


Условия сущСствования постоянного элСктричСского Ρ‚ΠΎΠΊΠ°.

 Для сущСствования постоянного элСктричСского Ρ‚ΠΎΠΊΠ° Π½Π΅ΠΎΠ±Ρ…ΠΎΠ΄ΠΈΠΌΠΎ Π½Π°Π»ΠΈΡ‡ΠΈΠ΅ свободных заряТСнных частиц ΠΈ Π½Π°Π»ΠΈΡ‡ΠΈΠ΅ источника Ρ‚ΠΎΠΊΠ°, Π² ΠΊΠΎΡ‚ΠΎΡ€ΠΎΠΌ осущСствляСтся ΠΏΡ€Π΅ΠΎΠ±Ρ€Π°Π·ΠΎΠ²Π°Π½ΠΈΠ΅ ΠΊΠ°ΠΊΠΎΠ³ΠΎ-Π»ΠΈΠ±ΠΎ Π²ΠΈΠ΄Π° энСргии Π² ΡΠ½Π΅Ρ€Π³ΠΈΡŽ элСктричСского поля.

Π˜ΡΡ‚ΠΎΡ‡Π½ΠΈΠΊ Ρ‚ΠΎΠΊΠ° — устройство, Π² ΠΊΠΎΡ‚ΠΎΡ€ΠΎΠΌ осущСствляСтся ΠΏΡ€Π΅ΠΎΠ±Ρ€Π°Π·ΠΎΠ²Π°Π½ΠΈΠ΅ ΠΊΠ°ΠΊΠΎΠ³ΠΎ-Π»ΠΈΠ±ΠΎ Π²ΠΈΠ΄Π° энСргии Π² ΡΠ½Π΅Ρ€Π³ΠΈΡŽ элСктричСского поля. Π’ источникС Ρ‚ΠΎΠΊΠ° Π½Π° заряТСнныС частицы Π² Π·Π°ΠΌΠΊΠ½ΡƒΡ‚ΠΎΠΉ Ρ†Π΅ΠΏΠΈ Π΄Π΅ΠΉΡΡ‚Π²ΡƒΡŽΡ‚ сторонниС силы. ΠŸΡ€ΠΈΡ‡ΠΈΠ½Ρ‹ возникновСния сторонних сил Π² Ρ€Π°Π·Π»ΠΈΡ‡Π½Ρ‹Ρ… источниках Ρ‚ΠΎΠΊΠ° Ρ€Π°Π·Π»ΠΈΡ‡Π½Ρ‹. НапримСр, Π² аккумуляторах ΠΈ Π³Π°Π»ΡŒΠ²Π°Π½ΠΈΡ‡Π΅ΡΠΊΠΈΡ… элСмСнтах сторонниС силы Π²ΠΎΠ·Π½ΠΈΠΊΠ°ΡŽΡ‚ благодаря ΠΏΡ€ΠΎΡ‚Π΅ΠΊΠ°Π½ΠΈΡŽ химичСских Ρ€Π΅Π°ΠΊΡ†ΠΈΠΉ, Π² Π³Π΅Π½Π΅Ρ€Π°Ρ‚ΠΎΡ€Π°Ρ… элСктростанций ΠΎΠ½ΠΈ Π²ΠΎΠ·Π½ΠΈΠΊΠ°ΡŽΡ‚Β  ΠΏΡ€ΠΈ Π΄Π²ΠΈΠΆΠ΅Π½ΠΈΠΈ ΠΏΡ€ΠΎΠ²ΠΎΠ΄Π½ΠΈΠΊΠ° Π² ΠΌΠ°Π³Π½ΠΈΡ‚Π½ΠΎΠΌ ΠΏΠΎΠ»Π΅, Π² фотоэлСмСнтах — ΠΏΡ€ΠΈ дСйствия свСта Π½Π° элСктроны Π² ΠΌΠ΅Ρ‚Π°Π»Π»Π°Ρ… ΠΈ ΠΏΠΎΠ»ΡƒΠΏΡ€ΠΎΠ²ΠΎΠ΄Π½ΠΈΠΊΠ°Ρ….

Π—Π°ΠΊΠΎΠ½ Ома для участка Ρ†Π΅ΠΏΠΈ.

НСмСцкий ΡƒΡ‡Π΅Π½Ρ‹ΠΉ Π“Π΅ΠΎΡ€Π³ Ом Π² 1827 Π³. связал Π²ΠΎΠ΅Π΄ΠΈΠ½ΠΎ Ρ‚Ρ€ΠΈ физичСскиС Π²Π΅Π»ΠΈΡ‡ΠΈΠ½Ρ‹ ΠΈ Π²Ρ‹Π²Π΅Π» Π·Π°ΠΊΠΎΠ½, ΠΊΠΎΡ‚ΠΎΡ€Ρ‹ΠΉ Π½Π°Π·Π²Π°Π»ΠΈ Π΅Π³ΠΎ ΠΈΠΌΠ΅Π½Π΅ΠΌ. Π—Π°ΠΊΠΎΠ½ Ома для участка Ρ†Π΅ΠΏΠΈ гласит:

Π‘ΠΈΠ»Π° Ρ‚ΠΎΠΊΠ° Π² участкС Ρ†Π΅ΠΏΠΈ прямо ΠΏΡ€ΠΎΠΏΠΎΡ€Ρ†ΠΈΠΎΠ½Π°Π»ΡŒΠ½Π° Π½Π°ΠΏΡ€ΡΠΆΠ΅Π½ΠΈΡŽ Π½Π° ΠΊΠΎΠ½Ρ†Π°Ρ… этого участка ΠΈ ΠΎΠ±Ρ€Π°Ρ‚Π½ΠΎ ΠΏΡ€ΠΎΠΏΠΎΡ€Ρ†ΠΈΠΎΠ½Π°Π»ΡŒΠ½Π° Π΅Π³ΠΎ ΡΠΎΠΏΡ€ΠΎΡ‚ΠΈΠ²Π»Π΅Π½ΠΈΡŽ.

I=U/R,

Π³Π΄Π΅ I – сила Ρ‚ΠΎΠΊΠ°, U – напряТСниС, R – сопротивлСниС.

Β ΠŸΠΎΡΠ»Π΅Π΄ΠΎΠ²Π°Ρ‚Π΅Π»ΡŒΠ½ΠΎΠ΅ ΠΈ ΠΏΠ°Ρ€Π°Π»Π»Π΅Π»ΡŒΠ½ΠΎΠ΅ соСдинСниС ΠΏΡ€ΠΎΠ²ΠΎΠ΄Π½ΠΈΠΊΠΎΠ².

 ЭлСктричСская Ρ†Π΅ΠΏΡŒ Π²ΠΊΠ»ΡŽΡ‡Π°Π΅Ρ‚ Π² сСбя источника Ρ‚ΠΎΠΊΠ° ΠΈ ΠΏΡ€ΠΎΠ²ΠΎΠ΄Π½ΠΈΠΊΠΈ (ΠΏΠΎΡ‚Ρ€Π΅Π±ΠΈΡ‚Π΅Π»ΠΈ, рСзисторы ΠΈ Π΄Ρ€), ΠΊΠΎΡ‚ΠΎΡ€Ρ‹Π΅ ΠΌΠΎΠ³ΡƒΡ‚ соСдинятся  ΠΏΠΎΡΠ»Π΅Π΄ΠΎΠ²Π°Ρ‚Π΅Π»ΡŒΠ½ΠΎ ΠΈΠ»ΠΈ ΠΏΠ°Ρ€Π°Π»Π»Π΅Π»ΡŒΠ½ΠΎ.

БмСшанноС соСдинСниС — комбинация  ΠΏΠ°Ρ€Π°Π»Π»Π΅Π»ΡŒΠ½ΠΎΠ³ΠΎ ΠΈ ΠΏΠΎΡΠ»Π΅Π΄ΠΎΠ²Π°Ρ‚Π΅Π»ΡŒΠ½ΠΎΠ³ΠΎΒ  соСдинСний.



РСшСниС Π·Π°Π΄Π°Ρ‡ Π½Π° Π·Π°ΠΊΠΎΠ½ Ома для участка ΠΈ ΠΏΠΎΠ»Π½ΠΎΠΉ Ρ†Π΅ΠΏΠΈ

РСшСниС Π·Π°Π΄Π°Ρ‡ Π½Π° Π·Π°ΠΊΠΎΠ½ Ома сводится ΠΊ Π½Π°Ρ…ΠΎΠΆΠ΄Π΅Π½ΠΈΡŽ ΠΎΠ΄Π½ΠΎΠΉ ΠΈΠ· Ρ‚Ρ€Π΅Ρ… нСизвСстных ΡΠΎΡΡ‚Π°Π²Π»ΡΡŽΡ‰ΠΈΡ…: Ρ‚ΠΎΠΊΠ°, сопротивлСния ΠΈΠ»ΠΈ напряТСния. Π‘Π°ΠΌ ΠΆΠ΅ Π·Π°ΠΊΠΎΠ½ описываСт, ΠΊΠ°ΠΊ ΠΎΠ½ΠΈ соотносятся ΠΌΠ΅ΠΆΠ΄Ρƒ собой.

Напомним, Ρ‡Ρ‚ΠΎ согласно Π·Π°ΠΊΠΎΠ½Ρƒ Ома сила Ρ‚ΠΎΠΊΠ° прямо ΠΏΡ€ΠΎΠΏΠΎΡ€Ρ†ΠΈΠΎΠ½Π°Π»ΡŒΠ½Π° Π½Π°ΠΏΡ€ΡΠΆΠ΅Π½ΠΈΡŽ ΠΈ ΠΎΠ±Ρ€Π°Ρ‚Π½ΠΎ ΠΏΡ€ΠΎΠΏΠΎΡ€Ρ†ΠΈΠΎΠ½Π°Π»ΡŒΠ½Π° ΡΠΎΠΏΡ€ΠΎΡ‚ΠΈΠ²Π»Π΅Π½ΠΈΡŽ.

Π€ΠΎΡ€ΠΌΡƒΠ»Π° Π·Π°ΠΊΠΎΠ½Π° Ома для участка Ρ†Π΅ΠΏΠΈ:

Π€ΠΎΡ€ΠΌΡƒΠ»Π° Π·Π°ΠΊΠΎΠ½Π° Ома для ΠΏΠΎΠ»Π½ΠΎΠΉ Ρ†Π΅ΠΏΠΈ:

Π—Π°Π΄Π°Ρ‡Π° 1

Π£Ρ‚ΡŽΠ³ Π²ΠΊΠ»ΡŽΡ‡Π΅Π½Π½Ρ‹ΠΉ Π² ΡΠ΅Ρ‚ΡŒ напряТСниСм 220 Π’, потрСбляСт Ρ‚ΠΎΠΊ 1,2 А. ΠžΠΏΡ€Π΅Π΄Π΅Π»ΠΈΡ‚Π΅ сопротивлСниС ΡƒΡ‚ΡŽΠ³Π°.

Π”Π°Π½ΠΎΒ 

U = 220 Π’

I = 1,2 А

РСшСниС

Богласно Π·Π°ΠΊΠΎΠ½Ρƒ Ома для участка Ρ†Π΅ΠΏΠΈ:

Найти

R — ?

ΠžΡ‚Π²Π΅Ρ‚: R = 183,3 Ом.

Π—Π°Π΄Π°Ρ‡Π° 2

К аккумулятору с Π­Π”Π‘ 12 Π’, ΠΏΠΎΠ΄ΠΊΠ»ΡŽΡ‡Π΅Π½Π° Π»Π°ΠΌΠΏΠΎΡ‡ΠΊΠ° ΠΈ Π΄Π²Π° ΠΏΠ°Ρ€Π°Π»Π»Π΅Π»ΡŒΠ½ΠΎ соСдинСнных рСзистора сопротивлСниСм ΠΊΠ°ΠΆΠ΄Ρ‹ΠΉ ΠΏΠΎ 10 Ом. Π˜Π·Π²Π΅ΡΡ‚Π½ΠΎ, Ρ‡Ρ‚ΠΎ Ρ‚ΠΎΠΊ Π² Ρ†Π΅ΠΏΠΈ 0,5 А, Π° сопротивлСниС Π»Π°ΠΌΠΏΠΎΡ‡ΠΊΠΈ R/2. Найти Π²Π½ΡƒΡ‚Ρ€Π΅Π½Π½Π΅Π΅ сопротивлСниС аккумулятора.

Π”Π°Π½ΠΎΒ 

E = 12 Π’

I = 0,5 А

RΠ» = RΡ€/2

RΡ€ = 10 Ом

РСшСниС

НайдСм экв. сопротивлСниС Π΄Π²ΡƒΡ… ΠΏΠ°Ρ€Π°Π»Π»Π΅Π»ΡŒΠ½ΠΎ соСдинённых рСзисторов:

Π‘ΠΎΠΏΡ€ΠΎΡ‚ΠΈΠ²Π»Π΅Π½ΠΈΠ΅ Π»Π°ΠΌΠΏΠΎΡ‡ΠΊΠΈ:

Богласно Π·Π°ΠΊΠΎΠ½Ρƒ Ома для ΠΏΠΎΠ»Π½ΠΎΠΉ Ρ†Π΅ΠΏΠΈ:

Найти

r — ?

ΠžΡ‚Π²Π΅Ρ‚: r = 14 Ом.

Π—Π°Π΄Π°Ρ‡Π° 3

К участку Ρ†Π΅ΠΏΠΈ с напряТСниСм 12 Π’ Ρ‡Π΅Ρ€Π΅Π· рСзистор сопротивлСниСм 2 Ом ΠΏΠΎΠ΄ΠΊΠ»ΡŽΡ‡Π΅Π½Ρ‹ Π΄Π΅ΡΡΡ‚ΡŒ ΠΎΠ΄ΠΈΠ½Π°ΠΊΠΎΠ²Ρ‹Ρ… Π»Π°ΠΌΠΏΠΎΡ‡Π΅ΠΊ сопротивлСниСм 10 Ом. Найти напряТСниС Π½Π° ΠΊΠ°ΠΆΠ΄ΠΎΠΉ Π»Π°ΠΌΠΏΠΎΡ‡ΠΊΠ΅.

Π”Π°Π½ΠΎΒ 

UΠΎΠ±Ρ‰ = 10 Π’

RΡ€ = 2 Ом

Rл = 10 Ом

РСшСниС

Π’Π°ΠΊ ΠΊΠ°ΠΊ Π»Π°ΠΌΠΏΠΎΡ‡ΠΊΠΈ ΠΏΠΎΠ΄ΠΊΠ»ΡŽΡ‡Π΅Π½Ρ‹ ΠΏΠ°Ρ€Π°Π»Π»Π΅Π»ΡŒΠ½ΠΎ, напряТСниС Π½Π° Π½ΠΈΡ… Π±ΡƒΠ΄Π΅Ρ‚ ΠΎΠ΄ΠΈΠ½Π°ΠΊΠΎΠ²Ρ‹ΠΌ, согласно Π·Π°ΠΊΠΎΠ½Ρƒ Ома для участка Ρ†Π΅ΠΏΠΈ:

ΠŸΡ€ΠΈ ΠΏΠΎΡΠ»Π΅Π΄ΠΎΠ²Π°Ρ‚Π΅Π»ΡŒΠ½ΠΎΠΌ соСдинСнии Ρ‚ΠΎΠΊ Π² Ρ†Π΅ΠΏΠΈ ΠΎΠ±Ρ‰ΠΈΠΉ:

Π’Ρ‹Ρ€Π°Π·ΠΈΠΌ UΠ» Ρ‡Π΅Ρ€Π΅Π· UΠΎΠ±Ρ‰:

НайдСм Rэкв:

ΠžΠΊΠΎΠ½Ρ‡Π°Ρ‚Π΅Π»ΡŒΠ½ΠΎ ΠΏΠΎΠ»ΡƒΡ‡ΠΈΠΌ:

Найти

UΠ» — ?

ΠžΡ‚Π²Π΅Ρ‚: UΠ» = 4 Π’.

Π—Π°Π΄Π°Ρ‡Π° 4

Как ΠΎΠΏΡ€Π΅Π΄Π΅Π»ΠΈΡ‚ΡŒ Π΄Π»ΠΈΠ½Ρƒ ΠΌΠΎΡ‚ΠΊΠ° ΠΌΠ΅Π΄Π½ΠΎΠΉ ΠΏΡ€ΠΎΠ²ΠΎΠ»ΠΎΠΊΠΈ, Π½Π΅ разматывая Π΅Π³ΠΎ?

РСшСниС:

Для Ρ€Π΅ΡˆΠ΅Π½ΠΈΡ Π΄Π°Π½Π½ΠΎΠΉ Π·Π°Π΄Π°Ρ‡ΠΈ Π½Π΅ΠΎΠ±Ρ…ΠΎΠ΄ΠΈΠΌΠΎ Π²ΠΎΡΠΏΠΎΠ»ΡŒΠ·ΠΎΠ²Π°Ρ‚ΡŒΡΡ Ρ„ΠΎΡ€ΠΌΡƒΠ»ΠΎΠΉ:

ΠΎΡ‚ΡΡŽΠ΄Π° Π΄Π»ΠΈΠ½Π° ΠΏΡ€ΠΎΠ²ΠΎΠ»ΠΎΠΊΠΈ

Π’ этой Ρ„ΠΎΡ€ΠΌΡƒΠ»Π΅, l – Π΄Π»ΠΈΠ½Π° ΠΏΡ€ΠΎΠ²ΠΎΠ»ΠΎΠΊΠΈ, R – сопротивлСниС, S – ΠΏΠ»ΠΎΡ‰Π°Π΄ΡŒ ΠΏΠΎΠΏΠ΅Ρ€Π΅Ρ‡Π½ΠΎΠ³ΠΎ сСчСния, ρ – ΡƒΠ΄Π΅Π»ΡŒΠ½ΠΎΠ΅ сопротивлСниС ΠΌΠ΅Ρ‚Π°Π»Π»ΠΎΠ², Π² Π΄Π°Π½Π½ΠΎΠΌ случаС ρ для ΠΌΠ΅Π΄ΠΈ Ρ€Π°Π²Π½ΠΎ 0.0175 Ом/ΠΌ.

Π‘ΠΎΠΏΡ€ΠΎΡ‚ΠΈΠ²Π»Π΅Π½ΠΈΠ΅ R ΠΏΡ€ΠΎΠ²ΠΎΠ»ΠΎΠΊΠΈ ΠΌΠΎΠΆΠ½ΠΎ ΠΈΠ·ΠΌΠ΅Ρ€ΠΈΡ‚ΡŒ с ΠΏΠΎΠΌΠΎΡ‰ΡŒΡŽ ΠΎΠΌΠΌΠ΅Ρ‚Ρ€Π°, Π° ΠΏΠ»ΠΎΡ‰Π°Π΄ΡŒ S с ΠΏΠΎΠΌΠΎΡ‰ΡŒΡŽ ΡˆΡ‚Π°Π½Π³Π΅Π½Ρ†ΠΈΡ€ΠΊΡƒΠ»Ρ, ΠΈΠ·ΠΌΠ΅Ρ€ΠΈΠ² Β Π΄ΠΈΠ°ΠΌΠ΅Ρ‚Ρ€ ΠΏΡ€ΠΎΠ²ΠΎΠ»ΠΎΠΊΠΈ ΠΈ ΠΏΠΎ Ρ„ΠΎΡ€ΠΌΡƒΠ»Π΅ Ξ r

2 вычислив Π΅Π΅ Π·Π½Π°Ρ‡Π΅Π½ΠΈΠ΅. Π—Π½Π°Ρ‡Π΅Π½ΠΈΠ΅ ΡƒΠ΄Π΅Π»ΡŒΠ½ΠΎΠ³ΠΎ сопротивлСния ρ Π½Π΅ Ρ‚ΠΎΠ»ΡŒΠΊΠΎ для ΠΌΠ΅Π΄ΠΈ, Π½ΠΎ ΠΈ Π΄Ρ€ΡƒΠ³ΠΈΡ… ΠΌΠ΅Ρ‚Π°Π»Π»ΠΎΠ² ΠΌΠΎΠΆΠ½ΠΎ Π½Π°ΠΉΡ‚ΠΈ Π² справочникС, ΠΈΠ»ΠΈ Ρ‚ΡƒΡ‚. ΠŸΠΎΠ΄ΡΡ‚Π°Π²ΠΈΠ² всС извСстныС Π²Π΅Π»ΠΈΡ‡ΠΈΠ½Ρ‹ Π² Ρ„ΠΎΡ€ΠΌΡƒΠ»Ρƒ, ΠΏΡ€ΠΈΠ²Π΅Π΄Π΅Π½Π½ΡƒΡŽ Π²Ρ‹ΡˆΠ΅, ΠΏΠΎΠ»ΡƒΡ‡ΠΈΠΌ Π΄Π»ΠΈΠ½Ρƒ ΠΏΡ€ΠΎΠ²ΠΎΠ»ΠΎΠΊΠΈ.

Π—Π°Π΄Π°Ρ‡Π° 5

НачСртитС схСму элСктричСской Ρ†Π΅ΠΏΠΈ, состоящСй ΠΈΠ· источника Ρ‚ΠΎΠΊΠ°, Π²Ρ‹ΠΊΠ»ΡŽΡ‡Π°Ρ‚Π΅Π»Ρ ΠΈ Π΄Π²ΡƒΡ… Π»Π°ΠΌΠΏ, Π²ΠΊΠ»ΡŽΡ‡Π΅Π½Π½Ρ‹Ρ… ΠΏΠ°Ρ€Π°Π»Π»Π΅Π»ΡŒΠ½ΠΎ. Π§Ρ‚ΠΎ ΠΏΡ€ΠΎΠΈΠ·ΠΎΠΉΠ΄Π΅Ρ‚ Π² Ρ†Π΅ΠΏΠΈ ΠΏΡ€ΠΈ ΠΏΠ΅Ρ€Π΅Π³ΠΎΡ€Π°Π½ΠΈΠΈ ΠΎΠ΄Π½ΠΎΠΉ Π»Π°ΠΌΠΏΡ‹?

РСшСниС:

ΠŸΡ€ΠΈ ΠΏΠ΅Ρ€Π΅Π³ΠΎΡ€Π°Π½ΠΈΠΈ ΠΎΠ΄Π½ΠΎΠΉ ΠΈΠ· Π»Π°ΠΌΠΏΠΎΡ‡Π΅ΠΊ, вторая Π±ΡƒΠ΄Π΅Ρ‚ Π³ΠΎΡ€Π΅Ρ‚ΡŒ, Ρ‚Π°ΠΊ ΠΊΠ°ΠΊ, ΠΏΡ€ΠΈ ΠΏΠ°Ρ€Π°Π»Π»Π΅Π»ΡŒΠ½ΠΎΠΌ Π²ΠΊΠ»ΡŽΡ‡Π΅Π½ΠΈΠΈ ΠΏΡ€ΠΎΠ²ΠΎΠ΄Π½ΠΈΠΊΠΎΠ² Ρ‚ΠΎΠΊΠΈ I1 ΠΈ I2 проходящиС Ρ‡Π΅Ρ€Π΅Π· Π½ΠΈΡ… Π½Π΅ зависят Π΄Ρ€ΡƒΠ³ ΠΎΡ‚ Π΄Ρ€ΡƒΠ³Π° ΠΈ ΠΏΡ€ΠΈ Ρ€Π°Π·Ρ€Ρ‹Π²Π΅ ΠΏΠ°Ρ€Π°Π»Π»Π΅Π»ΡŒΠ½ΠΎΠΉ Ρ†Π΅ΠΏΠΎΡ‡ΠΊΠΈ Ρ‚ΠΎΠΊ ΠΏΡ€ΠΎΠ΄ΠΎΠ»ΠΆΠ°Π΅Ρ‚ ΠΏΡ€ΠΎΡ‚Π΅ΠΊΠ°Ρ‚ΡŒ.

  • ΠŸΡ€ΠΎΡΠΌΠΎΡ‚Ρ€ΠΎΠ²: 43412
  • Π—Π°ΠΊΠΎΠ½ Ома для ΠΏΠΎΠ»Π½ΠΎΠΉ Ρ†Π΅ΠΏΠΈ

    1. Π˜ΡΡ‚ΠΎΡ‡Π½ΠΈΠΊ Ρ‚ΠΎΠΊΠ°

    ΠŸΡ€ΠΈ ΠΏΡ€ΠΎΡ…ΠΎΠΆΠ΄Π΅Π½ΠΈΠΈ Ρ‚ΠΎΠΊΠ° Π² ΠΏΡ€ΠΎΠ²ΠΎΠ΄Π½ΠΈΠΊΠ΅ выдСляСтся Π½Π΅ΠΊΠΎΡ‚ΠΎΡ€ΠΎΠ΅ количСство Ρ‚Π΅ΠΏΠ»ΠΎΡ‚Ρ‹. Богласно Π·Π°ΠΊΠΎΠ½Ρƒ сохранСния энСргии ΠΏΡ€ΠΈ этом Π² ΡΠ»Π΅ΠΊΡ‚Ρ€ΠΈΡ‡Π΅ΡΠΊΡƒΡŽ Ρ†Π΅ΠΏΡŒ Π΄ΠΎΠ»ΠΆΠ½Π° ΠΏΠΎΡΡ‚ΡƒΠΏΠ°Ρ‚ΡŒ энСргия.

    ΠœΠΎΠΆΠ΅Ρ‚ Π»ΠΈ источником этой энСргии Π±Ρ‹Ρ‚ΡŒ элСктростатичСскоС ΠΏΠΎΠ»Π΅? НСт, Π½Π΅ ΠΌΠΎΠΆΠ΅Ρ‚, ΠΏΠΎΡ‚ΠΎΠΌΡƒ Ρ‡Ρ‚ΠΎ ΠΏΡ€ΠΈ ΠΏΠ΅Ρ€Π΅ΠΌΠ΅Ρ‰Π΅Π½ΠΈΠΈ заряда вдоль всСй Ρ†Π΅ΠΏΠΈ, Ρ‚ΠΎ Π΅ΡΡ‚ΡŒ ΠΏΠΎ Π·Π°ΠΌΠΊΠ½ΡƒΡ‚ΠΎΠΉ Ρ‚Ρ€Π°Π΅ΠΊΡ‚ΠΎΡ€ΠΈΠΈ, Ρ€Π°Π±ΠΎΡ‚Π° элСктростатичСского поля Ρ€Π°Π²Π½Π° ΠΊΡƒΠ»ΡŽ.

    Π‘Π»Π΅Π΄ΠΎΠ²Π°Ρ‚Π΅Π»ΡŒΠ½ΠΎ, для сущСствования Ρ‚ΠΎΠΊΠ° Π² Π·Π°ΠΌΠΊΠ½ΡƒΡ‚ΠΎΠΉ Ρ†Π΅ΠΏΠΈ Π² Π½Π΅ΠΉ Π΄ΠΎΠ»ΠΆΠ΅Π½ Π±Ρ‹Ρ‚ΡŒ участок, Π½Π° ΠΊΠΎΡ‚ΠΎΡ€ΠΎΠΌ свободныС заряды двиТутся ΠΏΡ€ΠΎΡ‚ΠΈΠ² сил элСктростатичСского поля. Π’Π°ΠΊΠΈΠΌ участком Ρ†Π΅ΠΏΠΈ являСтся источник Ρ‚ΠΎΠΊΠ° (рис. 59.1).

    Π’ источникС Ρ‚ΠΎΠΊΠ° Π½Π° свободныС заряды Π΄Π΅ΠΉΡΡ‚Π²ΡƒΡŽΡ‚ силы, ΠΊΠΎΡ‚ΠΎΡ€Ρ‹Π΅ ΠΈΠΌΠ΅ΡŽΡ‚ Π½Π΅ ΡΠ»Π΅ΠΊΡ‚Ρ€ΠΎΡΡ‚Π°Ρ‚ΠΈΡ‡Π΅ΡΠΊΡƒΡŽ ΠΏΡ€ΠΈΡ€ΠΎΠ΄Ρƒ. Π˜Ρ… Π½Π°Π·Ρ‹Π²Π°ΡŽΡ‚ сторонними силами. Π’ Ρ€Π΅Π·ΡƒΠ»ΡŒΡ‚Π°Ρ‚Π΅ дСйствия сторонних сил происходит Ρ€Π°Π·Π΄Π΅Π»Π΅Π½ΠΈΠ΅ зарядов: Π½Π° ΠΎΠ΄Π½ΠΎΠΌ полюсС источника Ρ‚ΠΎΠΊΠ° накапливаСтся ΠΏΠΎΠ»ΠΎΠΆΠΈΡ‚Π΅Π»ΡŒΠ½Ρ‹ΠΉ заряд, Π° Π½Π° Π΄Ρ€ΡƒΠ³ΠΎΠΌ – ΠΎΡ‚Ρ€ΠΈΡ†Π°Ρ‚Π΅Π»ΡŒΠ½Ρ‹ΠΉ. ВслСдствиС этого Π²ΠΎΠ·Π½ΠΈΠΊΠ°Π΅Ρ‚ элСктростатичСскоС ΠΏΠΎΠ»Π΅, ΠΊΠΎΡ‚ΠΎΡ€ΠΎΠ΅ Π΄Π²ΠΈΠΆΠ΅Ρ‚ свободныС заряды Π² элСктричСской Ρ†Π΅ΠΏΠΈ Π²Π½Π΅ источника Ρ‚ΠΎΠΊΠ°, Ρ‚ΠΎ Π΅ΡΡ‚ΡŒ Π²ΠΎ внСшнСй Ρ†Π΅ΠΏΠΈ.

    Π’ химичСских источниках Ρ‚ΠΎΠΊΠ° сторонниС силы ΠΈΠΌΠ΅ΡŽΡ‚ Ρ…ΠΈΠΌΠΈΡ‡Π΅ΡΠΊΡƒΡŽ ΠΏΡ€ΠΈΡ€ΠΎΠ΄Ρƒ. НапримСр, Ссли ΠΏΠΎΠ³Ρ€ΡƒΠ·ΠΈΡ‚ΡŒ Ρ†ΠΈΠ½ΠΊΠΎΠ²Ρ‹ΠΉ ΠΈ ΠΌΠ΅Π΄Π½Ρ‹ΠΉ элСктроды Π² ΡΠ΅Ρ€Π½ΡƒΡŽ кислоту, Ρ‚ΠΎ ΠΏΠΎΠ»ΠΎΠΆΠΈΡ‚Π΅Π»ΡŒΠ½Ρ‹Π΅ ΠΈΠΎΠ½Ρ‹ Ρ†ΠΈΠ½ΠΊΠ° Π±ΡƒΠ΄ΡƒΡ‚ Ρ‡Π°Ρ‰Π΅ ΠΏΠΎΠΊΠΈΠ΄Π°Ρ‚ΡŒ элСктрод, Ρ‡Π΅ΠΌ ΠΏΠΎΠ»ΠΎΠΆΠΈΡ‚Π΅Π»ΡŒΠ½Ρ‹Π΅ ΠΈΠΎΠ½Ρ‹ ΠΌΠ΅Π΄ΠΈ. Π’ Ρ€Π΅Π·ΡƒΠ»ΡŒΡ‚Π°Ρ‚Π΅ ΠΌΠ΅ΠΆΠ΄Ρƒ ΠΌΠ΅Π΄Π½Ρ‹ΠΌ ΠΈ Ρ†ΠΈΠ½ΠΊΠΎΠ²Ρ‹ΠΌ элСктродами Π²ΠΎΠ·Π½ΠΈΠΊΠ½Π΅Ρ‚ Ρ€Π°Π·Π½ΠΎΡΡ‚ΡŒ ΠΏΠΎΡ‚Π΅Π½Ρ†ΠΈΠ°Π»ΠΎΠ²: ΠΏΠΎΡ‚Π΅Π½Ρ†ΠΈΠ°Π» ΠΌΠ΅Π΄Π½ΠΎΠ³ΠΎ элСктрода Π±ΡƒΠ΄Π΅Ρ‚ большС, Ρ‡Π΅ΠΌ Ρ†ΠΈΠ½ΠΊΠΎΠ²ΠΎΠ³ΠΎ. ΠœΠ΅Π΄Π½Ρ‹ΠΉ элСктрод станСт ΠΏΠΎΠ»ΠΎΠΆΠΈΡ‚Π΅Π»ΡŒΠ½Ρ‹ΠΌ полюсом источника Ρ‚ΠΎΠΊΠ°, Π° Ρ†ΠΈΠ½ΠΊΠΎΠ²Ρ‹ΠΉ – ΠΎΡ‚Ρ€ΠΈΡ†Π°Ρ‚Π΅Π»ΡŒΠ½Ρ‹ΠΌ.

    Π’ Π³Π΅Π½Π΅Ρ€Π°Ρ‚ΠΎΡ€Π°Ρ… элСктростанций сторонними силами ΡΠ²Π»ΡΡŽΡ‚ΡΡ силы, Π΄Π΅ΠΉΡΡ‚Π²ΡƒΡŽΡ‰ΠΈΠ΅ Π½Π° свободныС элСктроны Π² ΠΌΠ΅Ρ‚Π°Π»Π»Π΅ со стороны Π²ΠΈΡ…Ρ€Π΅Π²ΠΎΠ³ΠΎ элСктричСского поля, ΠΏΠΎΡ€ΠΎΠΆΠ΄Π°Π΅ΠΌΠΎΠ³ΠΎ ΠΏΠ΅Ρ€Π΅ΠΌΠ΅Π½Π½Ρ‹ΠΌ ΠΌΠ°Π³Π½ΠΈΡ‚Π½Ρ‹ΠΌ ΠΏΠΎΠ»Π΅ΠΌ. Π Π°Π±ΠΎΡ‚Π° Π²ΠΈΡ…Ρ€Π΅Π²ΠΎΠ³ΠΎ элСктричСского поля ΠΏΠΎ ΠΏΠ΅Ρ€Π΅ΠΌΠ΅Ρ‰Π΅Π½ΠΈΡŽ заряда вдоль Π·Π°ΠΌΠΊΠ½ΡƒΡ‚ΠΎΠ³ΠΎ ΠΊΠΎΠ½Ρ‚ΡƒΡ€Π° Π½Π΅ Ρ€Π°Π²Π½Π° Π½ΡƒΠ»ΡŽ. ДСйствиС Π³Π΅Π½Π΅Ρ€Π°Ρ‚ΠΎΡ€ΠΎΠ² Ρ‚ΠΎΠΊΠ° ΠΌΡ‹ рассмотрим Π² курсС Ρ„ΠΈΠ·ΠΈΠΊΠΈ 11-Π³ΠΎ класса.

    ЭлСктродвиТущая сила источника Ρ‚ΠΎΠΊΠ°

    Π’ источникС Ρ‚ΠΎΠΊΠ° сторонниС силы, пСрСмСщая свободныС заряды ΠΏΡ€ΠΎΡ‚ΠΈΠ² дСйствия сил элСктростатичСского поля, ΡΠΎΠ²Π΅Ρ€ΡˆΠ°ΡŽΡ‚ Ρ€Π°Π±ΠΎΡ‚Ρƒ, ΠΊΠΎΡ‚ΠΎΡ€ΡƒΡŽ ΠΌΡ‹ ΠΎΠ±ΠΎΠ·Π½Π°Ρ‡ΠΈΠΌ Aстор.

    Π­Ρ‚Π° Ρ€Π°Π±ΠΎΡ‚Π° ΠΏΡ€ΠΎΠΏΠΎΡ€Ρ†ΠΈΠΎΠ½Π°Π»ΡŒΠ½Π° заряду q, ΠΊΠΎΡ‚ΠΎΡ€Ρ‹ΠΉ пСрСмСщаСтся вдоль Ρ†Π΅ΠΏΠΈ Π·Π° Π΄Π°Π½Π½Ρ‹ΠΉ ΠΏΡ€ΠΎΠΌΠ΅ΠΆΡƒΡ‚ΠΎΠΊ Π²Ρ€Π΅ΠΌΠ΅Π½ΠΈ. ΠŸΠΎΡΡ‚ΠΎΠΌΡƒ ΠΎΡ‚Π½ΠΎΡˆΠ΅Π½ΠΈΠ΅ Ρ€Π°Π±ΠΎΡ‚Ρ‹ сторонних сил ΠΊ Π²Π΅Π»ΠΈΡ‡ΠΈΠ½Π΅ заряда Π½Π΅ зависит Π½ΠΈ ΠΎΡ‚ Aстор, Π½ΠΈ ΠΎΡ‚ q. Π‘Π»Π΅Π΄ΠΎΠ²Π°Ρ‚Π΅Π»ΡŒΠ½ΠΎ, ΠΎΠ½ΠΎ являСтся характСристикой источника Ρ‚ΠΎΠΊΠ°. Π­Ρ‚ΠΎ ΠΎΡ‚Π½ΠΎΡˆΠ΅Π½ΠΈΠ΅ Π½Π°Π·Ρ‹Π²Π°ΡŽΡ‚ элСктродвиТущСй силой источника (Π­Π”Π‘) ΠΈ ΠΎΠ±ΠΎΠ·Π½Π°Ρ‡Π°ΡŽΡ‚ ΞΎ:

    ΞΎ = Aстор/q. Β Β Β Β (1)

    (Π­Ρ‚ΠΎ Π½Π°Π·Π²Π°Π½ΠΈΠ΅ Π½Π΅ совсСм ΡƒΠ΄Π°Ρ‡Π½ΠΎ, ΠΏΠΎΡ‚ΠΎΠΌΡƒ Ρ‡Ρ‚ΠΎ Π­Π”Π‘ – Π½Π΅ «сила» Π² мСханичСском смыслС, Π° энСргСтичСская характСристика источника.)

    Π­Π”Π‘, ΠΊΠ°ΠΊ ΠΈ напряТСниС, ΠΈΠ·ΠΌΠ΅Ρ€ΡΡŽΡ‚ Π² Π²ΠΎΠ»ΡŒΡ‚Π°Ρ…. НапримСр, Π­Π”Π‘ Π±Π°Ρ‚Π°Ρ€Π΅ΠΉΠΊΠΈ составляСт нСсколько Π²ΠΎΠ»ΡŒΡ‚.

    2.

    Π—Π°ΠΊΠΎΠ½ Ома для ΠΏΠΎΠ»Π½ΠΎΠΉ Ρ†Π΅ΠΏΠΈ

    Если сила Ρ‚ΠΎΠΊΠ° Π² Ρ†Π΅ΠΏΠΈ Ρ€Π°Π²Π½Π° I, Ρ‚ΠΎ Π·Π° врСмя t ΠΏΠΎ Ρ†Π΅ΠΏΠΈ ΠΏΡ€ΠΎΡ…ΠΎΠ΄ΠΈΡ‚ заряд q = It. ΠŸΠΎΡΡ‚ΠΎΠΌΡƒ Ρ„ΠΎΡ€ΠΌΡƒΠ»Ρƒ (1) ΠΌΠΎΠΆΠ½ΠΎ Π·Π°ΠΏΠΈΡΠ°Ρ‚ΡŒ Π² Π²ΠΈΠ΄Π΅

    Aстор = ΞΎIt. Β Β Β Β (2)

    ΠŸΡ€ΠΈ этом Π²ΠΎ внСшнСй Ρ†Π΅ΠΏΠΈ сопротивлСниСм R выдСляСтся количСство Ρ‚Π΅ΠΏΠ»ΠΎΡ‚Ρ‹

    QвнСш = I2Rt,     (3)

    Π° Π²Π½ΡƒΡ‚Ρ€ΠΈ источника Ρ‚ΠΎΠΊΠ° выдСляСтся количСство Ρ‚Π΅ΠΏΠ»ΠΎΡ‚Ρ‹

    QΠ²Π½ΡƒΡ‚Ρ€ = I2rt, Β Β Β Β (4)

    Π³Π΄Π΅ r – сопротивлСниС источника, ΠΊΠΎΡ‚ΠΎΡ€ΠΎΠ΅ Π½Π°Π·Ρ‹Π²Π°ΡŽΡ‚ Π΅Π³ΠΎ Π²Π½ΡƒΡ‚Ρ€Π΅Π½Π½ΠΈΠΌ сопротивлСниСм.

    Из Π·Π°ΠΊΠΎΠ½Π° сохранСния энСргии слСдуСт, Ρ‡Ρ‚ΠΎ

    QвнСш + QΠ²Π½ΡƒΡ‚Ρ€ = Aстор. Β Β Β Β (5)

    ? 1. Π”ΠΎΠΊΠ°ΠΆΠΈΡ‚Π΅, Ρ‡Ρ‚ΠΎ ΠΈΠ· Ρ„ΠΎΡ€ΠΌΡƒΠ» (2) – (5) слСдуСт:

    I = ΞΎ / (R + r). Β Β Β Β (6)

    Π­Ρ‚ΠΎ ΡΠΎΠΎΡ‚Π½ΠΎΡˆΠ΅Π½ΠΈΠ΅ Π½Π°Π·Ρ‹Π²Π°ΡŽΡ‚ Π·Π°ΠΊΠΎΠ½ΠΎΠΌ Ома для ΠΏΠΎΠ»Π½ΠΎΠΉ Ρ†Π΅ΠΏΠΈ.

    Π‘ΡƒΠΌΠΌΡƒ сопротивлСний R + r Π½Π°Π·Ρ‹Π²Π°ΡŽΡ‚ ΠΏΠΎΠ»Π½Ρ‹ΠΌ сопротивлСниСм Ρ†Π΅ΠΏΠΈ.

    ? 2. Π­Π”Π‘ источника Ρ‚ΠΎΠΊΠ° 12 Π’, Π° Π΅Π³ΠΎ Π²Π½ΡƒΡ‚Ρ€Π΅Π½Π½Π΅Π΅ сопротивлСниС Ρ€Π°Π²Π½ΠΎ 2 Ом.
    Π°) Π§Π΅ΠΌΡƒ Ρ€Π°Π²Π½Π° сила Ρ‚ΠΎΠΊΠ° Π² Ρ†Π΅ΠΏΠΈ, Ссли сопротивлСниС внСшнСй Ρ†Π΅ΠΏΠΈ Ρ€Π°Π²Π½ΠΎ 4 Ом?
    Π±) Какова максимально возмоТная сила Ρ‚ΠΎΠΊΠ° Π² Ρ†Π΅ΠΏΠΈ? ΠŸΡ€ΠΈ ΠΊΠ°ΠΊΠΎΠΌ сопротивлСнии внСшнСй Ρ†Π΅ΠΏΠΈ это ΠΈΠΌΠ΅Π΅Ρ‚ мСсто?

    ? 3. ΠŸΡ€ΠΈ внСшнСм сопротивлСнии 2 Ом сила Ρ‚ΠΎΠΊΠ° Π² Ρ†Π΅ΠΏΠΈ Ρ€Π°Π²Π½Π° 1,5 А, Π° ΠΏΡ€ΠΈ внСшнСм сопротивлСнии 4 Ом сила Ρ‚ΠΎΠΊΠ° Ρ€Π°Π²Π½Π° 1 А.
    Π°) Π§Π΅ΠΌΡƒ Ρ€Π°Π²Π½ΠΎ Π²Π½ΡƒΡ‚Ρ€Π΅Π½Π½Π΅Π΅ сопротивлСниС источника?
    Π±) Π§Π΅ΠΌΡƒ Ρ€Π°Π²Π½Π° Π­Π”Π‘ источника?

    НапряТСниС Π½Π° ΠΏΠΎΠ»ΡŽΡΠ°Ρ… источника

    Π—Π°ΠΊΠΎΠ½ Ома для ΠΏΠΎΠ»Π½ΠΎΠΉ Ρ†Π΅ΠΏΠΈ ΠΌΠΎΠΆΠ½ΠΎ Π·Π°ΠΏΠΈΡΠ°Ρ‚ΡŒ Π² Π²ΠΈΠ΄Π΅

    ΞΎ = IR + Ir. Β Β Β Β (7)

    ΠŸΠ΅Ρ€Π²ΠΎΠ΅ слагаСмоС Π² этой Ρ„ΠΎΡ€ΠΌΡƒΠ»Π΅ согласно Π·Π°ΠΊΠΎΠ½Ρƒ Ома для участка Ρ†Π΅ΠΏΠΈ Ρ€Π°Π²Π½ΠΎ Π½Π°ΠΏΡ€ΡΠΆΠ΅Π½ΠΈΡŽ U Π½Π° ΠΏΠΎΠ»ΡŽΡΠ°Ρ… источника Ρ‚ΠΎΠΊΠ°:

    IR = U.

    ΠŸΠΎΡΡ‚ΠΎΠΌΡƒ Ρ„ΠΎΡ€ΠΌΡƒΠ»Ρƒ (7) ΠΌΠΎΠΆΠ½ΠΎ Π·Π°ΠΏΠΈΡΠ°Ρ‚ΡŒ Π² Π²ΠΈΠ΄Π΅

    U = ΞΎ – Ir. Β Β Β Β (8)

    Π€ΠΎΡ€ΠΌΡƒΠ»Π° (8) Π²Ρ‹Ρ€Π°ΠΆΠ°Π΅Ρ‚ Π·Π°Π²ΠΈΡΠΈΠΌΠΎΡΡ‚ΡŒ напряТСния U Π½Π° ΠΏΠΎΠ»ΡŽΡΠ°Ρ… источника Ρ‚ΠΎΠΊΠ° ΠΎΡ‚ силы Ρ‚ΠΎΠΊΠ° I Π² Ρ†Π΅ΠΏΠΈ.

    ΠŸΠΎΡΡ‚Π°Π²ΠΈΠΌ ΠΎΠΏΡ‹Ρ‚
    Π—Π°Π²ΠΈΡΠΈΠΌΠΎΡΡ‚ΡŒ U(I) ΠΌΠΎΠΆΠ½ΠΎ ΠΈΠ·ΠΌΠ΅Ρ€ΠΈΡ‚ΡŒ Π½Π° ΠΎΠΏΡ‹Ρ‚Π΅, измСняя силу Ρ‚ΠΎΠΊΠ° Π² Ρ†Π΅ΠΏΠΈ с ΠΏΠΎΠΌΠΎΡ‰ΡŒΡŽ рСостата (рис. 59.2, Π°, Π±). ΠšΡ€Π°ΡΠ½Π°Ρ пунктирная линия Π½Π° схСмС 59.2, Π± ΠΏΠΎΠΊΠ°Π·Ρ‹Π²Π°Π΅Ρ‚, ΠΊΠ°ΠΊ ΠΈΠ΄Π΅Ρ‚ Ρ‚ΠΎΠΊ Π² рСостатС. НапримСр, Ссли ΠΏΠΎΠ»Π·ΡƒΠ½ΠΎΠΊ рСостата, ΠΈΠ·ΠΎΠ±Ρ€Π°ΠΆΠ΅Π½Π½ΠΎΠ³ΠΎ Π½Π° рисункС 59,2, Π°, ΡΠ΄Π²ΠΈΠ½ΡƒΡ‚ΡŒ Π²ΠΏΡ€Π°Π²ΠΎ, Ρ‚ΠΎ сопротивлСниС рСостата увСличится, ΠΏΠΎΡ‚ΠΎΠΌΡƒ Ρ‡Ρ‚ΠΎ увСличится Π΄Π»ΠΈΠ½Π° ΠΎΠ±ΠΌΠΎΡ‚ΠΊΠΈ, ΠΏΠΎ ΠΊΠΎΡ‚ΠΎΡ€ΠΎΠΉ ΠΈΠ΄Π΅Ρ‚ Ρ‚ΠΎΠΊ.

    ? 4. На рисункС 59.3 ΠΈΠ·ΠΎΠ±Ρ€Π°ΠΆΠ΅Π½ Π³Ρ€Π°Ρ„ΠΈΠΊ зависимости U(I) для Π½Π΅ΠΊΠΎΡ‚ΠΎΡ€ΠΎΠ³ΠΎ источника Ρ‚ΠΎΠΊΠ°.

    Π°) Π§Π΅ΠΌΡƒ Ρ€Π°Π²Π½Π° Π­Π”Π‘ этого источника Ρ‚ΠΎΠΊΠ°?
    Π±) Π§Π΅ΠΌΡƒ Ρ€Π°Π²Π½Π° наибольшая сила Ρ‚ΠΎΠΊΠ°?
    Π²) Π§Π΅ΠΌΡƒ Ρ€Π°Π²Π½ΠΎ Π²Π½ΡƒΡ‚Ρ€Π΅Π½Π½Π΅Π΅ сопротивлСниС источника Ρ‚ΠΎΠΊΠ°?
    Π³) Π§Π΅ΠΌΡƒ Ρ€Π°Π²Π½ΠΎ внСшнСС сопротивлСниС, ΠΊΠΎΠ³Π΄Π° сила Ρ‚ΠΎΠΊΠ° Ρ€Π°Π²Π½Π° Π½ΡƒΠ»ΡŽ?
    Π΄) Π§Π΅ΠΌΡƒ Ρ€Π°Π²Π½ΠΎ внСшнСС сопротивлСниС, ΠΊΠΎΠ³Π΄Π° сила Ρ‚ΠΎΠΊΠ° максимальна?
    Π΅) Π§Π΅ΠΌΡƒ Ρ€Π°Π²Π½ΠΎ внСшнСС сопротивлСниС ΠΏΡ€ΠΈ I = 1,5 А?

    МаксимальноС напряТСниС Π½Π° ΠΏΠΎΠ»ΡŽΡΠ°Ρ… источника Ρ€Π°Π²Π½ΠΎ ΞΎ. Π­Ρ‚ΠΎ ΠΈΠΌΠ΅Π΅Ρ‚ мСсто ΠΏΡ€ΠΈ I = 0. Π‘ΠΈΠ»Π° Ρ‚ΠΎΠΊΠ° Ρ€Π°Π²Π½Π° Π½ΡƒΠ»ΡŽ, ΠΊΠΎΠ³Π΄Π° полюса источника Ρ€Π°Π·ΠΎΠΌΠΊΠ½ΡƒΡ‚Ρ‹ (Π² этом случаС внСшнСС сопротивлСниС Ρ†Π΅ΠΏΠΈ являСтся бСсконСчно большим).

    Π‘Π»Π΅Π΄ΠΎΠ²Π°Ρ‚Π΅Π»ΡŒΠ½ΠΎ, напряТСниС ΠΌΠ΅ΠΆΠ΄Ρƒ Ρ€Π°Π·ΠΎΠΌΠΊΠ½ΡƒΡ‚Ρ‹ΠΌΠΈ полюсами источника Ρ‚ΠΎΠΊΠ° Ρ€Π°Π²Π½ΠΎ Π­Π”Π‘ этого источника.

    МинимальноС ΠΆΠ΅ напряТСниС ΠΌΠ΅ΠΆΠ΄Ρƒ полюсами источника Ρ€Π°Π²Π½ΠΎ Π½ΡƒΠ»ΡŽ. Π­Ρ‚ΠΎ ΠΈΠΌΠ΅Π΅Ρ‚ мСсто ΠΏΡ€ΠΈ ΠΊΠΎΡ€ΠΎΡ‚ΠΊΠΎΠΌ Π·Π°ΠΌΡ‹ΠΊΠ°Π½ΠΈΠΈ, ΠΊΠΎΠ³Π΄Π° внСшнСС сопротивлСниС R = 0. Π’ этом случаС сила Ρ‚ΠΎΠΊΠ° максимальна. Π•Π΅ Π½Π°Π·Ρ‹Π²Π°ΡŽΡ‚ силой Ρ‚ΠΎΠΊΠ° ΠΊΠΎΡ€ΠΎΡ‚ΠΊΠΎΠ³ΠΎ замыкания.

    ? 5. ΠŸΠΎΠΊΠ°ΠΆΠΈΡ‚Π΅, Ρ‡Ρ‚ΠΎ сила Ρ‚ΠΎΠΊΠ° ΠΊΠΎΡ€ΠΎΡ‚ΠΊΠΎΠ³ΠΎ замыкания выраТаСтся Ρ„ΠΎΡ€ΠΌΡƒΠ»ΠΎΠΉ

    IΠΊΠ° = ΞΎ/r. Β Β Β Β (9)

    Подсказка. Π’ΠΎΡΠΏΠΎΠ»ΡŒΠ·ΡƒΠΉΡ‚Π΅ΡΡŒ Π·Π°ΠΊΠΎΠ½ΠΎΠΌ Ома для ΠΏΠΎΠ»Π½ΠΎΠΉ Ρ†Π΅ΠΏΠΈ.

    Из Ρ„ΠΎΡ€ΠΌΡƒΠ»Ρ‹ (9) Π²ΠΈΠ΄Π½ΠΎ, Ρ‡Ρ‚ΠΎ ΠΏΡ€ΠΈ ΠΎΡ‡Π΅Π½ΡŒ ΠΌΠ°Π»ΠΎΠΌ Π²Π½ΡƒΡ‚Ρ€Π΅Π½Π½Π΅ΠΌ сопротивлСнии источника (ΠΊΠ°ΠΊ, Π½Π°ΠΏΡ€ΠΈΠΌΠ΅Ρ€, Ρƒ Π°Π²Ρ‚ΠΎΠΌΠΎΠ±ΠΈΠ»ΡŒΠ½ΠΎΠ³ΠΎ аккумулятора) сила Ρ‚ΠΎΠΊΠ° ΠΊΠΎΡ€ΠΎΡ‚ΠΊΠΎΠ³ΠΎ замыкания Π±ΡƒΠ΄Π΅Ρ‚ ΠΎΡ‡Π΅Π½ΡŒ большой, Ρ‡Ρ‚ΠΎ ΠΌΠΎΠΆΠ΅Ρ‚ вывСсти источник Ρ‚ΠΎΠΊΠ° ΠΈΠ· строя.

    ? 6. Π‘ΠΈΠ»Π° Ρ‚ΠΎΠΊΠ° ΠΏΡ€ΠΈ ΠΊΠΎΡ€ΠΎΡ‚ΠΊΠΎΠΌ Π·Π°ΠΌΡ‹ΠΊΠ°Π½ΠΈΠΈ Π±Π°Ρ‚Π°Ρ€Π΅ΠΉΠΊΠΈ Ρ€Π°Π²Π½Π° 2 А. Когда ΠΊ Π±Π°Ρ‚Π°Ρ€Π΅ΠΉΠΊΠ΅ ΠΏΠΎΠ΄ΠΊΠ»ΡŽΡ‡ΠΈΠ»ΠΈ рСзистор сопротивлСниСм 4 Ом, сила Ρ‚ΠΎΠΊΠ° стала Ρ€Π°Π²Π½ΠΎΠΉ 1 А.
    Π°) Как измСнилось ΠΏΠΎΠ»Π½ΠΎΠ΅ сопротивлСниС Ρ†Π΅ΠΏΠΈ?
    Π±) Π§Π΅ΠΌΡƒ Ρ€Π°Π²Π½ΠΎ Π²Π½ΡƒΡ‚Ρ€Π΅Π½Π½Π΅Π΅ сопротивлСниС Π±Π°Ρ‚Π°Ρ€Π΅ΠΉΠΊΠΈ?

    Π˜Π·ΠΌΠ΅Ρ€ΠΈΠ² напряТСниС Π½Π° ΠΏΠΎΠ»ΡŽΡΠ°Ρ… источника ΠΈ силу Ρ‚ΠΎΠΊΠ° Π² Ρ†Π΅ΠΏΠΈ ΠΏΡ€ΠΈ Π΄Π²ΡƒΡ… Ρ€Π°Π·Π»ΠΈΡ‡Π½Ρ‹Ρ… значСниях сопротивлСния внСшнСй Ρ†Π΅ΠΏΠΈ, ΠΌΠΎΠΆΠ½ΠΎ Π½Π°ΠΉΡ‚ΠΈ Π­Π”Π‘ ΞΎ ΠΈ Π²Π½ΡƒΡ‚Ρ€Π΅Π½Π½Π΅Π΅ сопротивлСниС r источника Ρ‚ΠΎΠΊΠ°. Π­Ρ‚ΠΎ ΠΌΠΎΠΆΠ½ΠΎ ΡΠ΄Π΅Π»Π°Ρ‚ΡŒ графичСски ΠΈ аналитичСски.

    ? 7. ΠŸΡ€ΠΈ силС Ρ‚ΠΎΠΊΠ° Π² Ρ†Π΅ΠΏΠΈ 2 А напряТСниС Π½Π° ΠΏΠΎΠ»ΡŽΡΠ°Ρ… источника Ρ€Π°Π²Π½ΠΎ 8 Π’, Π° ΠΏΡ€ΠΈ силС Ρ‚ΠΎΠΊΠ° 4 А напряТСниС Π½Π° ΠΏΠΎΠ»ΡŽΡΠ°Ρ… Ρ€Π°Π²Π½ΠΎ 4 Π’.
    Π°) ΠŸΠΎΡΡ‚Ρ€ΠΎΠΉΡ‚Π΅ систСму ΠΊΠΎΠΎΡ€Π΄ΠΈΠ½Π°Ρ‚ I, U ΠΈ нанСситС Π΄Π²Π΅ Ρ‚ΠΎΡ‡ΠΊΠΈ Π³Ρ€Π°Ρ„ΠΈΠΊΠ° зависимости U(I) согласно ΠΏΡ€ΠΈΠ²Π΅Π΄Π΅Π½Π½Ρ‹ΠΌ Π΄Π°Π½Π½Ρ‹ΠΌ.
    Π±) ΠŸΡ€ΠΎΠ²Π΅Π΄ΠΈΡ‚Π΅ ΠΏΡ€ΡΠΌΡƒΡŽ Ρ‡Π΅Ρ€Π΅Π· эти Ρ‚ΠΎΡ‡ΠΊΠΈ ΠΈ ΠΎΡ‚ΠΌΠ΅Ρ‚ΡŒΡ‚Π΅ Ρ‚ΠΎΡ‡ΠΊΠΈ пСрСсСчСния этой прямой с осями ΠΊΠΎΠΎΡ€Π΄ΠΈΠ½Π°Ρ‚. Π˜ΡΠΏΠΎΠ»ΡŒΠ·ΡƒΡ этот Π³Ρ€Π°Ρ„ΠΈΠΊ, Π½Π°ΠΉΠ΄ΠΈΡ‚Π΅, Ρ‡Π΅ΠΌΡƒ Ρ€Π°Π²Π½Ρ‹ Π­Π”Π‘, сила Ρ‚ΠΎΠΊΠ° ΠΊΠΎΡ€ΠΎΡ‚ΠΊΠΎΠ³ΠΎ замыкания ΠΈ Π²Π½ΡƒΡ‚Ρ€Π΅Π½Π½Π΅Π΅ сопротивлСниС источника Ρ‚ΠΎΠΊΠ°.
    Π²) Π˜ΡΠΏΠΎΠ»ΡŒΠ·ΡƒΡ ΡƒΡ€Π°Π²Π½Π΅Π½ΠΈΠ΅ (8), ΡΠΎΡΡ‚Π°Π²ΡŒΡ‚Π΅ систСму Π΄Π²ΡƒΡ… ΡƒΡ€Π°Π²Π½Π΅Π½ΠΈΠΉ с двумя нСизвСстными ΞΎ ΠΈ r ΠΈ Ρ€Π΅ΡˆΠΈΡ‚Π΅ Π΅Π΅.

    3. ΠšΠŸΠ” источника Ρ‚ΠΎΠΊΠ°

    Π Π°Π±ΠΎΡ‚Ρƒ Ρ‚ΠΎΠΊΠ° Π²ΠΎ внСшнСй Ρ†Π΅ΠΏΠΈ Π½Π°Π·Ρ‹Π²Π°ΡŽΡ‚ ΠΏΠΎΠ»Π΅Π·Π½ΠΎΠΉ Ρ€Π°Π±ΠΎΡ‚ΠΎΠΉ. ΠžΠ±ΠΎΠ·Π½Π°Ρ‡ΠΈΠΌ Π΅Π΅ AΠΏΠΎΠ». Π˜ΡΠΏΠΎΠ»ΡŒΠ·ΡƒΡ Ρ„ΠΎΡ€ΠΌΡƒΠ»Ρƒ для Ρ€Π°Π±ΠΎΡ‚Ρ‹ Ρ‚ΠΎΠΊΠ°, ΠΏΠΎΠ»ΡƒΡ‡Π°Π΅ΠΌ:

    AΠΏΠΎΠ» = I2Rt.

    ΠŸΠΎΡΠΊΠΎΠ»ΡŒΠΊΡƒ источник ΠΎΠ±Π»Π°Π΄Π°Π΅Ρ‚ Π²Π½ΡƒΡ‚Ρ€Π΅Π½Π½ΠΈΠΌ сопротивлСниСм, полСзная Ρ€Π°Π±ΠΎΡ‚Π° мСньшС Ρ€Π°Π±ΠΎΡ‚Ρ‹ сторонних сил, ΠΏΠΎΡ‚ΠΎΠΌΡƒ Ρ‡Ρ‚ΠΎ Ρ‡Π°ΡΡ‚ΡŒ Ρ€Π°Π±ΠΎΡ‚Ρ‹ сторонних сил расходуСтся Π½Π° Π²Ρ‹Π΄Π΅Π»Π΅Π½ΠΈΠ΅ Π² источникС Ρ‚ΠΎΠΊΠ° количСства Ρ‚Π΅ΠΏΠ»ΠΎΡ‚Ρ‹ I2rt. ΠŸΠΎΡΠΊΠΎΠ»ΡŒΠΊΡƒ

    Aстор = I2Rt + I2rt,

    ΠΏΠΎΠ»ΡƒΡ‡Π°Π΅ΠΌ для ΠΎΡ‚Π½ΠΎΡˆΠ΅Π½ΠΈΡ ΠΏΠΎΠ»Π΅Π·Π½ΠΎΠΉ Ρ€Π°Π±ΠΎΡ‚Ρ‹ ΠΊ Ρ€Π°Π±ΠΎΡ‚Π΅ сторонних сил:

    Ξ· = AΠΏΠΎΠ» / Aстор = (I2Rt) / (I2Rt + I2rt) = R / (R + r).

    Π­Ρ‚ΠΎ ΠΎΡ‚Π½ΠΎΡˆΠ΅Π½ΠΈΠ΅, Π²Ρ‹Ρ€Π°ΠΆΠ΅Π½Π½ΠΎΠ΅ Π² ΠΏΡ€ΠΎΡ†Π΅Π½Ρ‚Π°Ρ…, Π½Π°Π·Ρ‹Π²Π°ΡŽΡ‚ ΠšΠŸΠ” источника Ρ‚ΠΎΠΊΠ°.

    ? 8. ΠŸΡ€ΠΈ ΠΊΠ°ΠΊΠΎΠΌ ΠΎΡ‚Π½ΠΎΡˆΠ΅Π½ΠΈΠΈ внСшнСго сопротивлСния ΠΊ Π²Π½ΡƒΡ‚Ρ€Π΅Π½Π½Π΅ΠΌΡƒ ΡΠΎΠΏΡ€ΠΎΡ‚ΠΈΠ²Π»Π΅Π½ΠΈΡŽ ΠšΠŸΠ” источника Ρ‚ΠΎΠΊΠ° Ρ€Π°Π²Π΅Π½: 50 %; 80 %? ΠŸΠΎΡ‡Π΅ΠΌΡƒ случай, ΠΊΠΎΠ³Π΄Π° ΠšΠŸΠ” источника Ρ‚ΠΎΠΊΠ° Ρ€Π°Π²Π΅Π½ 100 %, Π½Π΅ прСдставляСт практичСского интСрСса?


    Π”ΠΎΠΏΠΎΠ»Π½ΠΈΡ‚Π΅Π»ΡŒΠ½Ρ‹Π΅ вопросы ΠΈ задания

    9. На рисункС 59.4 ΠΈΠ·ΠΎΠ±Ρ€Π°ΠΆΠ΅Π½Π° схСма измСрСния зависимости напряТСния U Π½Π° ΠΏΠΎΠ»ΡŽΡΠ°Ρ… источника Ρ‚ΠΎΠΊΠ° ΠΎΡ‚ силы Ρ‚ΠΎΠΊΠ° I. АмпСрмСтр ΠΈ Π²ΠΎΠ»ΡŒΡ‚ΠΌΠ΅Ρ‚Ρ€ считайтС ΠΈΠ΄Π΅Π°Π»ΡŒΠ½Ρ‹ΠΌΠΈ. Π‘ΠΎΠΏΡ€ΠΎΡ‚ΠΈΠ²Π»Π΅Π½ΠΈΠ΅ всСй ΠΎΠ±ΠΌΠΎΡ‚ΠΊΠΈ рСостата 16 Ом. ΠŸΡ€ΠΈ ΠΏΠ΅Ρ€Π²ΠΎΠΌ ΠΏΠΎΠ»ΠΎΠΆΠ΅Π½ΠΈΠΈ ΠΏΠΎΠ»Π·ΡƒΠ½ΠΊΠ° рСостата показания ΠΏΡ€ΠΈΠ±ΠΎΡ€ΠΎΠ² 3 А ΠΈ 8 Π’, Π° ΠΏΡ€ΠΈ Π²Ρ‚ΠΎΡ€ΠΎΠΌ ΠΏΠΎΠ»ΠΎΠΆΠ΅Π½ΠΈΠΈ – 2 А ΠΈ 12 Π’.

    Π°) Как сдвинули ΠΏΠΎΠ»Π·ΡƒΠ½ΠΎΠΊ рСостата ΠΌΠ΅ΠΆΠ΄Ρƒ ΠΏΠ΅Ρ€Π²Ρ‹ΠΌ ΠΈ Π²Ρ‚ΠΎΡ€Ρ‹ΠΌ измСрСниями – Π²Π»Π΅Π²ΠΎ ΠΈΠ»ΠΈ Π²ΠΏΡ€Π°Π²ΠΎ?
    Π±) Π§Π΅ΠΌΡƒ Ρ€Π°Π²Π½Ρ‹ Π­Π”Π‘ источника Ρ‚ΠΎΠΊΠ° ΠΈ Π΅Π³ΠΎ Π²Π½ΡƒΡ‚Ρ€Π΅Π½Π½Π΅Π΅ сопротивлСниС?
    Π²) ΠšΠ°ΠΊΠΎΠ²Ρ‹ Π±ΡƒΠ΄ΡƒΡ‚ показания ΠΏΡ€ΠΈΠ±ΠΎΡ€ΠΎΠ², Ссли ΠΏΠΎΠ»Π·ΡƒΠ½ΠΎΠΊ рСостата ΠΏΠ΅Ρ€Π΅Π΄Π²ΠΈΠ½ΡƒΡ‚ΡŒ Π² ΠΊΡ€Π°ΠΉΠ½Π΅Π΅ Π»Π΅Π²ΠΎΠ΅ ΠΏΠΎΠ»ΠΎΠΆΠ΅Π½ΠΈΠ΅? Π² ΠΊΡ€Π°ΠΉΠ½Π΅Π΅ ΠΏΡ€Π°Π²ΠΎΠ΅?

    10. ΠŸΡ€ΠΈ силС Ρ‚ΠΎΠΊΠ° 6 А ΠΌΠΎΡ‰Π½ΠΎΡΡ‚ΡŒ Ρ‚ΠΎΠΊΠ° Π²ΠΎ внСшнСй Ρ†Π΅ΠΏΠΈ Ρ€Π°Π²Π½Π° 90 Π’Ρ‚, Π° ΠΏΡ€ΠΈ силС Ρ‚ΠΎΠΊΠ° 2 А ΠΎΠ½Π° Ρ€Π°Π²Π½Π° 60 Π’Ρ‚.
    Π°) Π§Π΅ΠΌΡƒ Ρ€Π°Π²Π½Π° Π­Π”Π‘ источника Ρ‚ΠΎΠΊΠ°?
    Π±) Π§Π΅ΠΌΡƒ Ρ€Π°Π²Π½ΠΎ Π²Π½ΡƒΡ‚Ρ€Π΅Π½Π½Π΅Π΅ сопротивлСниС источника Ρ‚ΠΎΠΊΠ°?
    Π²) Π§Π΅ΠΌΡƒ Ρ€Π°Π²Π½ΠΎ напряТСниС Π½Π° ΠΏΠΎΠ»ΡŽΡΠ°Ρ… источника Π² ΠΏΠ΅Ρ€Π²ΠΎΠΌ ΠΈ Π²Ρ‚ΠΎΡ€ΠΎΠΌ случаях?
    Π³) Π§Π΅ΠΌΡƒ Ρ€Π°Π²Π΅Π½ ΠšΠŸΠ” источника Ρ‚ΠΎΠΊΠ° Π² ΠΏΠ΅Ρ€Π²ΠΎΠΌ ΠΈ Π²Ρ‚ΠΎΡ€ΠΎΠΌ случаях?

    Π—Π°ΠΊΠΎΠ½Ρ‹ Ома для участка Ρ†Π΅ΠΏΠΈ ΠΈ для ΠΏΠΎΠ»Π½ΠΎΠΉ Ρ†Π΅ΠΏΠΈ

    Автор Alexey На Ρ‡Ρ‚Π΅Π½ΠΈΠ΅ 4 ΠΌΠΈΠ½. ΠŸΡ€ΠΎΡΠΌΠΎΡ‚Ρ€ΠΎΠ² 3.6k. ΠžΠΏΡƒΠ±Π»ΠΈΠΊΠΎΠ²Π°Π½ΠΎ ОбновлСно

    Π’ 1826 Π³ΠΎΠ΄Ρƒ Π½Π΅ΠΌΠ΅Ρ†ΠΊΠΈΠΉ ΡƒΡ‡Π΅Π½Ρ‹ΠΉ Π“Π΅ΠΎΡ€Π³ Ом ΡΠΎΠ²Π΅Ρ€ΡˆΠΈΠ» ΠΎΡ‚ΠΊΡ€Ρ‹Ρ‚ΠΈΠ΅ ΠΈ описал
    эмпиричСский Π·Π°ΠΊΠΎΠ½ ΠΎ ΡΠΎΠΎΡ‚Π½ΠΎΡˆΠ΅Π½ΠΈΠΈ ΠΌΠ΅ΠΆΠ΄Ρƒ собой Ρ‚Π°ΠΊΠΈΡ… ΠΏΠΎΠΊΠ°Π·Π°Ρ‚Π΅Π»Π΅ΠΉ ΠΊΠ°ΠΊ сила Ρ‚ΠΎΠΊΠ°, напряТСниС ΠΈ особСнности ΠΏΡ€ΠΎΠ²ΠΎΠ΄Π½ΠΈΠΊΠ° Π² Ρ†Π΅ΠΏΠΈ. ВпослСдствии, ΠΏΠΎ ΠΈΠΌΠ΅Π½ΠΈ ΡƒΡ‡Π΅Π½ΠΎΠ³ΠΎ ΠΎΠ½ стал Π½Π°Π·Ρ‹Π²Π°Ρ‚ΡŒΡΡΒ Π·Π°ΠΊΠΎΠ½ Ома.

    Π’ дальнСйшСм Π²Ρ‹ΡΡΠ½ΠΈΠ»ΠΎΡΡŒ, Ρ‡Ρ‚ΠΎ эти особСнности Π½Π΅ Ρ‡Ρ‚ΠΎ ΠΈΠ½ΠΎΠ΅, ΠΊΠ°ΠΊ сопротивлСниС ΠΏΡ€ΠΎΠ²ΠΎΠ΄Π½ΠΈΠΊΠ°, Π²ΠΎΠ·Π½ΠΈΠΊΠ°ΡŽΡ‰Π΅Π΅ Π² процСссС Π΅Π³ΠΎ ΠΊΠΎΠ½Ρ‚Π°ΠΊΡ‚Π° с элСктричСством. Π­Ρ‚ΠΎ внСшнСС сопротивлСниС (R). Π•ΡΡ‚ΡŒ Ρ‚Π°ΠΊΠΆΠ΅ Π²Π½ΡƒΡ‚Ρ€Π΅Π½Π½Π΅Π΅ сопротивлСниС (r), Ρ…Π°Ρ€Π°ΠΊΡ‚Π΅Ρ€Π½ΠΎΠ΅ для источника Ρ‚ΠΎΠΊΠ°.

    Π—Π°ΠΊΠΎΠ½ Ома для участка Ρ†Π΅ΠΏΠΈ

    Богласно ΠΎΠ±ΠΎΠ±Ρ‰Π΅Π½Π½ΠΎΠΌΡƒ Π·Π°ΠΊΠΎΠ½Ρƒ Ома для Π½Π΅ΠΊΠΎΡ‚ΠΎΡ€ΠΎΠ³ΠΎ участка Ρ†Π΅ΠΏΠΈ, сила Ρ‚ΠΎΠΊΠ° Π½Π° участкС Ρ†Π΅ΠΏΠΈ прямо ΠΏΡ€ΠΎΠΏΠΎΡ€Ρ†ΠΈΠΎΠ½Π°Π»ΡŒΠ½Π° Π½Π°ΠΏΡ€ΡΠΆΠ΅Π½ΠΈΡŽ Π½Π° ΠΊΠΎΠ½Ρ†Π°Ρ… участка ΠΈ ΠΎΠ±Ρ€Π°Ρ‚Π½ΠΎ ΠΏΡ€ΠΎΠΏΠΎΡ€Ρ†ΠΈΠΎΠ½Π°Π»ΡŒΠ½Π° ΡΠΎΠΏΡ€ΠΎΡ‚ΠΈΠ²Π»Π΅Π½ΠΈΡŽ.

    IΒ =Β U/Β R

    Π“Π΄Π΅Β U – напряТСниС ΠΊΠΎΠ½Ρ†ΠΎΠ² участка,I– сила Ρ‚ΠΎΠΊΠ°,Β R– сопротивлСниС ΠΏΡ€ΠΎΠ²ΠΎΠ΄Π½ΠΈΠΊΠ°.

    БСря Π²ΠΎ Π²Π½ΠΈΠΌΠ°Π½ΠΈΠ΅ Π²Ρ‹ΡˆΠ΅ΠΏΡ€ΠΈΠ²Π΅Π΄Π΅Π½Π½ΡƒΡŽ Ρ„ΠΎΡ€ΠΌΡƒΠ»Ρƒ, Π΅ΡΡ‚ΡŒ Π²ΠΎΠ·ΠΌΠΎΠΆΠ½ΠΎΡΡ‚ΡŒ Π½Π°ΠΉΡ‚ΠΈ нСизвСстныС значСнияUΠΈR, сдСлав нСслоТныС матСматичСскиС ΠΎΠΏΠ΅Ρ€Π°Ρ†ΠΈΠΈ.

    UΒ =Β I*R

    RΒ =Β UΒ /Β I

    Π”Π°Π½Π½Ρ‹Π΅ Π²Ρ‹ΡˆΠ΅ Ρ„ΠΎΡ€ΠΌΡƒΠ»Ρ‹ справСдливы лишь ΠΊΠΎΠ³Π΄Π° ΡΠ΅Ρ‚ΡŒ испытываСт Π½Π° сСбС ΠΎΠ΄Π½ΠΎ сопротивлСниС.

    Π—Π°ΠΊΠΎΠ½ Ома для Π·Π°ΠΌΠΊΠ½ΡƒΡ‚ΠΎΠΉ Ρ†Π΅ΠΏΠΈ

    Π‘ΠΈΠ»Π° Ρ‚ΠΎΠΊΠ° ΠΏΠΎΠ»Π½ΠΎΠΉ Ρ†Π΅ΠΏΠΈ Ρ€Π°Π²Π½Π° Π­Π”Π‘, Π΄Π΅Π»Π΅Π½Π½ΠΎΠΉ Π½Π° сумму сопротивлСний ΠΎΠ΄Π½ΠΎΡ€ΠΎΠ΄Π½ΠΎΠ³ΠΎ ΠΈ Π½Π΅ΠΎΠ΄Π½ΠΎΡ€ΠΎΠ΄Π½ΠΎΠ³ΠΎ участков Ρ†Π΅ΠΏΠΈ.

    Замкнутая ΡΠ΅Ρ‚ΡŒ ΠΈΠΌΠ΅Π΅Ρ‚ ΠΎΠ΄Π½ΠΎΠ²Ρ€Π΅ΠΌΠ΅Π½Π½ΠΎ сопротивлСния Π²Π½ΡƒΡ‚Ρ€Π΅Π½Π½Π΅Π³ΠΎ ΠΈ внСшнСго Ρ…Π°Ρ€Π°ΠΊΡ‚Π΅Ρ€Π°. ΠŸΠΎΡΡ‚ΠΎΠΌΡƒ Ρ„ΠΎΡ€ΠΌΡƒΠ»Ρ‹ ΠΎΡ‚Π½ΠΎΡˆΠ΅Π½ΠΈΡ Π±ΡƒΠ΄ΡƒΡ‚ ΡƒΠΆΠ΅ Π΄Ρ€ΡƒΠ³ΠΈΠΌΠΈ.

    IΒ =Β E/Β RΠ²Π½+r

    Π“Π΄Π΅Β E – элСктродвиТущая сила (Π­Π”Π‘), R- внСшнСС сопротивлСниС источника, r-Π²Π½ΡƒΡ‚Ρ€Π΅Π½Π½Π΅Π΅ сопротивлСниС источника.

    Π—Π°ΠΊΠΎΠ½ Ома для Π½Π΅ΠΎΠ΄Π½ΠΎΡ€ΠΎΠ΄Π½ΠΎΠ³ΠΎ участка Ρ†Π΅ΠΏΠΈ

    Замкнутая элСктричСская ΡΠ΅Ρ‚ΡŒ содСрТит участки Π»ΠΈΠ½Π΅ΠΉΠ½ΠΎΠ³ΠΎ ΠΈ Π½Π΅Π»ΠΈΠ½Π΅ΠΉΠ½ΠΎΠ³ΠΎ Ρ…Π°Ρ€Π°ΠΊΡ‚Π΅Ρ€Π°. Участки, Π½Π΅ ΠΈΠΌΠ΅ΡŽΡ‰ΠΈΠ΅ источника Ρ‚ΠΎΠΊΠ° ΠΈ Π½Π΅ зависящиС ΠΎΡ‚ стороннСго воздСйствия ΡΠ²Π»ΡΡŽΡ‚ΡΡ Π»ΠΈΠ½Π΅ΠΉΠ½Ρ‹ΠΌΠΈ, Π° участки, содСрТащиС источник – Π½Π΅Π»ΠΈΠ½Π΅ΠΉΠ½Ρ‹ΠΌΠΈ.

    Π—Π°ΠΊΠΎΠ½ Ома для участка сСти ΠΎΠ΄Π½ΠΎΡ€ΠΎΠ΄Π½ΠΎΠ³ΠΎ Ρ…Π°Ρ€Π°ΠΊΡ‚Π΅Ρ€Π° Π±Ρ‹Π» ΠΈΠ·Π»ΠΎΠΆΠ΅Π½ Π²Ρ‹ΡˆΠ΅. Π—Π°ΠΊΠΎΠ½ Π½Π° Π½Π΅Π»ΠΈΠ½Π΅ΠΉΠ½ΠΎΠΌ участкС Π±ΡƒΠ΄Π΅Ρ‚ ΠΈΠΌΠ΅Ρ‚ΡŒ ΡΠ»Π΅Π΄ΡƒΡŽΡ‰ΠΈΠΉ Π²ΠΈΠ΄:

    IΒ =Β U/Β RΒ =Β f1 – f2 +Β E/Β R

    Π“Π΄Π΅Β f1 – f2 – Ρ€Π°Π·Π½ΠΈΡ†Π° ΠΏΠΎΡ‚Π΅Π½Ρ†ΠΈΠ°Π»ΠΎΠ² Π½Π° ΠΊΠΎΠ½Π΅Ρ‡Π½Ρ‹Ρ… Ρ‚ΠΎΡ‡ΠΊΠ°Ρ… рассматриваСмого участка сСти

    R – ΠΎΠ±Ρ‰Π΅Π΅ сопротивлСниС Π½Π΅Π»ΠΈΠ½Π΅ΠΉΠ½ΠΎΠ³ΠΎ участка Ρ†Π΅ΠΏΠΈ

    Π­Π”Π‘ Π½Π΅Π»ΠΈΠ½Π΅ΠΉΠ½ΠΎΠ³ΠΎ участка Ρ†Π΅ΠΏΠΈ Π±Ρ‹Π²Π°Π΅Ρ‚ большС нуля ΠΈΠ»ΠΈ мСньшС. Если Π½Π°ΠΏΡ€Π°Π²Π»Π΅Π½ΠΈΠ΅ двиТСния Ρ‚ΠΎΠΊΠ°, ΠΈΠ΄ΡƒΡ‰Π΅Π³ΠΎ ΠΈΠ· источника с Π΄Π²ΠΈΠΆΠ΅Π½ΠΈΠ΅ΠΌ Ρ‚ΠΎΠΊΠ° Π² элСктричСской сСти, ΡΠΎΠ²ΠΏΠ°Π΄Π°ΡŽΡ‚, Π±ΡƒΠ΄Π΅Ρ‚ ΠΏΡ€Π΅ΠΎΠ±Π»Π°Π΄Π°Ρ‚ΡŒ Π΄Π²ΠΈΠΆΠ΅Π½ΠΈΠ΅ зарядов ΠΏΠΎΠ»ΠΎΠΆΠΈΡ‚Π΅Π»ΡŒΠ½ΠΎΠ³ΠΎ Ρ…Π°Ρ€Π°ΠΊΡ‚Π΅Ρ€Π° ΠΈ Π­Π”Π‘ Π±ΡƒΠ΄Π΅Ρ‚ ΠΏΠΎΠ»ΠΎΠΆΠΈΡ‚Π΅Π»ΡŒΠ½Π°Ρ. Π’ случаС ΠΆΠ΅ совпадСния Π½Π°ΠΏΡ€Π°Π²Π»Π΅Π½ΠΈΠΉ, Π² сСти Π±ΡƒΠ΄Π΅Ρ‚ ΡƒΠ²Π΅Π»ΠΈΡ‡Π΅Π½ΠΎ Π΄Π²ΠΈΠΆΠ΅Π½ΠΈΠ΅ ΠΎΡ‚Ρ€ΠΈΡ†Π°Ρ‚Π΅Π»ΡŒΠ½Ρ‹Ρ… зарядов, создаваСмых Π­Π”Π‘.

    Π—Π°ΠΊΠΎΠ½ Ома для ΠΏΠ΅Ρ€Π΅ΠΌΠ΅Π½Π½ΠΎΠ³ΠΎ Ρ‚ΠΎΠΊΠ°

    ΠŸΡ€ΠΈ ΠΈΠΌΠ΅ΡŽΡ‰Π΅ΠΉΡΡ Π² сСти Смкости ΠΈΠ»ΠΈ инСртности, Π½Π΅ΠΎΠ±Ρ…ΠΎΠ΄ΠΈΠΌΠΎ ΡƒΡ‡ΠΈΡ‚Ρ‹Π²Π°Ρ‚ΡŒ ΠΏΡ€ΠΈ ΠΏΡ€ΠΎΠ²ΠΎΠ΄ΠΈΠΌΡ‹Ρ… вычислСниях, Ρ‡Ρ‚ΠΎ ΠΎΠ½ΠΈ Π²Ρ‹Π΄Π°ΡŽΡ‚ своС сопротивлСниС, ΠΎΡ‚ дСйствия ΠΊΠΎΡ‚ΠΎΡ€ΠΎΠ³ΠΎ Ρ‚ΠΎΠΊ ΠΏΡ€ΠΈΠΎΠ±Ρ€Π΅Ρ‚Π°Π΅Ρ‚ ΠΏΠ΅Ρ€Π΅ΠΌΠ΅Π½Π½Ρ‹ΠΉ Ρ…Π°Ρ€Π°ΠΊΡ‚Π΅Ρ€.

    Π—Π°ΠΊΠΎΠ½ Ома для ΠΏΠ΅Ρ€Π΅ΠΌΠ΅Π½Π½ΠΎΠ³ΠΎ Ρ‚ΠΎΠΊΠ° выглядит Ρ‚Π°ΠΊ:

    IΒ =Β U/Β Z

    Β  Π³Π΄Π΅ Z – сопротивлСниС ΠΏΠΎ всСй Π΄Π»ΠΈΠ½Π΅ элСктричСской сСти. Π•Π³ΠΎ Π΅Ρ‰Π΅ Π½Π°Π·Ρ‹Π²Π°ΡŽΡ‚Β ΠΈΠΌΠΏΠ΅Π΄Π°Π½Ρ. ИмпСданс ΡΠΎΡΡ‚Π°Π²Π»ΡΡŽΡ‚ сопротивлСния Π°ΠΊΡ‚ΠΈΠ²Π½ΠΎΠ³ΠΎ ΠΈ Ρ€Π΅Π°ΠΊΡ‚ΠΈΠ²Π½ΠΎΠ³ΠΎ Ρ…Π°Ρ€Π°ΠΊΡ‚Π΅Ρ€Π°.

    Π—Π°ΠΊΠΎΠ½ Ома Π½Π΅ являСтся основным Π½Π°ΡƒΡ‡Π½Ρ‹ΠΌ Π·Π°ΠΊΠΎΠ½ΠΎΠΌ, Π° лишь эмпиричСским ΠΎΡ‚Π½ΠΎΡˆΠ΅Π½ΠΈΠ΅ΠΌ, ΠΏΡ€ΠΈΡ‡Π΅ΠΌ Π² Π½Π΅ΠΊΠΎΡ‚ΠΎΡ€Ρ‹Ρ… условиях ΠΎΠ½ΠΎ ΠΌΠΎΠΆΠ΅Ρ‚ Π½Π΅ ΡΠΎΠ±Π»ΡŽΠ΄Π°Ρ‚ΡŒΡΡ:

    • Когда ΡΠ΅Ρ‚ΡŒ ΠΎΠ±Π»Π°Π΄Π°Π΅Ρ‚ высокой частотой, элСктромагнитноС ΠΏΠΎΠ»Π΅ мСняСтся с большой ΡΠΊΠΎΡ€ΠΎΡΡ‚ΡŒΡŽ, ΠΈ ΠΏΡ€ΠΈ расчСтах Π½Π΅ΠΎΠ±Ρ…ΠΎΠ΄ΠΈΠΌΠΎ ΡƒΡ‡ΠΈΡ‚Ρ‹Π²Π°Ρ‚ΡŒ ΠΈΠ½Π΅Ρ€Ρ‚Π½ΠΎΡΡ‚ΡŒ носитСлСй заряда;
    • Π’ условиях Π½ΠΈΠ·ΠΊΠΎΠΉ Ρ‚Π΅ΠΌΠΏΠ΅Ρ€Π°Ρ‚ΡƒΡ€Ρ‹ с вСщСствами, ΠΊΠΎΡ‚ΠΎΡ€Ρ‹Π΅ ΠΎΠ±Π»Π°Π΄Π°ΡŽΡ‚ ΡΠ²Π΅Ρ€Ρ…ΠΏΡ€ΠΎΠ²ΠΎΠ΄ΠΈΠΌΠΎΡΡ‚ΡŒΡŽ;
    • Когда ΠΏΡ€ΠΎΠ²ΠΎΠ΄Π½ΠΈΠΊ сильно нагрСваСтся проходящим напряТСниСм, ΠΎΡ‚Π½ΠΎΡˆΠ΅Π½ΠΈΠ΅ Ρ‚ΠΎΠΊΠ° ΠΊ Π½Π°ΠΏΡ€ΡΠΆΠ΅Π½ΠΈΡŽ становится ΠΏΠ΅Ρ€Π΅ΠΌΠ΅Π½Π½Ρ‹ΠΌ ΠΈ ΠΌΠΎΠΆΠ΅Ρ‚ Π½Π΅ ΡΠΎΠΎΡ‚Π²Π΅Ρ‚ΡΡ‚Π²ΠΎΠ²Π°Ρ‚ΡŒ ΠΎΠ±Ρ‰Π΅ΠΌΡƒ Π·Π°ΠΊΠΎΠ½Ρƒ;
    • ΠŸΡ€ΠΈ Π½Π°Ρ…ΠΎΠΆΠ΄Π΅Π½ΠΈΠΈ ΠΏΠΎΠ΄ высоким напряТСниСм ΠΏΡ€ΠΎΠ²ΠΎΠ΄Π½ΠΈΠΊΠ° ΠΈΠ»ΠΈ диэлСктрика;
    • Π’ свСтодиодных Π»Π°ΠΌΠΏΠ°Ρ…;
    • Π’ ΠΏΠΎΠ»ΡƒΠΏΡ€ΠΎΠ²ΠΎΠ΄Π½ΠΈΠΊΠ°Ρ… ΠΈ ΠΏΠΎΠ»ΡƒΠΏΡ€ΠΎΠ²ΠΎΠ΄Π½ΠΈΠΊΠΎΠ²Ρ‹Ρ… ΠΏΡ€ΠΈΠ±ΠΎΡ€Π°Ρ….

    Π’ свою ΠΎΡ‡Π΅Ρ€Π΅Π΄ΡŒ элСмСнты ΠΈ ΠΏΡ€ΠΎΠ²ΠΎΠ΄Π½ΠΈΠΊΠΈ, ΡΠΎΠ±Π»ΡŽΠ΄Π°ΡŽΡ‰ΠΈΠ΅ Π·Π°ΠΊΠΎΠ½ Ома, Π½Π°Π·Ρ‹Π²Π°ΡŽΡ‚ΡΡΒ ΠΎΠΌΠΈΡ‡Π΅ΡΠΊΠΈΠΌΠΈ.

    Π—Π°ΠΊΠΎΠ½ Ома ΠΌΠΎΠΆΠ΅Ρ‚ Π΄Π°Ρ‚ΡŒ объяснСниС Π½Π΅ΠΊΠΎΡ‚ΠΎΡ€Ρ‹ΠΌ явлСниям ΠΏΡ€ΠΈΡ€ΠΎΠ΄Ρ‹. НапримСр, ΠΊΠΎΠ³Π΄Π° ΠΌΡ‹ Π²ΠΈΠ΄ΠΈΠΌ ΠΏΡ‚ΠΈΡ†, сидящих Π½Π° Π²Ρ‹ΡΠΎΠΊΠΎΠ²ΠΎΠ»ΡŒΡ‚Π½Ρ‹Ρ… ΠΏΡ€ΠΎΠ²ΠΎΠ΄Π°Ρ…, Ρƒ нас Π²ΠΎΠ·Π½ΠΈΠΊΠ°Π΅Ρ‚ вопрос – ΠΏΠΎΡ‡Π΅ΠΌΡƒ Π½Π° Π½ΠΈΡ… Π½Π΅ дСйствуСт элСктричСский Ρ‚ΠΎΠΊ? ΠžΠ±ΡŠΡΡΠ½ΡΠ΅Ρ‚ΡΡ это довольно просто. ΠŸΡ‚ΠΈΡ†Ρ‹, сидя Π½Π° ΠΏΡ€ΠΎΠ²ΠΎΠ΄Π°Ρ…, ΠΏΡ€Π΅Π΄ΡΡ‚Π°Π²Π»ΡΡŽΡ‚ собой своСобразныС ΠΏΡ€ΠΎΠ²ΠΎΠ΄Π½ΠΈΠΊΠΈ. Π‘ΠΎΠ»ΡŒΡˆΠ°Ρ Ρ‡Π°ΡΡ‚ΡŒ напряТСния приходится Π½Π° ΠΏΡ€ΠΎΠΌΠ΅ΠΆΡƒΡ‚ΠΊΠΈ ΠΌΠ΅ΠΆΠ΄Ρƒ ΠΏΡ‚ΠΈΡ†Π°ΠΌΠΈ, Π° Ρ‚Π° доля, Ρ‡Ρ‚ΠΎ приходится Π½Π° сами Β«ΠΏΡ€ΠΎΠ²ΠΎΠ΄Π½ΠΈΠΊΠΈΒ» Π½Π΅ прСдставляСт для Π½ΠΈΡ… опасности.

    Но это ΠΏΡ€Π°Π²ΠΈΠ»ΠΎ Ρ€Π°Π±ΠΎΡ‚Π°Π΅Ρ‚ лишь ΠΏΡ€ΠΈ Π΅Π΄ΠΈΠ½ΠΈΡ‡Π½ΠΎΠΌ соприкосновСнии. Если ΠΏΡ‚ΠΈΡ†Π° Π·Π°Π΄Π΅Π½Π΅Ρ‚ клювом ΠΈΠ»ΠΈ ΠΊΡ€Ρ‹Π»ΠΎΠΌ ΠΏΡ€ΠΎΠ²ΠΎΠ΄ ΠΈΠ»ΠΈ Ρ‚Π΅Π»Π΅Π³Ρ€Π°Ρ„Π½Ρ‹ΠΉ столб, ΠΎΠ½Π° Π½Π΅ΠΌΠΈΠ½ΡƒΠ΅ΠΌΠΎ ΠΏΠΎΠ³ΠΈΠ±Π½Π΅Ρ‚ ΠΎΡ‚ ΠΎΠ³Ρ€ΠΎΠΌΠ½ΠΎΠ³ΠΎ количСства напряТСния, ΠΊΠΎΡ‚ΠΎΡ€ΠΎΠ΅ нСсут Π² сСбС эти участки. Π’Π°ΠΊΠΈΠ΅ случаи происходят повсСмСстно. ΠŸΠΎΡΡ‚ΠΎΠΌΡƒ Π² цСлях бСзопасности Π² Π½Π΅ΠΊΠΎΡ‚ΠΎΡ€Ρ‹Ρ… насСлСнных ΠΏΡƒΠ½ΠΊΡ‚Π°Ρ… установлСны ΡΠΏΠ΅Ρ†ΠΈΠ°Π»ΡŒΠ½Ρ‹Π΅ приспособлСния, Π·Π°Ρ‰ΠΈΡ‰Π°ΡŽΡ‰ΠΈΠ΅ ΠΏΡ‚ΠΈΡ† ΠΎΡ‚ опасного напряТСния. На Ρ‚Π°ΠΊΠΈΡ… насСстах ΠΏΡ‚ΠΈΡ†Ρ‹ находятся Π² ΠΏΠΎΠ»Π½ΠΎΠΉ бСзопасности.

    Π—Π°ΠΊΠΎΠ½ Ома Ρ‚Π°ΠΊΠΆΠ΅ ΡˆΠΈΡ€ΠΎΠΊΠΎ примСнятся Π½Π° ΠΏΡ€Π°ΠΊΡ‚ΠΈΠΊΠ΅. ЭлСктричСство ΡΠΌΠ΅Ρ€Ρ‚Π΅Π»ΡŒΠ½ΠΎ опасно для Ρ‡Π΅Π»ΠΎΠ²Π΅ΠΊΠ° ΠΏΡ€ΠΈ ΠΎΠ΄Π½ΠΎΠΌ лишь касании ΠΊ ΠΎΠ³ΠΎΠ»Π΅Π½Π½ΠΎΠΌΡƒ ΠΏΡ€ΠΎΠ²ΠΎΠ΄Ρƒ. Но Π² Π½Π΅ΠΊΠΎΡ‚ΠΎΡ€Ρ‹Ρ… случаях сопротивлСниС чСловСчСского Ρ‚Π΅Π»Π° ΠΌΠΎΠΆΠ΅Ρ‚ Π±Ρ‹Ρ‚ΡŒ Ρ€Π°Π·Π½Ρ‹ΠΌ.

    Π’Π°ΠΊ, Π½Π°ΠΏΡ€ΠΈΠΌΠ΅Ρ€, сухая ΠΈ нСповрСТдСнная ΠΊΠΎΠΆΠ° ΠΎΠ±Π»Π°Π΄Π°Π΅Ρ‚ большим сопротивлСниСм ΠΊ Π²ΠΎΠ·Π΄Π΅ΠΉΡΡ‚Π²ΠΈΡŽ элСктричСства Π½Π΅ΠΆΠ΅Π»ΠΈ Ρ€Π°Π½Π° ΠΈΠ»ΠΈ ΠΊΠΎΠΆΠ°, покрытая ΠΏΠΎΡ‚ΠΎΠΌ. Π’ слСдствиС пСрСутомлСния, Π½Π΅Ρ€Π²Π½ΠΎΠ³ΠΎ напряТСния ΠΈ опьянСния, Π΄Π°ΠΆΠ΅ ΠΏΡ€ΠΈ нСбольшом напряТСнии Ρ‚ΠΎΠΊΠ° Ρ‡Π΅Π»ΠΎΠ²Π΅ΠΊ ΠΌΠΎΠΆΠ΅Ρ‚ ΠΏΠΎΠ»ΡƒΡ‡ΠΈΡ‚ΡŒ ΡΠΈΠ»ΡŒΠ½Ρ‹ΠΉ ΡƒΠ΄Π°Ρ€ Ρ‚ΠΎΠΊΠΎΠΌ.

    Π’ срСднСм, сопротивлСниС Ρ‚Π΅Π»Π° Ρ‡Π΅Π»ΠΎΠ²Π΅ΠΊΠ° – 700 Ом, Π·Π½Π°Ρ‡ΠΈΡ‚, для Ρ‡Π΅Π»ΠΎΠ²Π΅ΠΊΠ° являСтся бСзопасным напряТСниС Π² 35 Π’. Работая с большим напряТСниСм, спСциалисты ΠΈΡΠΏΠΎΠ»ΡŒΠ·ΡƒΡŽΡ‚ ΡΠΏΠ΅Ρ†ΠΈΠ°Π»ΡŒΠ½Ρ‹Π΅ срСдства Π·Π°Ρ‰ΠΈΡ‚Ρ‹.

    Π“Π»Π°Π²Π° 21. ЭлСктричСский Ρ‚ΠΎΠΊ.

    Π—Π°ΠΊΠΎΠ½Ρ‹ Ома ΠΈ ДТоуля-Π›Π΅Π½Ρ†Π°

    Для Ρ€Π΅ΡˆΠ΅Π½ΠΈΡ Π·Π°Π΄Π°Ρ‡ Π•Π“Π­ Π½Π° постоянный Ρ‚ΠΎΠΊ Π½Π°Π΄ΠΎ Π·Π½Π°Ρ‚ΡŒ опрСдСлСния Ρ‚ΠΎΠΊΠ°, напряТСния, сопротивлСния, Π·Π°ΠΊΠΎΠ½ Ома для участка Ρ†Π΅ΠΏΠΈ ΠΈ Π·Π°ΠΌΠΊΠ½ΡƒΡ‚ΠΎΠΉ Ρ†Π΅ΠΏΠΈ, Π·Π°ΠΊΠΎΠ½ ДТоуля-Π›Π΅Π½Ρ†Π°, Π° Ρ‚Π°ΠΊΠΆΠ΅ ΡƒΠΌΠ΅Ρ‚ΡŒ Π½Π°Ρ…ΠΎΠ΄ΠΈΡ‚ΡŒ эквивалСнтныС сопротивлСния ΠΏΡ€ΠΎΡΡ‚Π΅ΠΉΡˆΠΈΡ… элСктричСски Ρ†Π΅ΠΏΠ΅ΠΉ. Рассмотрим эти вопросы.

    ЭлСктричСским Ρ‚ΠΎΠΊΠΎΠΌ Π½Π°Π·Ρ‹Π²Π°ΡŽΡ‚ упорядочСнноС Π΄Π²ΠΈΠΆΠ΅Π½ΠΈΠ΅ заряТСнных частиц. Π‘ΠΈΠ»ΠΎΠΉ Ρ‚ΠΎΠΊΠ° Π² Π½Π΅ΠΊΠΎΡ‚ΠΎΡ€ΠΎΠΌ сСчСнии ΠΏΡ€ΠΎΠ²ΠΎΠ΄Π½ΠΈΠΊΠ° называСтся ΠΎΡ‚Π½ΠΎΡˆΠ΅Π½ΠΈΠ΅ заряда , ΠΏΡ€ΠΎΡ‚Π΅ΠΊΡˆΠ΅Π³ΠΎ Ρ‡Π΅Ρ€Π΅Π· это сСчСниС Π·Π° ΠΈΠ½Ρ‚Π΅Ρ€Π²Π°Π» Π²Ρ€Π΅ΠΌΠ΅Π½ΠΈ , ΠΊ этому ΠΈΠ½Ρ‚Π΅Ρ€Π²Π°Π»Ρƒ Π²Ρ€Π΅ΠΌΠ΅Π½ΠΈ

    (21.1)

    Π§Ρ‚ΠΎΠ±Ρ‹ Π² ΠΏΡ€ΠΎΠ²ΠΎΠ΄Π½ΠΈΠΊΠ΅ Ρ‚Π΅ΠΊ элСктричСский Ρ‚ΠΎΠΊ, Π² ΠΏΡ€ΠΎΠ²ΠΎΠ΄Π½ΠΈΠΊΠ΅ Π΄ΠΎΠ»ΠΆΠ½ΠΎ Π±Ρ‹Ρ‚ΡŒ элСктричСскоС ΠΏΠΎΠ»Π΅, ΠΈΠ»ΠΈ, Π΄Ρ€ΡƒΠ³ΠΈΠΌΠΈ словами, ΠΏΠΎΡ‚Π΅Π½Ρ†ΠΈΠ°Π»Ρ‹ Ρ€Π°Π·Π»ΠΈΡ‡Π½Ρ‹Ρ… Ρ‚ΠΎΡ‡Π΅ΠΊ ΠΏΡ€ΠΎΠ²ΠΎΠ΄Π½ΠΈΠΊΠ° Π΄ΠΎΠ»ΠΆΠ½Ρ‹ Π±Ρ‹Ρ‚ΡŒ Ρ€Π°Π·Π½Ρ‹ΠΌΠΈ. Но ΠΏΡ€ΠΈ Π΄Π²ΠΈΠΆΠ΅Π½ΠΈΠΈ элСктричСских зарядов ΠΏΠΎ ΠΏΡ€ΠΎΠ²ΠΎΠ΄Π½ΠΈΠΊΡƒ ΠΏΠΎΡ‚Π΅Π½Ρ†ΠΈΠ°Π»Ρ‹ Ρ€Π°Π·Π»ΠΈΡ‡Π½Ρ‹Ρ… Ρ‚ΠΎΡ‡Π΅ΠΊ ΠΏΡ€ΠΎΠ²ΠΎΠ΄Π½ΠΈΠΊΠ° Π±ΡƒΠ΄ΡƒΡ‚ Π²Ρ‹Ρ€Π°Π²Π½ΠΈΠ²Π°Ρ‚ΡŒΡΡ (см. Π³Π». 19). ΠŸΠΎΡΡ‚ΠΎΠΌΡƒ для протСкания Ρ‚ΠΎΠΊΠ° Π² Ρ‚Π΅Ρ‡Π΅Π½ΠΈΠ΅ Π΄Π»ΠΈΡ‚Π΅Π»ΡŒΠ½ΠΎΠ³ΠΎ Π²Ρ€Π΅ΠΌΠ΅Π½ΠΈ Π½Π° ΠΊΠ°ΠΊΠΈΡ…-Ρ‚ΠΎ участках Ρ†Π΅ΠΏΠΈ Π½Π΅ΠΎΠ±Ρ…ΠΎΠ΄ΠΈΠΌΠΎ ΠΎΠ±Π΅ΡΠΏΠ΅Ρ‡ΠΈΡ‚ΡŒ Π΄Π²ΠΈΠΆΠ΅Π½ΠΈΠ΅ зарядов Π² Π½Π°ΠΏΡ€Π°Π²Π»Π΅Π½ΠΈΠΈ ΠΏΡ€ΠΎΡ‚ΠΈΠ²ΠΎΠΏΠΎΠ»ΠΎΠΆΠ½ΠΎΠΌ полю. Π’Π°ΠΊΠΎΠ΅ Π΄Π²ΠΈΠΆΠ΅Π½ΠΈΠ΅ ΠΌΠΎΠΆΠ΅Ρ‚ Π±Ρ‹Ρ‚ΡŒ обСспСчСно Ρ‚ΠΎΠ»ΡŒΠΊΠΎ силами нСэлСктричСской ΠΏΡ€ΠΈΡ€ΠΎΠ΄Ρ‹, ΠΊΠΎΡ‚ΠΎΡ€Ρ‹Π΅ Π² этом контСкстС принято Π½Π°Π·Ρ‹Π²Π°Ρ‚ΡŒ сторонними. Π’ Π³Π°Π»ΡŒΠ²Π°Π½ΠΈΡ‡Π΅ΡΠΊΠΈΡ… элСмСнтах (Β«Π±Π°Ρ‚Π°Ρ€Π΅ΠΉΠΊΠ°Ρ…Β») сторонниС силы Π²ΠΎΠ·Π½ΠΈΠΊΠ°ΡŽΡ‚ Π² Ρ€Π΅Π·ΡƒΠ»ΡŒΡ‚Π°Ρ‚Π΅ элСктрохимичСских ΠΏΡ€Π΅Π²Ρ€Π°Ρ‰Π΅Π½ΠΈΠΉ Π½Π° Π³Ρ€Π°Π½ΠΈΡ†Π°Ρ… элСктродов ΠΈ элСктролита. Π­Ρ‚ΠΈ прСвращСния ΠΎΠ±Π΅ΡΠΏΠ΅Ρ‡ΠΈΠ²Π°ΡŽΡ‚ ΠΏΠ΅Ρ€Π΅ΠΌΠ΅Ρ‰Π΅Π½ΠΈΠ΅ заряда ΠΏΡ€ΠΎΡ‚ΠΈΠ²ΠΎΠΏΠΎΠ»ΠΎΠΆΠ½ΠΎ Π½Π°ΠΏΡ€Π°Π²Π»Π΅Π½ΠΈΡŽ поля, поддСрТивая Π΄Π²ΠΈΠΆΠ΅Π½ΠΈΠ΅ зарядов ΠΏΠΎ Π·Π°ΠΌΠΊΠ½ΡƒΡ‚ΠΎΠΌΡƒ ΠΏΡƒΡ‚ΠΈ.

    Π‘ΠΈΠ»Π° Ρ‚ΠΎΠΊΠ° Π² ΠΎΠ΄Π½ΠΎΡ€ΠΎΠ΄Π½ΠΎΠΌ участкС ΠΏΡ€ΠΎΠ²ΠΎΠ΄Π½ΠΈΠΊΠ° ΠΏΡ€ΠΎΠΏΠΎΡ€Ρ†ΠΈΠΎΠ½Π°Π»ΡŒΠ½Π° напряТСнности элСктричСского поля Π²Π½ΡƒΡ‚Ρ€ΠΈ ΠΏΡ€ΠΎΠ²ΠΎΠ΄Π½ΠΈΠΊΠ°. А ΠΏΠΎΡΠΊΠΎΠ»ΡŒΠΊΡƒ Π½Π°ΠΏΡ€ΡΠΆΠ΅Π½Π½ΠΎΡΡ‚ΡŒ поля Π²Π½ΡƒΡ‚Ρ€ΠΈ ΠΏΡ€ΠΎΠ²ΠΎΠ΄Π½ΠΈΠΊΠ° связана с Ρ€Π°Π·Π½ΠΎΡΡ‚ΡŒΡŽ ΠΏΠΎΡ‚Π΅Π½Ρ†ΠΈΠ°Π»ΠΎΠ² Π΅Π³ΠΎ ΠΊΠΎΠ½Ρ†ΠΎΠ² (ΠΈΠ»ΠΈ элСктричСским напряТСниСм Π½Π° ΠΏΡ€ΠΎΠ²ΠΎΠ΄Π½ΠΈΠΊΠ΅ ), Ρ‚ΠΎ

    (21.2)

    ΠšΠΎΡΡ„Ρ„ΠΈΡ†ΠΈΠ΅Π½Ρ‚ ΠΏΡ€ΠΎΠΏΠΎΡ€Ρ†ΠΈΠΎΠ½Π°Π»ΡŒΠ½ΠΎΡΡ‚ΠΈ , ΠΊΠΎΡ‚ΠΎΡ€Ρ‹ΠΉ принято Π·Π°ΠΏΠΈΡΡ‹Π²Π°Ρ‚ΡŒ Π² Π·Π½Π°ΠΌΠ΅Π½Π°Ρ‚Π΅Π»ΡŒ Ρ„ΠΎΡ€ΠΌΡƒΠ»Ρ‹ (21.2), являСтся характСристикой ΠΏΡ€ΠΎΠ²ΠΎΠ΄Π½ΠΈΠΊΠ° ΠΈ называСтся Π΅Π³ΠΎ сопротивлСниСм. Π’ Ρ€Π΅Π·ΡƒΠ»ΡŒΡ‚Π°Ρ‚Π΅ Ρ„ΠΎΡ€ΠΌΡƒΠ»Π° (21.2) ΠΏΡ€ΠΈΠ½ΠΈΠΌΠ°Π΅Ρ‚ Π²ΠΈΠ΄

    (21. 3)

    Π€ΠΎΡ€ΠΌΡƒΠ»Π° (21.3) называСтся Π·Π°ΠΊΠΎΠ½ΠΎΠΌ Ома для ΠΎΠ΄Π½ΠΎΡ€ΠΎΠ΄Π½ΠΎΠ³ΠΎ участка Ρ†Π΅ΠΏΠΈ, Π° сам участок Ρ†Π΅ΠΏΠΈ часто Π½Π°Π·Ρ‹Π²Π°ΡŽΡ‚ рСзистором (ΠΎΡ‚ английского слова resistance β€” сопротивлСниС).

    Если ΠΏΡ€ΠΎΠ²ΠΎΠ΄Π½ΠΈΠΊ являСтся ΠΎΠ΄Π½ΠΎΡ€ΠΎΠ΄Π½Ρ‹ΠΌ ΠΈ ΠΈΠΌΠ΅Π΅Ρ‚ Ρ†ΠΈΠ»ΠΈΠ½Π΄Ρ€ΠΈΡ‡Π΅ΡΠΊΡƒΡŽ Ρ„ΠΎΡ€ΠΌΡƒ (ΠΏΡ€ΠΎΠ²ΠΎΠ΄), Ρ‚ΠΎ Π΅Π³ΠΎ сопротивлСниС ΠΏΡ€ΠΎΠΏΠΎΡ€Ρ†ΠΈΠΎΠ½Π°Π»ΡŒΠ½ΠΎ Π΄Π»ΠΈΠ½Π΅ ΠΈ ΠΎΠ±Ρ€Π°Ρ‚Π½ΠΎ ΠΏΡ€ΠΎΠΏΠΎΡ€Ρ†ΠΈΠΎΠ½Π°Π»ΡŒΠ½ΠΎ ΠΏΠ»ΠΎΡ‰Π°Π΄ΠΈ сСчСния

    (21.4)

    Π³Π΄Π΅ коэффициСнт ΠΏΡ€ΠΎΠΏΠΎΡ€Ρ†ΠΈΠΎΠ½Π°Π»ΡŒΠ½ΠΎΡΡ‚ΠΈ зависит Ρ‚ΠΎΠ»ΡŒΠΊΠΎ ΠΎΡ‚ ΠΌΠ°Ρ‚Π΅Ρ€ΠΈΠ°Π»Π° ΠΏΡ€ΠΎΠ²ΠΎΠ΄Π½ΠΈΠΊΠ° ΠΈ называСтся Π΅Π³ΠΎ ΡƒΠ΄Π΅Π»ΡŒΠ½Ρ‹ΠΌ сопротивлСниСм.

    Если участок Ρ†Π΅ΠΏΠΈ прСдставляСт собой нСсколько ΠΏΠΎΡΠ»Π΅Π΄ΠΎΠ²Π°Ρ‚Π΅Π»ΡŒΠ½ΠΎ соСдинСнных ΠΎΠ΄Π½ΠΎΡ€ΠΎΠ΄Π½Ρ‹Ρ… ΠΏΡ€ΠΎΠ²ΠΎΠ΄Π½ΠΈΠΊΠΎΠ² с сопротивлСниями (см. рисунок), Ρ‚ΠΎ сила Ρ‚ΠΎΠΊΠ° Ρ‡Π΅Ρ€Π΅Π· ΠΊΠ°ΠΆΠ΄Ρ‹ΠΉ ΠΏΡ€ΠΎΠ²ΠΎΠ΄Π½ΠΈΠΊ Π±ΡƒΠ΄Π΅Ρ‚ ΠΎΠ΄ΠΈΠ½Π°ΠΊΠΎΠ²ΠΎΠΉ , элСктричСскоС напряТСниС Π½Π° всСм участкС Ρ†Π΅ΠΏΠΈ Ρ€Π°Π²Π½ΠΎ суммС напряТСний Π½Π° ΠΊΠ°ΠΆΠ΄ΠΎΠΌ ΠΏΡ€ΠΎΠ²ΠΎΠ΄Π½ΠΈΠΊΠ΅ , Π° эквивалСнтноС сопротивлСниС всСго участка Ρ€Π°Π²Π½ΠΎ суммС сопротивлСний ΠΎΡ‚Π΄Π΅Π»ΡŒΠ½Ρ‹Ρ… ΠΏΡ€ΠΎΠ²ΠΎΠ΄Π½ΠΈΠΊΠΎΠ²

    (21. 4)

    Если участок Ρ†Π΅ΠΏΠΈ прСдставляСт собой нСсколько ΠΎΠ΄Π½ΠΎΡ€ΠΎΠ΄Π½Ρ‹Ρ… ΠΏΡ€ΠΎΠ²ΠΎΠ΄Π½ΠΈΠΊΠΎΠ² с сопротивлСниями , соСдинСнных ΠΏΠ°Ρ€Π°Π»Π»Π΅Π»ΡŒΠ½ΠΎ (см. рисунок), Ρ‚ΠΎ элСктричСскоС напряТСниС Π½Π° ΠΊΠ°ΠΆΠ΄ΠΎΠΌ ΠΏΡ€ΠΎΠ²ΠΎΠ΄Π½ΠΈΠΊΠ΅ Π±ΡƒΠ΄Π΅Ρ‚ ΠΎΠ΄ΠΈΠ½Π°ΠΊΠΎΠ²Ρ‹ΠΌ , Ρ‚ΠΎΠΊ Ρ‡Π΅Ρ€Π΅Π· участок Π±ΡƒΠ΄Π΅Ρ‚ Ρ€Π°Π²Π΅Π½ суммС Ρ‚ΠΎΠΊΠΎΠ², Ρ‚Π΅ΠΊΡƒΡ‰ΠΈΡ… Ρ‡Π΅Ρ€Π΅Π· ΠΊΠ°ΠΆΠ΄Ρ‹ΠΉ ΠΏΡ€ΠΎΠ²ΠΎΠ΄Π½ΠΈΠΊ , Π° Π²Π΅Π»ΠΈΡ‡ΠΈΠ½Π°, обратная эквивалСнтному ΡΠΎΠΏΡ€ΠΎΡ‚ΠΈΠ²Π»Π΅Π½ΠΈΡŽ всСго участка, Ρ€Π°Π²Π½ΠΎ суммС ΠΎΠ±Ρ€Π°Ρ‚Π½Ρ‹Ρ… сопротивлСний ΠΎΡ‚Π΄Π΅Π»ΡŒΠ½Ρ‹Ρ… ΠΏΡ€ΠΎΠ²ΠΎΠ΄Π½ΠΈΠΊΠΎΠ²

    (21.5)

    Рассмотрим Ρ‚Π΅ΠΏΠ΅Ρ€ΡŒ Π·Π°ΠΊΠΎΠ½ Ома для Π·Π°ΠΌΠΊΠ½ΡƒΡ‚ΠΎΠΉ элСктричСской Ρ†Π΅ΠΏΠΈ. ΠŸΡƒΡΡ‚ΡŒ имССтся замкнутая элСктричСская Ρ†Π΅ΠΏΡŒ, состоящая ΠΈΠ· источника сторонних сил с Π²Π½ΡƒΡ‚Ρ€Π΅Π½Π½ΠΈΠΌ сопротивлСниСм ΠΈ внСшнСго сопротивлСния . ΠŸΡƒΡΡ‚ΡŒ ΠΏΡ€ΠΈ ΠΏΡ€ΠΎΡ…ΠΎΠΆΠ΄Π΅Π½ΠΈΠΈ заряда Ρ‡Π΅Ρ€Π΅Π· источник сторонниС силы ΡΠΎΠ²Π΅Ρ€ΡˆΠ°ΡŽΡ‚ Ρ€Π°Π±ΠΎΡ‚Ρƒ . Π­Π»Π΅ΠΊΡ‚Ρ€ΠΎΠ΄Π²ΠΈΠΆΡƒΡ‰Π΅ΠΉ силой источника (часто ΠΈΡΠΏΠΎΠ»ΡŒΠ·ΡƒΠ΅Ρ‚ΡΡ Π°Π±Π±Ρ€Π΅Π²ΠΈΠ°Ρ‚ΡƒΡ€Π° Π­Π”Π‘) называСтся ΠΎΡ‚Π½ΠΎΡˆΠ΅Π½ΠΈΠ΅ Ρ€Π°Π±ΠΎΡ‚Ρ‹ сторонних сил ΠΊ заряду

    (21. 6)

    Π’ этом случаС сила Ρ‚ΠΎΠΊΠ° Π² Ρ†Π΅ΠΏΠΈ Ρ€Π°Π²Π½Π°

    (21.7)

    Π€ΠΎΡ€ΠΌΡƒΠ»Π° (21.7) называСтся Π·Π°ΠΊΠΎΠ½ΠΎΠΌ Ома для Π·Π°ΠΌΠΊΠ½ΡƒΡ‚ΠΎΠΉ элСктричСской Ρ†Π΅ΠΏΠΈ.

    ΠŸΡ€ΠΈ ΠΏΡ€ΠΎΡ…ΠΎΠΆΠ΄Π΅Π½ΠΈΠΈ элСктричСского Ρ‚ΠΎΠΊΠ° Ρ‡Π΅Ρ€Π΅Π· участок Ρ†Π΅ΠΏΠΈ элСктричСскоС ΠΏΠΎΠ»Π΅ ΡΠΎΠ²Π΅Ρ€ΡˆΠ°Π΅Ρ‚ Ρ€Π°Π±ΠΎΡ‚Ρƒ (часто эту Ρ€Π°Π±ΠΎΡ‚Ρƒ Π½Π°Π·Ρ‹Π²Π°ΡŽΡ‚ Ρ€Π°Π±ΠΎΡ‚ΠΎΠΉ Ρ‚ΠΎΠΊΠ°, хотя Ρ‚Π΅Ρ€ΠΌΠΈΠ½ этот Π½Π΅ ΠΎΡ‡Π΅Π½ΡŒ Ρ‚ΠΎΡ‡Π½Ρ‹ΠΉ). ΠžΡ‡Π΅Π²ΠΈΠ΄Π½ΠΎ, вся эта Ρ€Π°Π±ΠΎΡ‚Π° прСвращаСтся Π² Ρ‚Π΅ΠΏΠ»ΠΎ. ΠŸΠΎΡΡ‚ΠΎΠΌΡƒ Ссли Ρ‡Π΅Ρ€Π΅Π· участок Ρ†Π΅ΠΏΠΈ ΠΏΡ€ΠΎΡˆΠ΅Π» заряд , Π³Π΄Π΅ β€” сила Ρ‚ΠΎΠΊΠ° Π² Ρ†Π΅ΠΏΠΈ, β€” врСмя, Ρ‚ΠΎ количСство Π²Ρ‹Π΄Π΅Π»ΠΈΠ²ΡˆΠ΅ΠΉΡΡ Ρ‚Π΅ΠΏΠ»ΠΎΡ‚Ρ‹ Ρ€Π°Π²Π½ΠΎ

    (21.8)

    (для получСния послСднСго ΠΈ прСдпослСднСго равСнств использован Π·Π°ΠΊΠΎΠ½ Ома для участка Ρ†Π΅ΠΏΠΈ). Π€ΠΎΡ€ΠΌΡƒΠ»Ρ‹ (21.8) Π½Π°Π·Ρ‹Π²Π°ΡŽΡ‚ΡΡ Π·Π°ΠΊΠΎΠ½ΠΎΠΌ ДТоуля-Π›Π΅Π½Ρ†Π°. Из Ρ„ΠΎΡ€ΠΌΡƒΠ»Ρ‹ (21.8) слСдуСт, Ρ‡Ρ‚ΠΎ количСство Π²Ρ‹Π΄Π΅Π»ΠΈΠ²ΡˆΠ΅ΠΉΡΡ ΠΏΡ€ΠΈ ΠΏΡ€ΠΎΡ‚Π΅ΠΊΠ°Π½ΠΈΠΈ элСктричСского Ρ‚ΠΎΠΊΠ° Ρ‚Π΅ΠΏΠ»ΠΎΡ‚Ρ‹ Π»ΠΈΠ½Π΅ΠΉΠ½ΠΎ зависит ΠΎΡ‚ Π²Ρ€Π΅ΠΌΠ΅Π½ΠΈ наблюдСния. ΠŸΠΎΡΡ‚ΠΎΠΌΡƒ ΠΎΡ‚Π½ΠΎΡˆΠ΅Π½ΠΈΠ΅

    (21.9)

    ΠΊΠΎΡ‚ΠΎΡ€ΠΎΠ΅ называСтся ΠΌΠΎΡ‰Π½ΠΎΡΡ‚ΡŒΡŽ Ρ‚ΠΎΠΊΠ°, Π½Π΅ зависит ΠΎΡ‚ Π²Ρ€Π΅ΠΌΠ΅Π½ΠΈ наблюдСния. Π€ΠΎΡ€ΠΌΡƒΠ»Ρƒ (21.9) Ρ‚Π°ΠΊΠΆΠ΅ Π½Π°Π·Ρ‹Π²Π°ΡŽΡ‚ Π·Π°ΠΊΠΎΠ½ΠΎΠΌ ДТоуля-Π›Π΅Π½Ρ†Π°.

    Рассмотрим Ρ‚Π΅ΠΏΠ΅Ρ€ΡŒ Π·Π°Π΄Π°Ρ‡ΠΈ.

    Π‘Ρ‚Ρ€ΡƒΠΊΡ‚ΡƒΡ€Π° ΠΌΠ΅Ρ‚Π°Π»Π»Π° ΠΊΡ€Π°Ρ‚ΠΊΠΎ ΠΎΠ±ΡΡƒΠΆΠ΄Π°Π»Π°ΡΡŒ Π² Π³Π». 16: ΠΏΠΎΠ»ΠΎΠΆΠΈΡ‚Π΅Π»ΡŒΠ½ΠΎ заряТСнныС ΠΈΠΎΠ½Ρ‹ располоТСны Π² ΡƒΠ·Π»Π°Ρ… кристалличСской Ρ€Π΅ΡˆΠ΅Ρ‚ΠΊΠΈ, ΠΎΠ±Ρ€Π°Π·ΠΎΠ²Π°Π²ΡˆΠΈΠ΅ΡΡ Π² Ρ€Π΅Π·ΡƒΠ»ΡŒΡ‚Π°Ρ‚Π΅ диссоциации Π²Π°Π»Π΅Π½Ρ‚Π½Ρ‹Π΅ элСктроны ΠΌΠΎΠ³ΡƒΡ‚ свободно ΠΏΠ΅Ρ€Π΅ΠΌΠ΅Ρ‰Π°Ρ‚ΡŒΡΡ ΠΏΠΎ ΠΏΡ€ΠΎΠ²ΠΎΠ΄Π½ΠΈΠΊΡƒ (свободныС элСктроны). Они ΠΈ ΠΎΡΡƒΡ‰Π΅ΡΡ‚Π²Π»ΡΡŽΡ‚ ΠΏΡ€ΠΎΠ²ΠΎΠ΄ΠΈΠΌΠΎΡΡ‚ΡŒ ΠΌΠ΅Ρ‚Π°Π»Π»Π° (Π·Π°Π΄Π°Ρ‡Π° 21.1.1 β€” ΠΎΡ‚Π²Π΅Ρ‚ 2).

    Богласно ΠΎΠΏΡ€Π΅Π΄Π΅Π»Π΅Π½ΠΈΡŽ (21.1) Π½Π°Ρ…ΠΎΠ΄ΠΈΠΌ ΡΡ€Π΅Π΄Π½ΡŽΡŽ силу Ρ‚ΠΎΠΊΠ° Π² ΠΊΠ°Π½Π°Π»Π΅ ΠΌΠΎΠ»Π½ΠΈΠΈ (Π·Π°Π΄Π°Ρ‡Π° 21.1.2)

    (ΠΎΡ‚Π²Π΅Ρ‚ 2).

    Если Π·Π° 1 ΠΌΠΈΠ½ Ρ‡Π΅Ρ€Π΅Π· сСчСниС ΠΏΡ€ΠΎΠ²ΠΎΠ΄Π½ΠΈΠΊΠ° ΠΏΡ€ΠΎΡ‚Π΅ΠΊΠ°Π΅Ρ‚ заряд 60 Кл (Π·Π°Π΄Π°Ρ‡Π° 21.1.3), Ρ‚ΠΎ сила Ρ‚ΠΎΠΊΠ° Π² этом ΠΏΡ€ΠΎΠ²ΠΎΠ΄Π½ΠΈΠΊΠ΅ Ρ€Π°Π²Π½Π° А. ΠŸΡ€ΠΈΠΌΠ΅Π½ΡΡ Π΄Π°Π»Π΅Π΅ ΠΊ этому ΠΏΡ€ΠΎΠ²ΠΎΠ΄Π½ΠΈΠΊΡƒ Π·Π°ΠΊΠΎΠ½ Ома для участка Ρ†Π΅ΠΏΠΈ, ΠΏΠΎΠ»ΡƒΡ‡Π°Π΅ΠΌ Π’ (ΠΎΡ‚Π²Π΅Ρ‚ 2).

    По Π·Π°ΠΊΠΎΠ½Ρƒ Ома для участка Ρ†Π΅ΠΏΠΈ ΠΈΠΌΠ΅Π΅ΠΌ для силы Ρ‚ΠΎΠΊΠ° Ρ‡Π΅Ρ€Π΅Π· участок Ρ†Π΅ΠΏΠΈ послС измСнСния Π΅Π³ΠΎ сопротивлСния ΠΈ элСктричСского напряТСния Π½Π° Π½Π΅ΠΌ (Π·Π°Π΄Π°Ρ‡Π° 21.1.4)

    Π’Π°ΠΊΠΈΠΌ ΠΎΠ±Ρ€Π°Π·ΠΎΠΌ, сила Ρ‚ΠΎΠΊΠ° ΡƒΠΌΠ΅Π½ΡŒΡˆΠΈΠ»Π°ΡΡŒ Π² 4 Ρ€Π°Π·Π° (ΠΎΡ‚Π²Π΅Ρ‚ 3).

    Богласно Π·Π°ΠΊΠΎΠ½Ρƒ Ома для участка Ρ†Π΅ΠΏΠΈ сопротивлСниС β€” это коэффициСнт ΠΏΡ€ΠΎΠΏΠΎΡ€Ρ†ΠΈΠΎΠ½Π°Π»ΡŒΠ½ΠΎΡΡ‚ΠΈ ΠΌΠ΅ΠΆΠ΄Ρƒ напряТСниСм Π½Π° этом участкС ΠΈ силой Ρ‚ΠΎΠΊΠ° Π² Π½Π΅ΠΌ. ΠŸΠΎΡΡ‚ΠΎΠΌΡƒ Π² Π·Π°Π΄Π°Ρ‡Π΅ 21.1.5 ΠΈΠΌΠ΅Π΅ΠΌ, Π½Π°ΠΏΡ€ΠΈΠΌΠ΅Ρ€, ΠΈΡΠΏΠΎΠ»ΡŒΠ·ΡƒΡ ΠΊΡ€Π°ΠΉΠ½ΡŽΡŽ Ρ‚ΠΎΡ‡ΠΊΡƒ Π³Ρ€Π°Ρ„ΠΈΠΊΠ°

    (ΠΎΡ‚Π²Π΅Ρ‚ 2). Из-Π·Π° Π»ΠΈΠ½Π΅ΠΉΠ½ΠΎΠΉ зависимости Ρ‚ΠΎΠΊΠ° ΠΎΡ‚ напряТСния вычислСния ΠΌΠΎΠΆΠ½ΠΎ Π±Ρ‹Π»ΠΎ Π²Ρ‹ΠΏΠΎΠ»Π½ΠΈΡ‚ΡŒ ΠΈ ΠΏΠΎ Π΄Ρ€ΡƒΠ³ΠΈΠΌ Ρ‚ΠΎΡ‡ΠΊΠ°ΠΌ Π³Ρ€Π°Ρ„ΠΈΠΊΠ°, ΠΎΡ‚Π²Π΅Ρ‚ Π±Ρ‹Π» Π±Ρ‹ Ρ‚Π°ΠΊΠΈΠΌ ΠΆΠ΅.

    Богласно Ρ„ΠΎΡ€ΠΌΡƒΠ»Π΅ (21.4) ΠΈΠΌΠ΅Π΅ΠΌ для ΠΏΠ΅Ρ€Π²ΠΎΠΉ ΠΏΡ€ΠΎΠ²ΠΎΠ»ΠΎΠΊΠΈ Π² Π·Π°Π΄Π°Ρ‡Π΅ 21.1.6

    Π³Π΄Π΅ β€” ΡƒΠ΄Π΅Π»ΡŒΠ½ΠΎΠ΅ сопротивлСниС ΠΌΠ΅Π΄ΠΈ, β€” Π΄Π»ΠΈΠ½Π° ΠΏΡ€ΠΎΠ²ΠΎΠ΄Π½ΠΈΠΊΠ°, β€” Π΅Π³ΠΎ радиус. Для ΠΌΠ΅Π΄Π½ΠΎΠΉ ΠΏΡ€ΠΎΠ²ΠΎΠ»ΠΎΠΊΠΈ с Π²Π΄Π²ΠΎΠ΅ большСй Π΄Π»ΠΈΠ½ΠΎΠΉ ΠΈ Π²Ρ‚Ρ€ΠΎΠ΅ бóльшим радиусом сСчСния ΠΈΠΌΠ΅Π΅ΠΌ

    (ΠΎΡ‚Π²Π΅Ρ‚ 3).

    Как слСдуСт ΠΈΠ· Ρ„ΠΎΡ€ΠΌΡƒΠ»Ρ‹ (21.4) ΠΏΡ€ΠΈ Π΄Π²ΡƒΠΊΡ€Π°Ρ‚Π½ΠΎΠΌ ΡƒΠΌΠ΅Π½ΡŒΡˆΠ΅Π½ΠΈΠΈ Π΄Π»ΠΈΠ½Ρ‹ ΠΏΡ€ΠΎΠ²ΠΎΠ΄Π½ΠΈΠΊΠ° Π²Π΄Π²ΠΎΠ΅ ΡƒΠΌΠ΅Π½ΡŒΡˆΠ°Π΅Ρ‚ΡΡ Π΅Π³ΠΎ сопротивлСниС. ΠŸΠΎΡΡ‚ΠΎΠΌΡƒ ΠΈΠ· Π·Π°ΠΊΠΎΠ½Π° Ома для участка Ρ†Π΅ΠΏΠΈ (21.3) Π·Π°ΠΊΠ»ΡŽΡ‡Π°Π΅ΠΌ, Ρ‡Ρ‚ΠΎ ΠΏΡ€ΠΈ Π΄Π²ΡƒΠΊΡ€Π°Ρ‚Π½ΠΎΠΌ ΡƒΠΌΠ΅Π½ΡŒΡˆΠ΅Π½ΠΈΠΈ напряТСния Π½Π° ΠΏΡ€ΠΎΠ²ΠΎΠ΄Π½ΠΈΠΊΠ΅ ΠΈ Π΄Π²ΡƒΠΊΡ€Π°Ρ‚Π½ΠΎΠΌ ΡƒΠΌΠ΅Π½ΡŒΡˆΠ΅Π½ΠΈΠΈ Π΅Π³ΠΎ Π΄Π»ΠΈΠ½Ρ‹ (Π·Π°Π΄Π°Ρ‡Π° 21.1.7) сила Ρ‚ΠΎΠΊΠ° Π² ΠΏΡ€ΠΎΠ²ΠΎΠ΄Π½ΠΈΠΊΠ΅ Π½Π΅ измСнится (ΠΎΡ‚Π²Π΅Ρ‚ 4).

    Π’ Π·Π°Π΄Π°Ρ‡Π΅ 21.1.8 слСдуСт ΠΈΡΠΏΠΎΠ»ΡŒΠ·ΠΎΠ²Π°Ρ‚ΡŒ Π·Π°ΠΊΠΎΠ½ Ома для Π·Π°ΠΌΠΊΠ½ΡƒΡ‚ΠΎΠΉ элСктричСской Ρ†Π΅ΠΏΠΈ (21.7). ИмССм

    Π³Π΄Π΅ β€” Π­Π”Π‘ источника, β€” сопротивлСнии Π΅ внСшнСй Ρ†Π΅ΠΏΠΈ, β€” сопротивлСниС источника (ΠΎΡ‚Π²Π΅Ρ‚ 1).

    Π’ Π·Π°Π΄Π°Ρ‡Π΅ 21.1.9 слСдуСт ΠΏΡ€ΠΈΠΌΠ΅Π½ΠΈΡ‚ΡŒ Π·Π°ΠΊΠΎΠ½ Ома для Π·Π°ΠΌΠΊΠ½ΡƒΡ‚ΠΎΠΉ элСктричСской Ρ†Π΅ΠΏΠΈ (21.7) ΠΊ ΠΊΠ°ΠΊΠΎΠΌΡƒ-Π½ΠΈΠ±ΡƒΠ΄ΡŒ Π·Π½Π°Ρ‡Π΅Π½ΠΈΡŽ внСшнСго сопротивлСния, ΠΏΠΎ Π³Ρ€Π°Ρ„ΠΈΠΊΡƒ Π½Π°ΠΉΡ‚ΠΈ силу Ρ‚ΠΎΠΊΠ° Π² Ρ†Π΅ΠΏΠΈ, Π° Π·Π°Ρ‚Π΅ΠΌ ΠΈ Π­Π”Π‘ источника. ΠŸΡ€ΠΎΡ‰Π΅ всСго ΠΏΡ€ΠΈΠΌΠ΅Π½ΠΈΡ‚ΡŒ Π·Π°ΠΊΠΎΠ½ Ома ΠΊ ΡΠ»ΡƒΡ‡Π°ΡŽ . Из Π³Ρ€Π°Ρ„ΠΈΠΊΠ° Π½Π°Ρ…ΠΎΠ΄ΠΈΠΌ силу Ρ‚ΠΎΠΊΠ° . ΠŸΠΎΡΡ‚ΠΎΠΌΡƒ

    Π³Π΄Π΅ β€” Π²Π½ΡƒΡ‚Ρ€Π΅Π½Π½Π΅Π΅ сопротивлСниС источника (ΠΎΡ‚Π²Π΅Ρ‚ 3).

    Из Ρ„ΠΎΡ€ΠΌΡƒΠ»Ρ‹ (21.9) слСдуСт, Ρ‡Ρ‚ΠΎ ΠΏΡ€ΠΈ фиксированном сопротивлСнии участка Ρ†Π΅ΠΏΠΈ ΡƒΠ²Π΅Π»ΠΈΡ‡Π΅Π½ΠΈΠ΅ элСктричСского напряТСния Π² 2 Ρ€Π°Π·Π° (Π·Π°Π΄Π°Ρ‡Π° 21.1.10) ΠΏΡ€ΠΈΠ²Π΅Π΄Π΅Ρ‚ ΠΊ ΡƒΠ²Π΅Π»ΠΈΡ‡Π΅Π½ΠΈΡŽ мощности Ρ‚ΠΎΠΊΠ° Π² 4 Ρ€Π°Π·Π° (ΠΎΡ‚Π²Π΅Ρ‚ 2).

    Π’ Π·Π°Π΄Π°Ρ‡Π΅ 21.2.1 ΡƒΠ΄ΠΎΠ±Π½ΠΎ ΠΈΡΠΏΠΎΠ»ΡŒΠ·ΠΎΠ²Π°Ρ‚ΡŒ Π²Ρ‚ΠΎΡ€ΡƒΡŽ ΠΈΠ· Ρ„ΠΎΡ€ΠΌΡƒΠ» (21.9) . ИмССм Π’Ρ‚ (ΠΎΡ‚Π²Π΅Ρ‚ 3).

    Часто школьники Π½Π΅ ΠΌΠΎΠ³ΡƒΡ‚ ΠΎΡ‚Π²Π΅Ρ‚ΠΈΡ‚ΡŒ Π½Π° Ρ‚Π°ΠΊΠΎΠΉ вопрос: ΠΈΠ· Ρ„ΠΎΡ€ΠΌΡƒΠ»Ρ‹ для мощности Ρ‚ΠΎΠΊΠ° слСдуСт, Ρ‡Ρ‚ΠΎ ΠΌΠΎΡ‰Π½ΠΎΡΡ‚ΡŒ Π»ΠΈΠ½Π΅ΠΉΠ½ΠΎ растСт с ростом сопротивлСния, Π° ΠΈΠ· Ρ„ΠΎΡ€ΠΌΡƒΠ»Ρ‹ β€” ΡƒΠ±Ρ‹Π²Π°Π΅Ρ‚ с ростом сопротивлСния. А ΠΊΠ°ΠΊ ΠΆΠ΅ Π² Π΄Π΅ΠΉΡΡ‚Π²ΠΈΡ‚Π΅Π»ΡŒΠ½ΠΎΡΡ‚ΠΈ ΠΌΠΎΡ‰Π½ΠΎΡΡ‚ΡŒ зависит ΠΎΡ‚ сопротивлСния? Π”Π°Π²Π°ΠΉΡ‚Π΅ разбСрСмся Π² этом вопросС Π½Π° ΠΏΡ€ΠΈΠΌΠ΅Ρ€Π΅ Π·Π°Π΄Π°Ρ‡ΠΈ 21.2.2. ΠšΠΎΠ½Π΅Ρ‡Π½ΠΎ, ΠΎΠ±Π° ΠΏΡ€Π΅Π΄Π»ΠΎΠΆΠ΅Π½Π½Ρ‹Ρ… Β«Ρ€Π΅ΡˆΠ΅Π½ΠΈΡΒ» Π½Π΅ΠΏΡ€Π°Π²ΠΈΠ»ΡŒΠ½Ρ‹: Π² Π½ΠΈΡ… ΠΌΠΎΠ»Ρ‡Π°Π»ΠΈΠ²ΠΎ ΠΏΡ€Π΅Π΄ΠΏΠΎΠ»Π°Π³Π°Π»ΠΎΡΡŒ, Ρ‡Ρ‚ΠΎ сила Ρ‚ΠΎΠΊΠ°, Ρ‚Π΅ΠΊΡƒΡ‰Π΅Π³ΠΎ Ρ‡Π΅Ρ€Π΅Π· это сопротивлСниС, ΠΈΠ»ΠΈ напряТСниС Π½Π° этом сопротивлСнии Π½Π΅ зависят ΠΎΡ‚ Π΅Π³ΠΎ Π²Π΅Π»ΠΈΡ‡ΠΈΠ½Ρ‹. А Π½Π° самом Π΄Π΅Π»Π΅ эти Π²Π΅Π»ΠΈΡ‡ΠΈΠ½Ρ‹ ΠΎΡ‚ сопротивлСния зависят, ΠΏΡ€ΠΈΡ‡Π΅ΠΌ эти зависимости ΠΌΠΎΠ³ΡƒΡ‚ Π±Ρ‹Ρ‚ΡŒ Ρ€Π°Π·Π½Ρ‹ΠΌΠΈ для Ρ€Π°Π·Π½Ρ‹Ρ… источников Ρ‚ΠΎΠΊΠ°. Π’Π½ΡƒΡ‚Ρ€Π΅Π½Π½Π΅Π΅ сопротивлСниС Π±Ρ‹Ρ‚ΠΎΠ²Ρ‹Ρ… элСктричСских сСтСй ΠΎΡ‡Π΅Π½ΡŒ ΠΌΠ°Π»ΠΎ. Π’ этом случаС ΠΈΠ· Π·Π°ΠΊΠΎΠ½ΠΎΠ² Ома для Π·Π°ΠΌΠΊΠ½ΡƒΡ‚ΠΎΠΉ Ρ†Π΅ΠΏΠΈ ΠΈ участка Ρ†Π΅ΠΏΠΈ (21.7), (21.3) слСдуСт, Ρ‡Ρ‚ΠΎ напряТСниС Π½Π° любом элСмСнтС, Π²ΠΊΠ»ΡŽΡ‡Π΅Π½Π½ΠΎΠΌ Π² Ρ‚Π°ΠΊΡƒΡŽ ΡΠ΅Ρ‚ΡŒ, Π½Π΅ зависит ΠΎΡ‚ сопротивлСния этого элСмСнта ΠΈ Ρ€Π°Π²Π½ΠΎ Π½ΠΎΠΌΠΈΠ½Π°Π»ΡŒΠ½ΠΎΠΌΡƒ Π½Π°ΠΏΡ€ΡΠΆΠ΅Π½ΠΈΡŽ сСти . ΠŸΠΎΡΡ‚ΠΎΠΌΡƒ ΠΈΠ· Ρ„ΠΎΡ€ΠΌΡƒΠ»Ρ‹ Π·Π°ΠΊΠ»ΡŽΡ‡Π°Π΅ΠΌ, Ρ‡Ρ‚ΠΎ ΠΌΠΎΡ‰Π½ΠΎΡΡ‚ΡŒ, которая выдСляСтся Π½Π° Ρ‚Π°ΠΊΠΎΠΌ элСмСнтС ΠΎΠ±Ρ€Π°Ρ‚Π½ΠΎ ΠΏΡ€ΠΎΠΏΠΎΡ€Ρ†ΠΈΠΎΠ½Π°Π»ΡŒΠ½Π° Π΅Π³ΠΎ ΡΠΎΠΏΡ€ΠΎΡ‚ΠΈΠ²Π»Π΅Π½ΠΈΡŽ (ΠΎΡ‚Π²Π΅Ρ‚ 3). ΠžΡ‚ΠΌΠ΅Ρ‚ΠΈΠΌ, Ρ‡Ρ‚ΠΎ ΠΈΠ· ΠΏΡ€ΠΎΠ²Π΅Π΄Π΅Π½Π½ΠΎΠ³ΠΎ рассуТдСния слСдуСт, Ρ‡Ρ‚ΠΎ выдСляСмая ΠΌΠΎΡ‰Π½ΠΎΡΡ‚ΡŒ Π±ΡƒΠ΄Π΅Ρ‚ ΠΎΡ‡Π΅Π½ΡŒ большой (опасная Π² Π±Ρ‹Ρ‚Ρƒ ситуация!) для ΠΌΠ°Π»ΠΎΠ³ΠΎ сопротивлСния внСшнСго участка Ρ†Π΅ΠΏΠΈ, Ρ‚.Π΅. Π² случаС ΠΊΠΎΡ€ΠΎΡ‚ΠΊΠΎΠ³ΠΎ замыкания, ΠΊΠΎΡ‚ΠΎΡ€ΠΎΠ³ΠΎ, Ρ‚Π°ΠΊΠΈΠΌ ΠΎΠ±Ρ€Π°Π·ΠΎΠΌ, Π½Π΅ΠΎΠ±Ρ…ΠΎΠ΄ΠΈΠΌΠΎ ΠΈΠ·Π±Π΅Π³Π°Ρ‚ΡŒ.

    Если Π±Ρ‹ Π²Π½ΡƒΡ‚Ρ€Π΅Π½Π½Π΅Π΅ сопротивлСниС источника Π±Ρ‹Π»ΠΎ Π±Ρ‹ ΠΌΠ½ΠΎΠ³ΠΎ большС внСшнСго сопротивлСния, Ρ‚ΠΎΠΊ Π² Ρ†Π΅ΠΏΠΈ опрСдСлялся Π±Ρ‹, Π³Π»Π°Π²Π½Ρ‹ΠΌ ΠΎΠ±Ρ€Π°Π·ΠΎΠΌ, Π²Π½ΡƒΡ‚Ρ€Π΅Π½Π½ΠΈΠΌ сопротивлСниСм источника, Π° ΠΎΡ‚ внСшнСго сопротивлСния зависСл Π±Ρ‹ слабо. Π’ этом случаС ΠΌΠΎΡ‰Π½ΠΎΡΡ‚ΡŒ Ρ‚ΠΎΠΊΠ° Π±Ρ‹Π»Π° Π±Ρ‹ прямо ΠΏΡ€ΠΎΠΏΠΎΡ€Ρ†ΠΈΠΎΠ½Π°Π»ΡŒΠ½Π° ΡΠΎΠΏΡ€ΠΎΡ‚ΠΈΠ²Π»Π΅Π½ΠΈΡŽ участка Ρ†Π΅ΠΏΠΈ.

    Как ΠΎΠ±ΡΡƒΠΆΠ΄Π°Π»ΠΎΡΡŒ Π² Ρ€Π΅ΡˆΠ΅Π½ΠΈΠΈ ΠΏΡ€Π΅Π΄Ρ‹Π΄ΡƒΡ‰Π΅ΠΉ Π·Π°Π΄Π°Ρ‡ΠΈ, сопротивлСниС элСмСнта, Ρ€Π°Π±ΠΎΡ‚Π°ΡŽΡ‰Π΅Π³ΠΎ Π² Π±Ρ‹Ρ‚ΠΎΠ²ΠΎΠΉ элСктросСти Ρ€Π°Π²Π½ΠΎ , Π³Π΄Π΅ β€” номинальная ΠΌΠΎΡ‰Π½ΠΎΡΡ‚ΡŒ Π΄Π°Π½Π½ΠΎΠ³ΠΎ элСмСнта, β€” напряТСниС Π² сСти. ΠŸΠΎΡΡ‚ΠΎΠΌΡƒ ΠΎΡ‚Π½ΠΎΡˆΠ΅Π½ΠΈΠ΅ сопротивлСний Π»Π°ΠΌΠΏ ΠΌΠΎΡ‰Π½ΠΎΡΡ‚ΡŒΡŽ Π’Ρ‚ ΠΈ Π’Ρ‚, рассчитанных Π½Π° Ρ€Π°Π±ΠΎΡ‚Ρƒ Π² ΠΎΠ΄Π½ΠΎΠΉ ΠΈ Ρ‚ΠΎΠΉ ΠΆΠ΅ Π±Ρ‹Ρ‚ΠΎΠ²ΠΎΠΉ элСктричСской сСти (Π·Π°Π΄Π°Ρ‡Π° 21.2.3) Ρ€Π°Π²Π½ΠΎ

    (ΠΎΡ‚Π²Π΅Ρ‚ 2).

    ΠŸΠΎΡΠΊΠΎΠ»ΡŒΠΊΡƒ рСзисторы Π² Π·Π°Π΄Π°Ρ‡Π΅ 21.2.4 соСдинСны ΠΏΠΎΡΠ»Π΅Π΄ΠΎΠ²Π°Ρ‚Π΅Π»ΡŒΠ½ΠΎ, Ρ‚ΠΎ сила Ρ‚ΠΎΠΊΠ° Π² Π½ΠΈΡ… ΠΎΠ΄ΠΈΠ½Π°ΠΊΠΎΠ²Π°. ΠŸΠΎΡΡ‚ΠΎΠΌΡƒ ΠΈΠ· Π·Π°ΠΊΠΎΠ½Π° Ома для участка Ρ†Π΅ΠΏΠΈ Π·Π°ΠΊΠ»ΡŽΡ‡Π°Π΅ΠΌ, Ρ‡Ρ‚ΠΎ

    (ΠΎΡ‚Π²Π΅Ρ‚ 2).

    ΠŸΡ€ΠΈ ΠΏΠ°Ρ€Π°Π»Π»Π΅Π»ΡŒΠ½ΠΎΠΌ соСдинСнии Π»Π°ΠΌΠΏ (Π·Π°Π΄Π°Ρ‡Π° 21.2.5) напряТСниС Π½Π° Π½ΠΈΡ… ΠΎΠ΄ΠΈΠ½Π°ΠΊΠΎΠ²ΠΎ (см. Π²Π²Π΅Π΄Π΅Π½ΠΈΠ΅ ΠΊ настоящСй Π³Π»Π°Π²Π΅). ΠŸΠΎΡΡ‚ΠΎΠΌΡƒ ΠΈΠ· Π·Π°ΠΊΠΎΠ½Π° Ома для участка Ρ†Π΅ΠΏΠΈ слСдуСт, Ρ‡Ρ‚ΠΎ

    (ΠΎΡ‚Π²Π΅Ρ‚ 1).

    РассматриваСмый Π² Π·Π°Π΄Π°Ρ‡Π΅ 21.2.6 участок прСдставляСт собой Π΄Π²Π° ΠΏΠΎΡΠ»Π΅Π΄ΠΎΠ²Π°Ρ‚Π΅Π»ΡŒΠ½Ρ‹Ρ… соСдинСнных элСмСнта, ΠΎΠ΄ΠΈΠ½ ΠΈΠ· ΠΊΠΎΡ‚ΠΎΡ€Ρ‹Ρ… Π΅ΡΡ‚ΡŒ рСзистор 6 Ом, Π²Ρ‚ΠΎΡ€ΠΎΠΉ β€” Π΄Π²Π° Ρ‚Π°ΠΊΠΈΡ… ΠΆΠ΅ рСзистора, соСдинСнных ΠΏΠ°Ρ€Π°Π»Π»Π΅Π»ΡŒΠ½ΠΎ. По ΠΏΡ€Π°Π²ΠΈΠ»Π°ΠΌ слоТСния сопротивлСний Π½Π°Ρ…ΠΎΠ΄ΠΈΠΌ эквивалСнтноС сопротивлСниС Π²Ρ‚ΠΎΡ€ΠΎΠ³ΠΎ участка

    Π° Π·Π°Ρ‚Π΅ΠΌ ΠΈ эквивалСнтноС сопротивлСниС всСй Ρ†Π΅ΠΏΠΈ

    (ΠΎΡ‚Π²Π΅Ρ‚ 3).

    ΠŸΡ€ΠΈ Ρ€Π°Π·ΠΎΠΌΠΊΠ½ΡƒΡ‚ΠΎΠΌ ΠΊΠ»ΡŽΡ‡Π΅ сопротивлСниС участка Ρ†Π΅ΠΏΠΈ, Π΄Π°Π½Π½ΠΎΠ³ΠΎ Π² Π·Π°Π΄Π°Ρ‡Π΅ 21.2.7, ΠΌΠΎΠΆΠ½ΠΎ Π½Π°ΠΉΡ‚ΠΈ ΠΊΠ°ΠΊ Π² ΠΏΡ€Π΅Π΄Ρ‹Π΄ΡƒΡ‰Π΅ΠΉ Π·Π°Π΄Π°Ρ‡Π΅ , Π³Π΄Π΅ β€” сопротивлСниС ΠΊΠ°ΠΆΠ΄ΠΎΠ³ΠΎ рСзистора. Если ΠΊΠ»ΡŽΡ‡ Π·Π°ΠΌΠΊΠ½ΡƒΡ‚, Ρ‚ΠΎ Ρ†Π΅ΠΏΡŒ сводится ΠΊ ΠΎΠ΄Π½ΠΎΠΌΡƒ рСзистору (Ρ‚.ΠΊ. ΠΏΠ°Ρ€Π°Π»Π»Π΅Π»ΡŒΠ½ΠΎ Π΄Π²ΡƒΠΌ рСзисторам Π²ΠΊΠ»ΡŽΡ‡Π°Π΅Ρ‚ΡΡ ΠΏΡ€ΠΎΠ²ΠΎΠ΄Π½ΠΈΠΊ с ΠΏΡ€Π΅Π½Π΅Π±Ρ€Π΅ΠΆΠΈΠΌΠΎ ΠΌΠ°Π»Ρ‹ΠΌ сопротивлСниСм). ΠŸΠΎΡΡ‚ΠΎΠΌΡƒ Π² этом случаС сопротивлСниС Ρ†Π΅ΠΏΠΈ Ρ€Π°Π²Π½ΠΎ . Π’Π°ΠΊΠΈΠΌ ΠΎΠ±Ρ€Π°Π·ΠΎΠΌ, сопротивлСниС Π²Ρ‚ΠΎΡ€ΠΎΠΉ Ρ†Π΅ΠΏΠΈ составляСт Π΄Π²Π΅ Ρ‚Ρ€Π΅Ρ‚ΠΈ ΠΎΡ‚ сопротивлСния ΠΏΠ΅Ρ€Π²ΠΎΠΉ (ΠΎΡ‚Π²Π΅Ρ‚ 1).

    Как ΠΎΠ±ΡΡƒΠΆΠ΄Π°Π»ΠΎΡΡŒ Π² Ρ€Π΅ΡˆΠ΅Π½ΠΈΠΈ Π·Π°Π΄Π°Ρ‡ΠΈ 21.2.2, сопротивлСниС элСмСнта номинальной мощности , Ρ€Π°Π±ΠΎΡ‚Π°ΡŽΡ‰Π΅Π³ΠΎ Π² Π±Ρ‹Ρ‚ΠΎΠ²ΠΎΠΉ элСктросСти Ρ€Π°Π²Π½Π°

    Π³Π΄Π΅ Π’ β€” напряТСниС сСти. Из этой Ρ„ΠΎΡ€ΠΌΡƒΠ»Ρ‹ слСдуСт, Ρ‡Ρ‚ΠΎ Ρ‡Π΅ΠΌ большС номинальная ΠΌΠΎΡ‰Π½ΠΎΡΡ‚ΡŒ элСмСнта, Ρ‚Π΅ΠΌ мСньшС Π΄ΠΎΠ»ΠΆΠ½ΠΎ Π±Ρ‹Ρ‚ΡŒ Π΅Π³ΠΎ сопротивлСниС. Если Π΄Π²Π΅ Π»Π°ΠΌΠΏΡ‹ накаливания Π²ΠΊΠ»ΡŽΡ‡Π΅Π½Ρ‹ ΠΏΠΎΡΠ»Π΅Π΄ΠΎΠ²Π°Ρ‚Π΅Π»ΡŒΠ½ΠΎ (Π·Π°Π΄Π°Ρ‡Π° 21.2.8), Ρ‚ΠΎ сила Ρ‚ΠΎΠΊΠ° Π² Π½ΠΈΡ… ΠΎΠ΄ΠΈΠ½Π°ΠΊΠΎΠ²Π° ΠΈ ΠΎΡ‚Π½ΠΎΡˆΠ΅Π½ΠΈΠ΅ мощностСй Ρ‚ΠΎΠΊΠ° Π² этих Π»Π°ΠΌΠΏΠ°Ρ… Ρ€Π°Π²Π½ΠΎ ΠΎΡ‚Π½ΠΎΡˆΠ΅Π½ΠΈΡŽ ΠΈΡ… сопротивлСний. ΠžΡ‚ΡΡŽΠ΄Π° слСдуСт, Ρ‡Ρ‚ΠΎ ΠΎΡ‚Π½ΠΎΡˆΠ΅Π½ΠΈΠ΅ Ρ€Π΅Π°Π»ΡŒΠ½ΠΎ выдСляСмых Π² Π»Π°ΠΌΠΏΠ°Ρ… мощностСй ΠΈ ΠΎΠ±Ρ€Π°Ρ‚Π½ΠΎ ΠΎΡ‚Π½ΠΎΡˆΠ΅Π½ΠΈΡŽ Π½ΠΎΠΌΠΈΠ½Π°Π»ΡŒΠ½Ρ‹Ρ… мощностСй этих Π»Π°ΠΌΠΏ:

    (ΠΎΡ‚Π²Π΅Ρ‚ 2).

    Π Π°Π±ΠΎΡ‚Π°, ΡΠΎΠ²Π΅Ρ€ΡˆΠ°Π΅ΠΌΠ°Ρ элСктричСским ΠΏΠΎΠ»Π΅ΠΌ Π² ΠΏΡ€ΠΎΠ²ΠΎΠ΄Π½ΠΈΠΊΠ΅ ΠΏΡ€ΠΈ ΠΏΡ€ΠΎΡ‚Π΅ΠΊΠ°Π½ΠΈΠΈ ΠΏΠΎ Π½Π΅ΠΌΡƒ элСктричСского Ρ‚ΠΎΠΊΠ°, прСвращаСтся Π² ΡΠ½Π΅Ρ€Π³ΠΈΡŽ Ρ‚ΠΎΠΊΠ°, которая Π·Π°Ρ‚Π΅ΠΌ прСвращаСтся Π² Ρ‚Π΅ΠΏΠ»ΠΎΠ²ΡƒΡŽ ΡΠ½Π΅Ρ€Π³ΠΈΡŽ. ΠŸΠΎΡΡ‚ΠΎΠΌΡƒ Ρ€Π°Π±ΠΎΡ‚Ρƒ поля ΠΌΠΎΠΆΠ½ΠΎ Π½Π°ΠΉΡ‚ΠΈ ΠΈΠ· Π·Π°ΠΊΠΎΠ½Π° ДТоуля-Π›Π΅Π½Ρ†Π°. Для Ρ€Π°Π±ΠΎΡ‚Ρ‹ поля Π·Π° врСмя ΠΏΠΎΠ»ΡƒΡ‡Π°Π΅ΠΌ . Из этой Ρ„ΠΎΡ€ΠΌΡƒΠ»Ρ‹ Π½Π°Ρ…ΠΎΠ΄ΠΈΠΌ сопротивлСниС ΠΏΡ€ΠΎΠ²ΠΎΠ΄Π½ΠΈΠΊΠ° Π² Π·Π°Π΄Π°Ρ‡Π΅ 21.2.9 β€”

    (ΠΎΡ‚Π²Π΅Ρ‚ 1).

    ΠŸΠΎΡΠΊΠΎΠ»ΡŒΠΊΡƒ ΠΏΡ€ΠΈ ΠΏΠΎΡΠ»Π΅Π΄ΠΎΠ²Π°Ρ‚Π΅Π»ΡŒΠ½ΠΎΠΌ соСдинСнии рСзисторов Ρ‚ΠΎΠΊ Ρ‡Π΅Ρ€Π΅Π· ΠΊΠ°ΠΆΠ΄Ρ‹ΠΉ ΠΈΠ· Π½ΠΈΡ… ΠΎΠ΄ΠΈΠ½Π°ΠΊΠΎΠ², ΠΈΠ· Π·Π°ΠΊΠΎΠ½Π° ДТоуля-Π›Π΅Π½Ρ†Π° (22.8) Π·Π°ΠΊΠ»ΡŽΡ‡Π°Π΅ΠΌ, Ρ‡Ρ‚ΠΎ ΠΈΠ· Π΄Π²ΡƒΡ… сопротивлСний ΠΈ (Π·Π°Π΄Π°Ρ‡Π° 21.2.10; см. рисунок) наибольшСй Π±ΡƒΠ΄Π΅Ρ‚ ΠΌΠΎΡ‰Π½ΠΎΡΡ‚ΡŒ Ρ‚ΠΎΠΊΠ° Π½Π° сопротивлСнии , ΠΈΠ· Π΄Π²ΡƒΡ… сопротивлСний ΠΈ β€” Π½Π° сопротивлСнии . Π‘Ρ€Π°Π²Π½ΠΈΠΌ мощности Ρ‚ΠΎΠΊΠ° Π½Π° этих сопротивлСниях. Учитывая, Ρ‡Ρ‚ΠΎ ΠΏΡ€ΠΈ ΠΏΠ°Ρ€Π°Π»Π»Π΅Π»ΡŒΠ½ΠΎΠΌ соСдинСнии элСмСнтов элСктричСскоС напряТСниС Π½Π° ΠΊΠ°ΠΆΠ΄ΠΎΠΌ элСмСнтС ΠΎΠ΄ΠΈΠ½Π°ΠΊΠΎΠ²ΠΎΠ΅, Π° ΠΏΡ€ΠΈ ΠΏΠΎΡΠ»Π΅Π΄ΠΎΠ²Π°Ρ‚Π΅Π»ΡŒΠ½ΠΎΠΌ β€” ΡΠΊΠ»Π°Π΄Ρ‹Π²Π°ΡŽΡ‚ΡΡ значСния сопротивлСний, ΠΏΠΎΠ»ΡƒΡ‡ΠΈΠΌ ΠΈΠ· Π·Π°ΠΊΠΎΠ½ΠΎΠ² Ома для Π²Π΅Ρ€Ρ…Π½Π΅Π³ΠΎ ΠΈ Π½ΠΈΠΆΠ½Π΅Π³ΠΎ участков Ρ†Π΅ΠΏΠΈ ΠΈ Π·Π°ΠΊΠΎΠ½Π° ДТоуля-Π›Π΅Π½Ρ†Π°

    Π³Π΄Π΅ β€” элСктричСскоС напряТСниС, ΠΏΡ€ΠΈΠ»ΠΎΠΆΠ΅Π½Π½ΠΎΠ΅ ΠΊΠΎ всСй Ρ†Π΅ΠΏΠΈ. ΠŸΠΎΡΠΊΠΎΠ»ΡŒΠΊΡƒ Ρ‚ΠΎ Π² прСдставлСнной схСмС наибольшая ΠΌΠΎΡ‰Π½ΠΎΡΡ‚ΡŒ Π±ΡƒΠ΄Π΅Ρ‚ Π²Ρ‹Π΄Π΅Π»ΡΡ‚ΡŒΡΡ Π½Π° сопротивлСнии (ΠΎΡ‚Π²Π΅Ρ‚ 2).

    Π—Π°ΠΊΠΎΠ½ ΠΎΠΌΠ° — Ρ„ΠΎΡ€ΠΌΡƒΠ»ΠΈΡ€ΠΎΠ²ΠΊΠ° простыми словами, ΠΎΠΏΡ€Π΅Π΄Π΅Π»Π΅Π½ΠΈΠ΅,

    Π‘ΠΎΠΏΡ€ΠΎΡ‚ΠΈΠ²Π»Π΅Π½ΠΈΠ΅

    ΠŸΡ€Π΅Π΄ΡΡ‚Π°Π²ΡŒΡ‚Π΅, Ρ‡Ρ‚ΠΎ Π΅ΡΡ‚ΡŒ Ρ‚Ρ€ΡƒΠ±Π°, Π² ΠΊΠΎΡ‚ΠΎΡ€ΡƒΡŽ Π·Π°Ρ‚ΠΎΠ»ΠΊΠ°Π»ΠΈ ΠΊΠ°ΠΌΠ½ΠΈ. Π’ΠΎΠ΄Π°, которая ΠΏΡ€ΠΎΡ‚Π΅ΠΊΠ°Π΅Ρ‚ ΠΏΠΎ этой Ρ‚Ρ€ΡƒΠ±Π΅, станСт Ρ‚Π΅Ρ‡ΡŒ ΠΌΠ΅Π΄Π»Π΅Π½Π½Π΅Π΅, ΠΏΠΎΡ‚ΠΎΠΌΡƒ Ρ‡Ρ‚ΠΎ Ρƒ Π½Π΅Π΅ появилось сопротивлСниС. Π’ΠΎΡ‡Π½ΠΎ Ρ‚Π°ΠΊΠΆΠ΅ Π±ΡƒΠ΄Π΅Ρ‚ ΠΏΡ€ΠΎΠΈΡΡ…ΠΎΠ΄ΠΈΡ‚ΡŒ с элСктричСским Ρ‚ΠΎΠΊΠΎΠΌ.

    • Π‘ΠΎΠΏΡ€ΠΎΡ‚ΠΈΠ²Π»Π΅Π½ΠΈΠ΅ β€” физичСская Π²Π΅Π»ΠΈΡ‡ΠΈΠ½Π°, которая ΠΏΠΎΠΊΠ°Π·Ρ‹Π²Π°Π΅Ρ‚ ΡΠΏΠΎΡΠΎΠ±Π½ΠΎΡΡ‚ΡŒ ΠΏΡ€ΠΎΠ²ΠΎΠ΄Π½ΠΈΠΊΠ° ΠΏΡ€ΠΎΠΏΡƒΡΠΊΠ°Ρ‚ΡŒ элСктричСский Ρ‚ΠΎΠΊ. Π§Π΅ΠΌ Π²Ρ‹ΡˆΠ΅ сопротивлСниС, Ρ‚Π΅ΠΌ Π½ΠΈΠΆΠ΅ эта ΡΠΏΠΎΡΠΎΠ±Π½ΠΎΡΡ‚ΡŒ.

    Π’Π΅ΠΏΠ΅Ρ€ΡŒ сдСлаСм Β«ΠΊΠ°ΠΌΠ΅Π½Π½Ρ‹ΠΉ участок» Π΄Π»ΠΈΠ½Π½Π΅Π΅, Ρ‚ΠΎ Π΅ΡΡ‚ΡŒ Π΄ΠΎΠ±Π°Π²ΠΈΠΌ Π΅Ρ‰Π΅ ΠΊΠ°ΠΌΠ½Π΅ΠΉ. Π’ΠΎΠ΄Π΅ Π±ΡƒΠ΄Π΅Ρ‚ Π΅Ρ‰Π΅ слоТнСС Ρ‚Π΅Ρ‡ΡŒ.

    Π‘Π΄Π΅Π»Π°Π΅ΠΌ Ρ‚Ρ€ΡƒΠ±Ρƒ ΡˆΠΈΡ€Π΅, оставив количСство ΠΊΠ°ΠΌΠ½Π΅ΠΉ Ρ‚Π΅ΠΌ ΠΆΠ΅ β€” Π²ΠΎΠ΄Π΅ ΠΏΠΎΠ»Π΅Π³Ρ‡Π°Π΅Ρ‚, ΠΏΠΎΡ‚ΠΎΠΊ увСличится.

    Π’Π΅ΠΏΠ΅Ρ€ΡŒ Π·Π°ΠΌΠ΅Π½ΠΈΠΌ ΡˆΠ΅Ρ€ΠΎΡ…ΠΎΠ²Π°Ρ‚Ρ‹Π΅ ΠΊΠ°ΠΌΠ½ΠΈ, ΠΊΠΎΡ‚ΠΎΡ€Ρ‹Π΅ ΠΌΡ‹ Π½Π°Π±Ρ€Π°Π»ΠΈ Π½Π° стройкС, Π½Π° Π³Π»Π°Π΄ΠΊΠΈΠ΅ ΠΊΠ°ΠΌΡƒΡˆΠΊΠΈ ΠΈΠ· моря. Π§Π΅Ρ€Π΅Π· Π½ΠΈΡ… ΠΏΡ€ΠΎΡ…ΠΎΠ΄ΠΈΡ‚ΡŒ Ρ‚ΠΎΠΆΠ΅ Π»Π΅Π³Ρ‡Π΅, Π° Π·Π½Π°Ρ‡ΠΈΡ‚ сопротивлСниС ΡƒΠΌΠ΅Π½ΡŒΡˆΠ°Π΅Ρ‚ΡΡ.

    ЭлСктричСский Ρ‚ΠΎΠΊ Ρ€Π΅Π°Π³ΠΈΡ€ΡƒΠ΅Ρ‚ Π½Π° эти ΠΏΠ°Ρ€Π°ΠΌΠ΅Ρ‚Ρ€Ρ‹ Π°Π½Π°Π»ΠΎΠ³ΠΈΡ‡Π½Ρ‹ΠΌ ΠΎΠ±Ρ€Π°Π·ΠΎΠΌ: ΠΏΡ€ΠΈ ΡƒΠ΄Π»ΠΈΠ½Π΅Π½ΠΈΠΈ ΠΏΡ€ΠΎΠ²ΠΎΠ΄Π½ΠΈΠΊΠ° сопротивлСниС увСличиваСтся, ΠΏΡ€ΠΈ ΡƒΠ²Π΅Π»ΠΈΡ‡Π΅Π½ΠΈΠΈ ΠΏΠΎΠΏΠ΅Ρ€Π΅Ρ‡Π½ΠΎΠ³ΠΎ сСчСния (ΡˆΠΈΡ€ΠΈΠ½Ρ‹) ΠΏΡ€ΠΎΠ²ΠΎΠ΄Π½ΠΈΠΊΠ° сопротивлСниС ΡƒΠΌΠ΅Π½ΡŒΡˆΠ°Π΅Ρ‚ΡΡ, Π° Ссли Π·Π°ΠΌΠ΅Π½ΠΈΡ‚ΡŒ ΠΌΠ°Ρ‚Π΅Ρ€ΠΈΠ°Π» β€” измСнится Π² зависимости ΠΎΡ‚ ΠΌΠ°Ρ‚Π΅Ρ€ΠΈΠ°Π»Π°. 2.

    Π—Π½Π°ΠΉΡ‚Π΅!

    БИ β€” мСТдународная систСма Π΅Π΄ΠΈΠ½ΠΈΡ†. Β«ΠŸΠ΅Ρ€Π΅Π²Π΅ΡΡ‚ΠΈ Π² БИ» ΠΎΠ·Π½Π°Ρ‡Π°Π΅Ρ‚ ΠΏΠ΅Ρ€Π΅Π²ΠΎΠ΄ всСх Π²Π΅Π»ΠΈΡ‡ΠΈΠ½ Π² ΠΌΠ΅Ρ‚Ρ€Ρ‹, ΠΊΠΈΠ»ΠΎΠ³Ρ€Π°ΠΌΠΌΡ‹, сСкунды ΠΈ Π΄Ρ€ΡƒΠ³ΠΈΠ΅ Π΅Π΄ΠΈΠ½ΠΈΡ†Ρ‹ измСрСния Π±Π΅Π· приставок. Π˜ΡΠΊΠ»ΡŽΡ‡Π΅Π½ΠΈΠ΅ составляСт ΠΊΠΈΠ»ΠΎΠ³Ρ€Π°ΠΌΠΌ с приставкой Β«ΠΊΠΈΠ»ΠΎΒ».

    • УдСльноС сопротивлСниС ΠΏΡ€ΠΎΠ²ΠΎΠ΄Π½ΠΈΠΊΠ° β€” это физичСская Π²Π΅Π»ΠΈΡ‡ΠΈΠ½Π°, которая ΠΏΠΎΠΊΠ°Π·Ρ‹Π²Π°Π΅Ρ‚ ΡΠΏΠΎΡΠΎΠ±Π½ΠΎΡΡ‚ΡŒ ΠΌΠ°Ρ‚Π΅Ρ€ΠΈΠ°Π»Π° ΠΏΡ€ΠΎΠΏΡƒΡΠΊΠ°Ρ‚ΡŒ элСктричСский Ρ‚ΠΎΠΊ. Π­Ρ‚ΠΎ табличная Π²Π΅Π»ΠΈΡ‡ΠΈΠ½Π°, ΠΎΠ½Π° зависит Ρ‚ΠΎΠ»ΡŒΠΊΠΎ ΠΎΡ‚ ΠΌΠ°Ρ‚Π΅Ρ€ΠΈΠ°Π»Π°.

    Π’Π°Π±Π»ΠΈΡ†Π° ΡƒΠ΄Π΅Π»ΡŒΠ½Ρ‹Ρ… сопротивлСний Ρ€Π°Π·Π»ΠΈΡ‡Π½Ρ‹Ρ… ΠΌΠ°Ρ‚Π΅Ρ€ΠΈΠ°Π»ΠΎΠ²

    УдСльноС сопротивлСниС

    ρ, Ом*мм2/м

    УдСльноС сопротивлСниС

    ρ, Ом*мм2/м

    Алюминий

    0,028

    Π‘Ρ€ΠΎΠ½Π·Π°

    0,095 — 0,1

    Висмут

    1,2

    Π’ΠΎΠ»ΡŒΡ„Ρ€Π°ΠΌ

    0,05

    Π–Π΅Π»Π΅Π·ΠΎ

    0,1

    Π—ΠΎΠ»ΠΎΡ‚ΠΎ

    0,023

    Π˜Ρ€ΠΈΠ΄ΠΈΠΉ

    0,0474

    ΠšΠΎΠ½ΡΡ‚Π°Π½Ρ‚Π°Π½ ( сплав Ni-Cu + Mn)

    0,5

    Π›Π°Ρ‚ΡƒΠ½ΡŒ

    0,025 — 0,108

    Магний

    0,045

    Манганин (сплав ΠΌΠ΅Π΄ΠΈ ΠΌΠ°Ρ€Π³Π°Π½Ρ†Π° ΠΈ никСля — ΠΏΡ€ΠΈΠ±ΠΎΡ€Π½Ρ‹ΠΉ)

    0,43 — 0,51

    МСдь

    0,0175

    МолибдСн

    0,059

    ΠΠ΅ΠΉΠ·ΠΈΠ»ΡŒΠ±Π΅Ρ€ (сплав ΠΌΠ΅Π΄ΠΈ Ρ†ΠΈΠ½ΠΊΠ° ΠΈ никСля)

    0,2

    Натрий

    0,047

    НикСлин ( сплав мСди и никСля)

    0,42

    НикСль

    0,087

    Нихром ( сплав никСля Ρ…Ρ€ΠΎΠΌΠ° ΠΆΠ΅Π»Π΅Π·Ρ‹ ΠΈ ΠΌΠ°Ρ€Π³Π°Π½Ρ†Π°)

    1,05 — 1,4

    Олово

    0,12

    ΠŸΠ»Π°Ρ‚ΠΈΠ½Π°

    0. 107

    Π Ρ‚ΡƒΡ‚ΡŒ

    0,94

    Π‘Π²ΠΈΠ½Π΅Ρ†

    0,22

    Π‘Π΅Ρ€Π΅Π±Ρ€ΠΎ

    0,015

    Π‘Ρ‚Π°Π»ΡŒ

    0,103 — 0,137

    Π’ΠΈΡ‚Π°Π½

    0,6

    Π₯Ρ€ΠΎΠΌΠ°Π»ΡŒ

    1,3 — 1,5

    Π¦ΠΈΠ½ΠΊ

    0,054

    Π§ΡƒΠ³ΡƒΠ½

    0,5-1,0

    РСзистор

    ВсС Ρ€Π΅Π°Π»ΡŒΠ½Ρ‹Π΅ ΠΏΡ€ΠΎΠ²ΠΎΠ΄Π½ΠΈΠΊΠΈ ΠΈΠΌΠ΅ΡŽΡ‚ сопротивлСниС, Π½ΠΎ Π΅Π³ΠΎ ΡΡ‚Π°Ρ€Π°ΡŽΡ‚ΡΡ ΡΠ΄Π΅Π»Π°Ρ‚ΡŒ Π½Π΅Π·Π½Π°Ρ‡ΠΈΡ‚Π΅Π»ΡŒΠ½Ρ‹ΠΌ. Π’ Π·Π°Π΄Π°Ρ‡Π°Ρ… Π²ΠΎΠΎΠ±Ρ‰Π΅ ΠΈΡΠΏΠΎΠ»ΡŒΠ·ΡƒΡŽΡ‚ словосочСтаниС Β«ΠΈΠ΄Π΅Π°Π»ΡŒΠ½Ρ‹ΠΉ ΠΏΡ€ΠΎΠ²ΠΎΠ΄Π½ΠΈΠΊΒ», Π° Π·Π½Π°Ρ‡ΠΈΡ‚ Π»ΠΈΡˆΠ°ΡŽΡ‚ Π΅Π³ΠΎ сопротивлСния.

    Из-Π·Π° Ρ‚ΠΎΠ³ΠΎ, Ρ‡Ρ‚ΠΎ ΠΏΡ€ΠΎΠ²ΠΎΠ΄Π½ΠΈΠΊ Ρƒ нас Β«ΠΊΡ€ΡƒΠ³ΠΎΠΌ-Π±Π΅Π³ΠΎΠΌ-Ρ‚Π°ΠΊΠΎΠΉ-ΠΈΠ΄Π΅Π°Π»ΡŒΠ½Ρ‹ΠΉΒ», Ρ‡Π°Ρ‰Π΅ всСго Π·Π° сопротивлСниС Π² Ρ†Π΅ΠΏΠΈ ΠΎΡ‚Π²Π΅Ρ‡Π°Π΅Ρ‚ рСзистор. Π­Ρ‚ΠΎ устройство, ΠΊΠΎΡ‚ΠΎΡ€ΠΎΠ΅ Π½Π°Π³Ρ€ΡƒΠΆΠ°Π΅Ρ‚ Ρ†Π΅ΠΏΡŒ сопротивлСниСм.

    Π’ΠΎΡ‚ Ρ‚Π°ΠΊ рСзистор изобраТаСтся Π½Π° схСмах:


    Π’ школьном курсС Ρ„ΠΈΠ·ΠΈΠΊΠΈ ΠΈΡΠΏΠΎΠ»ΡŒΠ·ΡƒΡŽΡ‚ ЕвропСйскоС ΠΎΠ±ΠΎΠ·Π½Π°Ρ‡Π΅Π½ΠΈΠ΅, поэтому Π·Π°ΠΏΠΎΠΌΠΈΠ½Π°Π΅ΠΌ Ρ‚ΠΎΠ»ΡŒΠΊΠΎ Π΅Π³ΠΎ. АмСриканскоС ΠΎΠ±ΠΎΠ·Π½Π°Ρ‡Π΅Π½ΠΈΠ΅ ΠΌΠΎΠΆΠ½ΠΎ Π²ΡΡ‚Ρ€Π΅Ρ‚ΠΈΡ‚ΡŒ, Π½Π°ΠΏΡ€ΠΈΠΌΠ΅Ρ€, Π² ΠΏΡ€ΠΎΠ³Ρ€Π°ΠΌΠΌΠ΅ Micro-Cap, Π² ΠΊΠΎΡ‚ΠΎΡ€ΠΎΠΉ ΠΈΠ½ΠΆΠ΅Π½Π΅Ρ€Ρ‹ ΠΌΠΎΠ΄Π΅Π»ΠΈΡ€ΡƒΡŽΡ‚ схСмы.

    Π’ΠΎΡ‚ Ρ‚Π°ΠΊ рСзистор выглядит Π² СстСствСнной срСдС обитания:


    ΠŸΠΎΠ»ΠΎΡΠΎΡ‡ΠΊΠΈ Π½Π° Π½Π΅ΠΌ ΠΏΠΎΠΊΠ°Π·Ρ‹Π²Π°ΡŽΡ‚ Π΅Π³ΠΎ сопротивлСниС.

    На сайтС ΠΊΠΎΠΌΠΏΠ°Π½ΠΈΠΈ Ekits, которая занимаСтся ΠΏΡ€ΠΎΠ΄Π°ΠΆΠ΅ΠΉ элСктронных ΠΌΠΎΠ΄ΡƒΠ»Π΅ΠΉ, ΠΌΠΎΠΆΠ½ΠΎ Π²Ρ‹Π±Ρ€Π°Ρ‚ΡŒ Ρ†Π²Π΅Ρ‚ рСзистора ΠΈ ΡƒΠ·Π½Π°Ρ‚ΡŒ Π·Π½Π°Ρ‡Π΅Π½ΠΈΠ΅ Π΅Π³ΠΎ сопротивлСния:


    Π˜ΡΡ‚ΠΎΡ‡Π½ΠΈΠΊ: сайт ΠΊΠΎΠΌΠΏΠ°Π½ΠΈΠΈ Ekits

    О Ρ‚ΠΎΠΌ, Π·Π°Ρ‡Π΅ΠΌ Π΄ΠΎΠΏΠΎΠ»Π½ΠΈΡ‚Π΅Π»ΡŒΠ½ΠΎ Π½Π°Π³Ρ€ΡƒΠΆΠ°Ρ‚ΡŒ сопротивлСниСм Ρ†Π΅ΠΏΡŒ, ΠΌΡ‹ ΠΏΠΎΠ³ΠΎΠ²ΠΎΡ€ΠΈΠΌ Π² этой ΠΆΠ΅ ΡΡ‚Π°Ρ‚ΡŒΠ΅ Ρ‡ΡƒΡ‚ΡŒ ΠΏΠΎΠ·ΠΆΠ΅.

    НС ΡΠΎΠΏΡ€ΠΎΡ‚ΠΈΠ²Π»ΡΠΉΡ‚Π΅ΡΡŒ Π·ΠΎΠ²Ρƒ сСрдца ΠΈ Π·Π°ΠΏΠΈΡˆΠΈΡ‚Π΅ Ρ€Π΅Π±Π΅Π½ΠΊΠ° Π² ΡΠΎΠ²Ρ€Π΅ΠΌΠ΅Π½Π½ΡƒΡŽ ΡˆΠΊΠΎΠ»Ρƒ Skysmart. Π—Π΄Π΅ΡΡŒ школьники Ρ€Π΅ΡˆΠ°ΡŽΡ‚ Π·Π°Ρ…Π²Π°Ρ‚Ρ‹Π²Π°ΡŽΡ‰ΠΈΠ΅ Π·Π°Π΄Π°Ρ‡ΠΊΠΈ ΠΏΠΎ Ρ„ΠΈΠ·ΠΈΠΊΠ΅ ΠΈ ΠΏΠΎΠ½ΠΈΠΌΠ°ΡŽΡ‚, ΠΊΠ°ΠΊ это пригодится Π² ΠΆΠΈΠ·Π½ΠΈ.

    А Π΅Ρ‰Π΅ слСдят Π·Π° прогрСссом Π² Π»ΠΈΡ‡Π½ΠΎΠΌ ΠΊΠ°Π±ΠΈΠ½Π΅Ρ‚Π΅, Π·Π°Π΄Π°ΡŽΡ‚ учитСлям Π»ΡŽΠ±Ρ‹Π΅ β€” Π΄Π°ΠΆΠ΅ самыС Π½Π΅Π»ΠΎΠ²ΠΊΠΈΠ΅ β€” вопросы ΠΈ Ρ‡ΡƒΠ²ΡΡ‚Π²ΡƒΡŽΡ‚ сСбя ΡƒΠ²Π΅Ρ€Π΅Π½Π½Π΅Π΅ Π½Π° ΡˆΠΊΠΎΠ»ΡŒΠ½Ρ‹Ρ… экзамСнах ΠΈ ΠΊΠΎΠ½Ρ‚Ρ€ΠΎΠ»ΡŒΠ½Ρ‹Ρ….2/ΠΌ]

    Π—Π°ΠΊΠΎΠ½ Ома для участка Ρ†Π΅ΠΏΠΈ

    Π‘ ΠΊΠ°ΠΌΡƒΡˆΠΊΠ°ΠΌΠΈ Π² Ρ‚Ρ€ΡƒΠ±Π΅ всС понятно, Π½ΠΎ Π½Π΅ Ρ‚ΠΎΠ»ΡŒΠΊΠΎ ΠΆΠ΅ ΠΎΡ‚ Π½ΠΈΡ… зависит сила, с ΠΊΠΎΡ‚ΠΎΡ€ΠΎΠΉ ΠΏΠΎΡ‚ΠΎΠΊ Π²ΠΎΠ΄Ρ‹ ΠΈΠ΄Π΅Ρ‚ ΠΏΠΎ Ρ‚Ρ€ΡƒΠ±Π΅ β€” ΠΎΡ‚ насоса, ΠΊΠΎΡ‚ΠΎΡ€Ρ‹ΠΌ ΠΌΡ‹ эту Π²ΠΎΠ΄Ρƒ ΠΊΠ°Ρ‡Π°Π΅ΠΌ, Ρ‚ΠΎΠΆΠ΅ зависит. Π§Π΅ΠΌ сильнСС ΠΊΠ°Ρ‡Π°Π΅ΠΌ, Ρ‚Π΅ΠΌ большС Ρ‚Π΅Ρ‡Π΅Π½ΠΈΠ΅. Π’ элСктричСской Ρ†Π΅ΠΏΠΈ Ρ„ΡƒΠ½ΠΊΡ†ΠΈΡŽ насоса выполняСт источник Ρ‚ΠΎΠΊΠ°.

    НапримСр, источником ΠΌΠΎΠΆΠ΅Ρ‚ Π±Ρ‹Ρ‚ΡŒ Π³Π°Π»ΡŒΠ²Π°Π½ΠΈΡ‡Π΅ΡΠΊΠΈΠΉ элСмСнт (привычная Π±Π°Ρ‚Π°Ρ€Π΅ΠΉΠΊΠ°). Π‘Π°Ρ‚Π°Ρ€Π΅ΠΉΠΊΠ° Ρ€Π°Π±ΠΎΡ‚Π°Π΅Ρ‚ Π½Π° основС химичСских Ρ€Π΅Π°ΠΊΡ†ΠΈΠΉ Π²Π½ΡƒΡ‚Ρ€ΠΈ Π½Π΅Π΅. Π­Ρ‚ΠΈ Ρ€Π΅Π°ΠΊΡ†ΠΈΠΈ Π²Ρ‹Π΄Π΅Π»ΡΡŽΡ‚ ΡΠ½Π΅Ρ€Π³ΠΈΡŽ, которая ΠΏΠΎΡ‚ΠΎΠΌ пСрСдаСтся элСктричСской Ρ†Π΅ΠΏΠΈ.

    Π£ любого источника ΠΎΠ±ΡΠ·Π°Ρ‚Π΅Π»ΡŒΠ½ΠΎ Π΅ΡΡ‚ΡŒ полюса β€” «плюс» ΠΈ «минус». Полюса β€” это Π΅Π³ΠΎ ΠΊΡ€Π°ΠΉΠ½ΠΈΠ΅ полоТСния, ΠΏΠΎ сути ΠΊΠ»Π΅ΠΌΠΌΡ‹, ΠΊ ΠΊΠΎΡ‚ΠΎΡ€Ρ‹ΠΌ присоСдиняСтся элСктричСская Ρ†Π΅ΠΏΡŒ. БобствСнно, Ρ‚ΠΎΠΊ ΠΊΠ°ΠΊ Ρ€Π°Π· Ρ‚Π΅Ρ‡Π΅Ρ‚ ΠΎΡ‚ Β«+Β» ΠΊ Β«-Β».


    Π£ нас ΡƒΠΆΠ΅ Π΅ΡΡ‚ΡŒ Π΄Π²Π΅ Π²Π΅Π»ΠΈΡ‡ΠΈΠ½Ρ‹, ΠΎΡ‚ ΠΊΠΎΡ‚ΠΎΡ€Ρ‹Ρ… зависит элСктричСский Ρ‚ΠΎΠΊ Π² Ρ†Π΅ΠΏΠΈ β€” напряТСниС ΠΈ сопротивлСниС. ΠšΠ°ΠΆΠ΅Ρ‚ΡΡ, ΠΏΠΎΡ€Π° ΠΎΠ±ΡŠΠ΅Π΄ΠΈΠ½ΡΡ‚ΡŒ ΠΈΡ… Π² Π·Π°ΠΊΠΎΠ½.

    Π‘ΠΈΠ»Π° Ρ‚ΠΎΠΊΠ° Π² участкС Ρ†Π΅ΠΏΠΈ прямо ΠΏΡ€ΠΎΠΏΠΎΡ€Ρ†ΠΈΠΎΠ½Π°Π»ΡŒΠ½Π° Π½Π°ΠΏΡ€ΡΠΆΠ΅Π½ΠΈΡŽ Π½Π° Π΅Π³ΠΎ ΠΊΠΎΠ½Ρ†Π°Ρ… ΠΈ ΠΎΠ±Ρ€Π°Ρ‚Π½ΠΎ ΠΏΡ€ΠΎΠΏΠΎΡ€Ρ†ΠΈΠΎΠ½Π°Π»ΡŒΠ½Π° Π΅Π³ΠΎ ΡΠΎΠΏΡ€ΠΎΡ‚ΠΈΠ²Π»Π΅Π½ΠΈΡŽ.

    ΠœΠ°Ρ‚Π΅ΠΌΠ°Ρ‚ΠΈΡ‡Π΅ΡΠΊΠΈ Π΅Π³ΠΎ ΠΌΠΎΠΆΠ½ΠΎ ΠΎΠΏΠΈΡΠ°Ρ‚ΡŒ Π²ΠΎΡ‚ Ρ‚Π°ΠΊ:

    Π—Π°ΠΊΠΎΠ½ Ома для участка Ρ†Π΅ΠΏΠΈ

    I = U/R

    I β€” сила Ρ‚ΠΎΠΊΠ° [A]

    U β€” напряТСниС [Π’]

    R β€” сопротивлСниС [Ом]

    НапряТСниС измСряСтся Π² Π’ΠΎΠ»ΡŒΡ‚Π°Ρ… ΠΈ ΠΏΠΎΠΊΠ°Π·Ρ‹Π²Π°Π΅Ρ‚ Ρ€Π°Π·Π½ΠΈΡ†Ρƒ ΠΌΠ΅ΠΆΠ΄Ρƒ двумя Ρ‚ΠΎΡ‡ΠΊΠ°ΠΌΠΈ Ρ†Π΅ΠΏΠΈ: ΠΎΡ‚ этой Ρ€Π°Π·Π½ΠΈΡ†Ρ‹ зависит, насколько сильно Π±ΡƒΠ΄Π΅Ρ‚ Ρ‚Π΅Ρ‡ΡŒ Ρ‚ΠΎΠΊ β€” Ρ‡Π΅ΠΌ большС Ρ€Π°Π·Π½ΠΈΡ†Π°, Ρ‚Π΅ΠΌ Π²Ρ‹ΡˆΠ΅ напряТСниС ΠΈ Ρ‚ΠΎΠΊ Π±ΡƒΠ΄Π΅Ρ‚ Ρ‚Π΅Ρ‡ΡŒ сильнСС.

    Π‘ΠΈΠ»Π° Ρ‚ΠΎΠΊΠ° измСряСтся Π² АмпСрах, Π° ΠΏΠΎΠ΄Ρ€ΠΎΠ±Π½Π΅Π΅ ΠΎ Π½Π΅ΠΉ Π²Ρ‹ ΠΌΠΎΠΆΠ΅Ρ‚Π΅ ΠΏΡ€ΠΎΡ‡ΠΈΡ‚Π°Ρ‚ΡŒ Π² нашСй ΡΡ‚Π°Ρ‚ΡŒΠ΅ πŸ˜‡

    Π”Π°Π²Π°ΠΉΡ‚Π΅ Ρ€Π΅ΡˆΠΈΠΌ нСсколько Π·Π°Π΄Π°Ρ‡ Π½Π° Π—Π°ΠΊΠΎΠ½ Ома для участка Ρ†Π΅ΠΏΠΈ.

    Π—Π°Π΄Π°Ρ‡Π° Ρ€Π°Π·

    Найти силу Ρ‚ΠΎΠΊΠ° Π² Π»Π°ΠΌΠΏΠΎΡ‡ΠΊΠ΅ накаливания, Ссли Ρ‚ΠΎΡ€ΡˆΠ΅Ρ€ Π²ΠΊΠ»ΡŽΡ‡ΠΈΠ»ΠΈ Π² ΡΠ΅Ρ‚ΡŒ напряТСниСм 220 Π’, Π° сопротивлСниС Π½ΠΈΡ‚ΠΈ накаливания Ρ€Π°Π²Π½ΠΎ 880 Ом.2/ΠΌ

    ΠžΠ±Ρ€Π°Ρ‚ΠΈΠΌΡΡ ΠΊ Ρ‚Π°Π±Π»ΠΈΡ†Π΅ ΡƒΠ΄Π΅Π»ΡŒΠ½Ρ‹Ρ… сопротивлСний ΠΌΠ°Ρ‚Π΅Ρ€ΠΈΠ°Π»ΠΎΠ², Ρ‡Ρ‚ΠΎΠ±Ρ‹ Π²Ρ‹ΡΡΠ½ΠΈΡ‚ΡŒ, ΠΈΠ· ΠΊΠ°ΠΊΠΎΠ³ΠΎ ΠΌΠ°Ρ‚Π΅Ρ€ΠΈΠ°Π»Π° сдСлана эта Π½ΠΈΡ‚ΡŒ накаливания.

    Π’Π°Π±Π»ΠΈΡ†Π° ΡƒΠ΄Π΅Π»ΡŒΠ½Ρ‹Ρ… сопротивлСний Ρ€Π°Π·Π»ΠΈΡ‡Π½Ρ‹Ρ… ΠΌΠ°Ρ‚Π΅Ρ€ΠΈΠ°Π»ΠΎΠ²

    УдСльноС сопротивлСниС

    ρ, Ом*мм2/м

    УдСльноС сопротивлСниС

    ρ, Ом*мм2/м

    Алюминий

    0,028

    Π‘Ρ€ΠΎΠ½Π·Π°

    0,095 — 0,1

    Висмут

    1,2

    Π’ΠΎΠ»ΡŒΡ„Ρ€Π°ΠΌ

    0,05

    Π–Π΅Π»Π΅Π·ΠΎ

    0,1

    Π—ΠΎΠ»ΠΎΡ‚ΠΎ

    0,023

    Π˜Ρ€ΠΈΠ΄ΠΈΠΉ

    0,0474

    ΠšΠΎΠ½ΡΡ‚Π°Π½Ρ‚Π°Π½ ( сплав Ni-Cu + Mn)

    0,5

    Π›Π°Ρ‚ΡƒΠ½ΡŒ

    0,025 — 0,108

    Магний

    0,045

    Манганин (сплав ΠΌΠ΅Π΄ΠΈ ΠΌΠ°Ρ€Π³Π°Π½Ρ†Π° ΠΈ никСля — ΠΏΡ€ΠΈΠ±ΠΎΡ€Π½Ρ‹ΠΉ)

    0,43 — 0,51

    МСдь

    0,0175

    МолибдСн

    0,059

    ΠΠ΅ΠΉΠ·ΠΈΠ»ΡŒΠ±Π΅Ρ€ (сплав ΠΌΠ΅Π΄ΠΈ Ρ†ΠΈΠ½ΠΊΠ° ΠΈ никСля)

    0,2

    Натрий

    0,047

    НикСлин ( сплав мСди и никСля)

    0,42

    НикСль

    0,087

    Нихром ( сплав никСля Ρ…Ρ€ΠΎΠΌΠ° ΠΆΠ΅Π»Π΅Π·Ρ‹ ΠΈ ΠΌΠ°Ρ€Π³Π°Π½Ρ†Π°)

    1,05 — 1,4

    Олово

    0,12

    ΠŸΠ»Π°Ρ‚ΠΈΠ½Π°

    0.107

    Π Ρ‚ΡƒΡ‚ΡŒ

    0,94

    Π‘Π²ΠΈΠ½Π΅Ρ†

    0,22

    Π‘Π΅Ρ€Π΅Π±Ρ€ΠΎ

    0,015

    Π‘Ρ‚Π°Π»ΡŒ

    0,103 — 0,137

    Π’ΠΈΡ‚Π°Π½

    0,6

    Π₯Ρ€ΠΎΠΌΠ°Π»ΡŒ

    1,3 — 1,5

    Π¦ΠΈΠ½ΠΊ

    0,054

    Π§ΡƒΠ³ΡƒΠ½

    0,5-1,0

    ΠžΡ‚Π²Π΅Ρ‚: Π½ΠΈΡ‚ΡŒ накаливания сдСлана ΠΈΠ· константана.

    Π—Π°ΠΊΠΎΠ½ Ома для ΠΏΠΎΠ»Π½ΠΎΠΉ Ρ†Π΅ΠΏΠΈ

    ΠœΡ‹ Ρ€Π°Π·ΠΎΠ±Ρ€Π°Π»ΠΈΡΡŒ с Π·Π°ΠΊΠΎΠ½ΠΎΠΌ Ома для участка Ρ†Π΅ΠΏΠΈ. А Ρ‚Π΅ΠΏΠ΅Ρ€ΡŒ Π΄Π°Π²Π°ΠΉΡ‚Π΅ ΡƒΠ·Π½Π°Π΅ΠΌ, Ρ‡Ρ‚ΠΎ происходит, Ссли Ρ†Π΅ΠΏΡŒ полная: Ρƒ Π½Π΅Π΅ Π΅ΡΡ‚ΡŒ источник, ΠΏΡ€ΠΎΠ²ΠΎΠ΄Π½ΠΈΠΊΠΈ, рСзисторы ΠΈ Π΄Ρ€ΡƒΠ³ΠΈΠ΅ элСмСнты.

    Π’ Ρ‚Π°ΠΊΠΎΠΌ случаС вводится Π—Π°ΠΊΠΎΠ½ Ома для ΠΏΠΎΠ»Π½ΠΎΠΉ Ρ†Π΅ΠΏΠΈ: сила Ρ‚ΠΎΠΊΠ° Π² ΠΏΠΎΠ»Π½ΠΎΠΉ Ρ†Π΅ΠΏΠΈ Ρ€Π°Π²Π½Π° ΠΎΡ‚Π½ΠΎΡˆΠ΅Π½ΠΈΡŽ Π­Π”Π‘ Ρ†Π΅ΠΏΠΈ ΠΊ Π΅Π΅ ΠΏΠΎΠ»Π½ΠΎΠΌΡƒ ΡΠΎΠΏΡ€ΠΎΡ‚ΠΈΠ²Π»Π΅Π½ΠΈΡŽ.

    Π’Π°ΠΊ, стоп. Блишком ΠΌΠ½ΠΎΠ³ΠΎ Π½Π΅Π·Π½Π°ΠΊΠΎΠΌΡ‹Ρ… слов β€” разбираСмся ΠΏΠΎ-порядку.

    Π§Ρ‚ΠΎ Ρ‚Π°ΠΊΠΎΠ΅ Π­Π”Π‘ ΠΈ ΠΎΡ‚ΠΊΡƒΠ΄Π° ΠΎΠ½Π° бСрСтся

    Π­Π”Π‘ Ρ€Π°ΡΡˆΠΈΡ„Ρ€ΠΎΠ²Ρ‹Π²Π°Π΅Ρ‚ΡΡ, ΠΊΠ°ΠΊ элСктродвиТущая сила. ΠžΠ±ΠΎΠ·Π½Π°Ρ‡Π°Π΅Ρ‚ΡΡ грСчСской Π±ΡƒΠΊΠ²ΠΎΠΉ Ξ΅ ΠΈ измСряСтся, ΠΊΠ°ΠΊ ΠΈ напряТСниС, Π² Π’ΠΎΠ»ΡŒΡ‚Π°Ρ….

    • Π­Π”Π‘ β€” это сила, которая Π΄Π²ΠΈΠΆΠ΅Ρ‚ заряТСнныС частицы Π² Ρ†Π΅ΠΏΠΈ. Она бСрСтся ΠΈΠ· источника Ρ‚ΠΎΠΊΠ°. НапримСр, ΠΈΠ· Π±Π°Ρ‚Π°Ρ€Π΅ΠΉΠΊΠΈ.

    Π₯имичСская рСакция Π²Π½ΡƒΡ‚Ρ€ΠΈ Π³Π°Π»ΡŒΠ²Π°Π½ΠΈΡ‡Π΅ΡΠΊΠΎΠ³ΠΎ элСмСнта (это синоним Π±Π°Ρ‚Π°Ρ€Π΅ΠΉΠΊΠΈ) происходит с Π²Ρ‹Π΄Π΅Π»Π΅Π½ΠΈΠ΅ΠΌ энСргии Π² ΡΠ»Π΅ΠΊΡ‚Ρ€ΠΈΡ‡Π΅ΡΠΊΡƒΡŽ Ρ†Π΅ΠΏΡŒ. ИмСнно эта энСргия заставляСт частицы Π΄Π²ΠΈΠ³Π°Ρ‚ΡŒΡΡ ΠΏΠΎ ΠΏΡ€ΠΎΠ²ΠΎΠ΄Π½ΠΈΠΊΡƒ.

    Π—Π°Ρ‡Π°ΡΡ‚ΡƒΡŽ напряТСниС ΠΈ Π­Π”Π‘ ΠΏΡ€ΠΈΡ€Π°Π²Π½ΠΈΠ²Π°ΡŽΡ‚ ΠΈ говорят, Ρ‡Ρ‚ΠΎ это ΠΎΠ΄Π½ΠΎ ΠΈ Ρ‚ΠΎ ΠΆΠ΅. Π€ΠΎΡ€ΠΌΠ°Π»ΡŒΠ½ΠΎ, это Π½Π΅ Ρ‚Π°ΠΊ, Π½ΠΎ ΠΏΡ€ΠΈ Ρ€Π΅ΡˆΠ΅Π½ΠΈΠΈ Π·Π°Π΄Π°Ρ‡ Ρ‡Π°Ρ‰Π΅ всСго ΠΈ ΠΏΡ€Π°Π²Π΄Π° Π½Π΅Ρ‚ Ρ€Π°Π·Π½ΠΈΡ†Ρ‹, Ρ‚Π°ΠΊ ΠΊΠ°ΠΊ эти Π²Π΅Π»ΠΈΡ‡ΠΈΠ½Ρ‹ ΠΎΠ±Π΅ ΠΈΠ·ΠΌΠ΅Ρ€ΡΡŽΡ‚ΡΡ Π² Π’ΠΎΠ»ΡŒΡ‚Π°Ρ… ΠΈ ΠΎΠΏΡ€Π΅Π΄Π΅Π»ΡΡŽΡ‚ ΠΎΡ‡Π΅Π½ΡŒ ΠΏΠΎΡ…ΠΎΠΆΠΈΠ΅ ΠΏΠΎ сути своСй процСссы.

    Π’ Π²ΠΈΠ΄Π΅ Ρ„ΠΎΡ€ΠΌΡƒΠ»Ρ‹ Π—Π°ΠΊΠΎΠ½ Ома для ΠΏΠΎΠ»Π½ΠΎΠΉ Ρ†Π΅ΠΏΠΈ Π±ΡƒΠ΄Π΅Ρ‚ Π²Ρ‹Π³Π»ΡΠ΄Π΅Ρ‚ΡŒ ΡΠ»Π΅Π΄ΡƒΡŽΡ‰ΠΈΠΌ ΠΎΠ±Ρ€Π°Π·ΠΎΠΌ:

    Π—Π°ΠΊΠΎΠ½ Ома для ΠΏΠΎΠ»Π½ΠΎΠΉ Ρ†Π΅ΠΏΠΈ

    I = Ξ΅/(R + r)

    I β€” сила Ρ‚ΠΎΠΊΠ° [A]

    Ξ΅ β€” Π­Π”Π‘ [Π’]

    R β€” сопротивлСниС [Ом]

    r β€” Π²Π½ΡƒΡ‚Ρ€Π΅Π½Π½Π΅Π΅ сопротивлСниС источника [Ом]

    Π›ΡŽΠ±ΠΎΠΉ источник Π½Π΅ ΠΈΠ΄Π΅Π°Π»Π΅Π½. Π’ Π·Π°Π΄Π°Ρ‡Π°Ρ… это Π²ΠΎΠ·ΠΌΠΎΠΆΠ½ΠΎ («источник ΡΡ‡ΠΈΡ‚Π°Ρ‚ΡŒ ΠΈΠ΄Π΅Π°Π»ΡŒΠ½Ρ‹ΠΌΒ», Π²ΠΎΡ‚ эти Π²ΠΎΡ‚ Ρ„Ρ€Π°Π·ΠΎΡ‡ΠΊΠΈ), Π½ΠΎ Π² Ρ€Π΅Π°Π»ΡŒΠ½ΠΎΠΉ ΠΆΠΈΠ·Π½ΠΈ β€” Ρ‚ΠΎΡ‡Π½ΠΎ Π½Π΅Ρ‚. Π’ связи с этим Ρƒ источника Π΅ΡΡ‚ΡŒ Π²Π½ΡƒΡ‚Ρ€Π΅Π½Π½Π΅Π΅ сопротивлСниС, ΠΊΠΎΡ‚ΠΎΡ€ΠΎΠ΅ ΠΌΠ΅ΡˆΠ°Π΅Ρ‚ ΠΏΡ€ΠΎΡ‚Π΅ΠΊΠ°Π½ΠΈΡŽ Ρ‚ΠΎΠΊΠ°.

    РСшим Π·Π°Π΄Π°Ρ‡Ρƒ Π½Π° ΠΏΠΎΠ»Π½ΡƒΡŽ Ρ†Π΅ΠΏΡŒ.

    Π—Π°Π΄Π°Ρ‡ΠΊΠ°

    Найти силу Ρ‚ΠΎΠΊΠ° Π² ΠΏΠΎΠ»Π½ΠΎΠΉ Ρ†Π΅ΠΏΠΈ, состоящСй ΠΈΠ· ΠΎΠ΄Π½ΠΎΠ³ΠΎ рСзистора сопротивлСниСм 3 Ом ΠΈ источником с Π­Π”Π‘ Ρ€Π°Π²Π½ΠΎΠΉ 4 Π’ ΠΈ Π²Π½ΡƒΡ‚Ρ€Π΅Π½Π½ΠΈΠΌ сопротивлСниСм 1 Ом

    РСшСниС:

    Π’ΠΎΠ·ΡŒΠΌΠ΅ΠΌ Π·Π°ΠΊΠΎΠ½ Ома для ΠΏΠΎΠ»Π½ΠΎΠΉ Ρ†Π΅ΠΏΠΈ:

    I = Ξ΅/(R + r)

    ΠŸΠΎΠ΄ΡΡ‚Π°Π²ΠΈΠΌ значСния:

    I = 4/(3+1) = 1 A

    ΠžΡ‚Π²Π΅Ρ‚: сила Ρ‚ΠΎΠΊΠ° Π² Ρ†Π΅ΠΏΠΈ Ρ€Π°Π²Π½Π° 1 А.

    Когда «сопротивлСниС бСсполСзно»

    ЭлСктричСский Ρ‚ΠΎΠΊ β€” ΡƒΠΌΠ½Ρ‹ΠΉ ΠΈ Ρ…ΠΈΡ‚Ρ€Ρ‹ΠΉ ΠΏΠ°Ρ€Π΅Π½ΡŒ. Если Ρƒ Π½Π΅Π³ΠΎ Π΅ΡΡ‚ΡŒ Π²ΠΎΠ·ΠΌΠΎΠΆΠ½ΠΎΡΡ‚ΡŒ ΠΎΠ±ΠΎΠΉΡ‚ΠΈ рСзистор ΠΈ ΠΏΠΎΠΉΡ‚ΠΈ ΠΏΠΎ ΠΈΠ΄Π΅Π°Π»ΡŒΠ½ΠΎΠΌΡƒ ΠΏΡ€ΠΎΠ²ΠΎΠ΄Π½ΠΈΠΊΡƒ Π±Π΅Π· сопротивлСния, ΠΎΠ½ это сдСлаСт. ΠŸΡ€ΠΈ этом с рСзисторами просто Ρ€Π°Π·Π½Ρ‹Ρ… Π½ΠΎΠΌΠΈΠ½Π°Π»ΠΎΠ² это Π½Π΅ сработаСт: ΠΎΠ½ Π½Π΅ ΠΏΠΎΠΉΠ΄Π΅Ρ‚ просто Ρ‡Π΅Ρ€Π΅Π· мСньшСС сопротивлСниС, Π° распрСдСлится согласно Π·Π°ΠΊΠΎΠ½Ρƒ Ома β€” большС Ρ‚ΠΎΠΊΠ° ΠΏΠΎΠΉΠ΄Π΅Ρ‚ Ρ‚ΡƒΠ΄Π°, Π³Π΄Π΅ сопротивлСниС мСньшС, ΠΈ Π½Π°ΠΎΠ±ΠΎΡ€ΠΎΡ‚.

    А Π²ΠΎΡ‚ Π½Π° рисункС Π½ΠΈΠΆΠ΅ сопротивлСниС Ρ†Π΅ΠΏΠΈ Ρ€Π°Π²Π½ΠΎ Π½ΡƒΠ»ΡŽ, ΠΏΠΎΡ‚ΠΎΠΌΡƒ Ρ‡Ρ‚ΠΎ Ρ‚ΠΎΠΊ Ρ‡Π΅Ρ€Π΅Π· рСзистор Π½Π΅ ΠΏΠΎΠΉΠ΄Π΅Ρ‚.


    Π’ΠΎΠΊ ΠΈΠ΄Π΅Ρ‚ ΠΏΠΎ ΠΏΡƒΡ‚ΠΈ наимСньшСго сопротивлСния.

    Π’Π΅ΠΏΠ΅Ρ€ΡŒ Π΄Π°Π²Π°ΠΉΡ‚Π΅ посмотрим Π½Π° Π·Π°ΠΊΠΎΠ½ Ома для участка Ρ†Π΅ΠΏΠΈ Π΅Ρ‰Π΅ Ρ€Π°Π·.

    Π—Π°ΠΊΠΎΠ½ Ома для участка Ρ†Π΅ΠΏΠΈ

    I = U/R

    I β€” сила Ρ‚ΠΎΠΊΠ° [A]

    U β€” напряТСниС [Π’]

    R β€” сопротивлСниС [Ом]

    ΠŸΠΎΠ΄ΡΡ‚Π°Π²ΠΈΠΌ сопротивлСниС, Ρ€Π°Π²Π½ΠΎΠ΅ 0. ΠŸΠΎΠ»ΡƒΡ‡Π°Π΅Ρ‚ΡΡ, Ρ‡Ρ‚ΠΎ Π·Π½Π°ΠΌΠ΅Π½Π°Ρ‚Π΅Π»ΡŒ Ρ€Π°Π²Π΅Π½ Π½ΡƒΠ»ΡŽ, Π° Π½Π° ΠΌΠ°Ρ‚Π΅ΠΌΠ°Ρ‚ΠΈΠΊΠ΅ говорят, Ρ‡Ρ‚ΠΎ Π½Π° ноль Π΄Π΅Π»ΠΈΡ‚ΡŒ нСльзя. Но ΠΌΡ‹ Π²Π°ΠΌ раскроСм ΡΡ‚Ρ€Π°ΡˆΠ½ΡƒΡŽ Ρ‚Π°ΠΉΠ½Ρƒ, Ρ‚ΠΎΠ»ΡŒΠΊΠΎ Π½Π΅ Π³ΠΎΠ²ΠΎΡ€ΠΈΡ‚Π΅ ΠΌΠ°Ρ‚Π΅ΠΌΠ°Ρ‚ΠΈΠΊΠ°ΠΌ: Π½Π° ноль Π΄Π΅Π»ΠΈΡ‚ΡŒ ΠΌΠΎΠΆΠ½ΠΎ. Если совсСм ΡƒΠΏΡ€ΠΎΡ‰Π°Ρ‚ΡŒ Ρ‚Π°ΠΊΠΎΠ΅ слоТноС вычислСниС (Π° ΠΈΠΌΠ΅Π½Π½ΠΎ ΠΏΠΎΡ‚ΠΎΠΌΡƒ Ρ‡Ρ‚ΠΎ ΠΎΠ½ΠΎ слоТноС, ΠΌΡ‹ всСгда Π³ΠΎΠ²ΠΎΡ€ΠΈΠΌ, Ρ‡Ρ‚ΠΎ Π΅Π³ΠΎ нСльзя ΠΏΡ€ΠΎΠΈΠ·Π²ΠΎΠ΄ΠΈΡ‚ΡŒ), Ρ‚ΠΎ получится Π±Π΅ΡΠΊΠΎΠ½Π΅Ρ‡Π½ΠΎΡΡ‚ΡŒ.

    Π’ΠΎ Π΅ΡΡ‚ΡŒ:

    I = U/0 = ∞

    Π’Π°ΠΊΠΎΠΉ случай Π½Π°Π·Ρ‹Π²Π°ΡŽΡ‚ ΠΊΠΎΡ€ΠΎΡ‚ΠΊΠΈΠΌ Π·Π°ΠΌΡ‹ΠΊΠ°Π½ΠΈΠ΅ΠΌ β€” ΠΊΠΎΠ³Π΄Π° Π²Π΅Π»ΠΈΡ‡ΠΈΠ½Π° силы Ρ‚ΠΎΠΊΠ° Π½Π°ΡΡ‚ΠΎΠ»ΡŒΠΊΠΎ Π²Π΅Π»ΠΈΠΊΠ°, Ρ‡Ρ‚ΠΎ ΠΌΠΎΠΆΠ½ΠΎ ΡƒΡΡ‚Ρ€Π΅ΠΌΠΈΡ‚ΡŒ Π΅Π΅ ΠΊ бСсконСчности. Π’ Ρ‚Π°ΠΊΠΈΡ… ситуациях ΠΌΡ‹ Π²ΠΈΠ΄ΠΈΠΌ искру, Π±ΡƒΡ€ΡŽ, Π±Π΅Π·ΡƒΠΌΠΈΠ΅ β€” ΠΈ всС ломаСтся.

    Π­Ρ‚ΠΎ происходит, ΠΏΠΎΡ‚ΠΎΠΌΡƒ Ρ‡Ρ‚ΠΎ Π΄Π²Π΅ Ρ‚ΠΎΡ‡ΠΊΠΈ Ρ†Π΅ΠΏΠΈ ΠΈΠΌΠ΅ΡŽΡ‚ ΠΌΠ΅ΠΆΠ΄Ρƒ собой напряТСниС (Ρ‚ΠΎ Π΅ΡΡ‚ΡŒ ΠΌΠ΅ΠΆΠ΄Ρƒ Π½ΠΈΠΌΠΈ Π΅ΡΡ‚ΡŒ Ρ€Π°Π·Π½ΠΈΡ†Π°). Π­Ρ‚ΠΎ ΠΊΠ°ΠΊ Ссли вдоль Ρ€Π΅ΠΊΠΈ Π½Π΅ΠΎΠΆΠΈΠ΄Π°Π½Π½ΠΎ появляСтся Π²ΠΎΠ΄ΠΎΠΏΠ°Π΄. Из-Π·Π° этой Ρ€Π°Π·Π½ΠΈΡ†Ρ‹ Π²ΠΎΠ·Π½ΠΈΠΊΠ°Π΅Ρ‚ искра, ΠΊΠΎΡ‚ΠΎΡ€ΡƒΡŽ ΠΌΠΎΠΆΠ½ΠΎ ΠΈΠ·Π±Π΅ΠΆΠ°Ρ‚ΡŒ, поставив Π² Ρ†Π΅ΠΏΡŒ рСзистор.

    ИмСнно Π²ΠΎ ΠΈΠ·Π±Π΅ΠΆΠ°Π½ΠΈΠ΅ ΠΊΠΎΡ€ΠΎΡ‚ΠΊΠΈΡ… Π·Π°ΠΌΡ‹ΠΊΠ°Π½ΠΈΠΉ Π½ΡƒΠΆΠ½ΠΎ Π΄ΠΎΠΏΠΎΠ»Π½ΠΈΡ‚Π΅Π»ΡŒΠ½ΠΎΠ΅ сопротивлСниС Π² Ρ†Π΅ΠΏΠΈ.

    ΠŸΠ°Ρ€Π°Π»Π»Π΅Π»ΡŒΠ½ΠΎΠ΅ ΠΈ ΠΏΠΎΡΠ»Π΅Π΄ΠΎΠ²Π°Ρ‚Π΅Π»ΡŒΠ½ΠΎΠ΅ соСдинСниС

    ВсС это врСмя Ρ€Π΅Ρ‡ΡŒ шла ΠΎ цСпях с ΠΎΠ΄Π½ΠΈΠΌ рСзистором. Рассмотрим, Ρ‡Ρ‚ΠΎ происходит, Ссли ΠΈΡ… большС.


    ΠŸΠΎΡΠ»Π΅Π΄ΠΎΠ²Π°Ρ‚Π΅Π»ΡŒΠ½ΠΎΠ΅ соСдинСниС

    ΠŸΠ°Ρ€Π°Π»Π»Π΅Π»ΡŒΠ½ΠΎΠ΅ соСдинСниС

    Π‘Ρ…Π΅ΠΌΠ°

    РСзисторы ΡΠ»Π΅Π΄ΡƒΡŽΡ‚ Π΄Ρ€ΡƒΠ³ Π·Π° Π΄Ρ€ΡƒΠ³ΠΎΠΌ

    ΠœΠ΅ΠΆΠ΄Ρƒ рСзисторами Π΅ΡΡ‚ΡŒ Π΄Π²Π° ΡƒΠ·Π»Π°

    Π£Π·Π΅Π» β€” это соСдинСниС Ρ‚Ρ€Π΅Ρ… ΠΈ Π±ΠΎΠ»Π΅Π΅ ΠΏΡ€ΠΎΠ²ΠΎΠ΄Π½ΠΈΠΊΠΎΠ²

    Π‘ΠΈΠ»Π° Ρ‚ΠΎΠΊΠ°

    Π‘ΠΈΠ»Π° Ρ‚ΠΎΠΊΠ° ΠΎΠ΄ΠΈΠ½Π°ΠΊΠΎΠ²Π° Π½Π° всСх рСзисторах

    I = I1 = I2

    Π‘ΠΈΠ»Π° Ρ‚ΠΎΠΊΠ°, входящСго Π² ΡƒΠ·Π΅Π», Ρ€Π°Π²Π½Π° суммС сил Ρ‚ΠΎΠΊΠΎΠ², выходящих ΠΈΠ· Π½Π΅Π³ΠΎ

    I = I1 + I2

    НапряТСниС

    ΠžΠ±Ρ‰Π΅Π΅ напряТСниС Ρ†Π΅ΠΏΠΈ складываСтся ΠΈΠ· напряТСний Π½Π° ΠΊΠ°ΠΆΠ΄ΠΎΠΌ рСзисторС

    U = U1 + U2

    НапряТСниС ΠΎΠ΄ΠΈΠ½Π°ΠΊΠΎΠ²ΠΎ Π½Π° всСх рСзисторах

    U = U1 = U2

    Π‘ΠΎΠΏΡ€ΠΎΡ‚ΠΈΠ²Π»Π΅Π½ΠΈΠ΅

    ΠžΠ±Ρ‰Π΅Π΅ сопротивлСниС Ρ†Π΅ΠΏΠΈ складываСтся ΠΈΠ· сопротивлСний ΠΊΠ°ΠΆΠ΄ΠΎΠ³ΠΎ рСзистора

    R = R1 + R2

    ΠžΠ±Ρ‰Π΅Π΅ сопротивлСниС для бСсконСчного количСства ΠΏΠ°Ρ€Π°Π»Π»Π΅Π»ΡŒΠ½ΠΎ соСдинСнных рСзисторов

    1/R = 1/R1 + 1/R2 + … + 1/Rn

    ΠžΠ±Ρ‰Π΅Π΅ сопротивлСниС для Π΄Π²ΡƒΡ… ΠΏΠ°Ρ€Π°Π»Π»Π΅Π»ΡŒΠ½ΠΎ соСдинСнных рСзисторов

    R = (R1 * R2)/R1 + R2

    ΠžΠ±Ρ‰Π΅Π΅ сопротивлСниС бСсконСчного количСства ΠΏΠ°Ρ€Π°Π»Π»Π΅Π»ΡŒΠ½ΠΎ соСдинСнных ΠΎΠ΄ΠΈΠ½Π°ΠΊΠΎΠ²Ρ‹Ρ… рСзисторов

    R = R1/n

    Π—Π°Ρ‡Π΅ΠΌ Π½ΡƒΠΆΠ½Ρ‹ эти соСдинСния, Ссли ΠΌΠΎΠΆΠ½ΠΎ сразу Π²Π·ΡΡ‚ΡŒ рСзистор Π½ΡƒΠΆΠ½ΠΎΠ³ΠΎ Π½ΠΎΠΌΠΈΠ½Π°Π»Π°?

    НачнСм с Ρ‚ΠΎΠ³ΠΎ, Ρ‡Ρ‚ΠΎ всС элСктронныС ΠΊΠΎΠΌΠΏΠΎΠ½Π΅Π½Ρ‚Ρ‹ ΠΈΠ·Π³ΠΎΡ‚Π°Π²Π»ΠΈΠ²Π°ΡŽΡ‚ΡΡ ΠΏΠΎ Π“ΠžΠ‘Π’Ρƒ. Π’ΠΎ Π΅ΡΡ‚ΡŒ Π΅ΡΡ‚ΡŒ ΠΎΠΏΡ€Π΅Π΄Π΅Π»Π΅Π½Π½Ρ‹Π΅ значСния рСзисторов, ΠΎΡ‚ ΠΊΠΎΡ‚ΠΎΡ€Ρ‹Ρ… нСльзя ΠΎΡ‚ΠΎΠΉΡ‚ΠΈ ΠΏΡ€ΠΈ производствС. Π­Ρ‚ΠΎ Π·Π½Π°Ρ‡ΠΈΡ‚, Ρ‡Ρ‚ΠΎ Π½Π΅ всСгда Π΅ΡΡ‚ΡŒ рСзистор Π½ΡƒΠΆΠ½ΠΎΠ³ΠΎ Π½ΠΎΠΌΠΈΠ½Π°Π»Π° ΠΈ Π΅Π³ΠΎ Π½ΡƒΠΆΠ½ΠΎ ΡΠΎΠΎΡ€ΡƒΠ΄ΠΈΡ‚ΡŒ ΠΈΠ· Π΄Ρ€ΡƒΠ³ΠΈΡ… рСзисторов.

    ΠŸΠ°Ρ€Π°Π»Π»Π΅Π»ΡŒΠ½ΠΎΠ΅ соСдинСниС Ρ‚Π°ΠΊΠΆΠ΅ ΠΈΡΠΏΠΎΠ»ΡŒΠ·ΡƒΡŽΡ‚, ΠΊΠ°ΠΊ «запасной аэродром»: ΠΊΠΎΠ³Π΄Π° Π½Π° ΠΊΠΎΠ½Π΅Ρ‡Π½Ρ‹ΠΉ Ρ€Π΅Π·ΡƒΠ»ΡŒΡ‚Π°Ρ‚ ΠΎΠ±Ρ‰Π΅Π΅ сопротивлСниС сильно Π½Π΅ повлияСт, Π½ΠΎ Π² случаС ΠΎΡ‚ΠΊΠ°Π·Π° ΠΎΠ΄Π½ΠΎΠ³ΠΎ ΠΈΠ· рСзисторов, Π±ΡƒΠ΄Π΅Ρ‚ Ρ€Π°Π±ΠΎΡ‚Π°Ρ‚ΡŒ Π΄Ρ€ΡƒΠ³ΠΎΠΉ.

    ΠŸΡ€ΠΈΠ·Π½Π°Π΅ΠΌΡΡ чСстно: схСмы, ΠΊΠΎΡ‚ΠΎΡ€Ρ‹Π΅ ΠΎΠ±Ρ‹Ρ‡Π½ΠΎ Π΄Π°ΡŽΡ‚ Π² Π·Π°Π΄Π°Ρ‡Π°Ρ… (ΠΌΠΈΠ»Π»ΠΈΠΎΠ½ ΠΏΠ°Ρ€Π°Π»Π»Π΅Π»ΡŒΠ½ΠΎ соСдинСнных рСзисторов, ΠΊ Π½ΠΈΠΌ Π΅Ρ‰Π΅ ΠΏΠΎΡΠ»Π΅Π΄ΠΎΠ²Π°Ρ‚Π΅Π»ΡŒΠ½Ρ‹ΠΉ, Π° ΠΊ этому ΠΏΠΎΡΠ»Π΅Π΄ΠΎΠ²Π°Ρ‚Π΅Π»ΡŒΠ½ΠΎΠΌΡƒ Π΅Ρ‰Π΅ ΠΌΠΈΠ»Π»ΠΈΠΎΠ½ ΠΏΠ°Ρ€Π°Π»Π»Π΅Π»ΡŒΠ½Ρ‹Ρ…) β€” Π² ΠΆΠΈΠ·Π½ΠΈ Π½Π΅ Π²ΡΡ‚Ρ€Π΅Ρ‡Π°ΡŽΡ‚ΡΡ. Но Π½Π°Π²Ρ‹ΠΊ расчСта Ρ‚Π°ΠΊΠΈΡ… схСм впослСдствии ΡƒΠΏΡ€ΠΎΡ‰Π°Π΅Ρ‚ подсчСт схСм Ρ€Π΅Π°Π»ΡŒΠ½Ρ‹Ρ…, ΠΏΠΎΡ‚ΠΎΠΌΡƒ Ρ‡Ρ‚ΠΎ Ρ‚Π°ΠΊ Π²Ρ‹ Π½Π΅Π²ΠΎΠΎΡ€ΡƒΠΆΠ΅Π½Π½Ρ‹ΠΌ Π³Π»Π°Π·ΠΎΠΌ ΠΎΡ‚Π»ΠΈΡ‡Π°Π΅Ρ‚Π΅ ΠΏΠΎΡΠ»Π΅Π΄ΠΎΠ²Π°Ρ‚Π΅Π»ΡŒΠ½ΠΎΠ΅ соСдинСниС ΠΎΡ‚ ΠΏΠ°Ρ€Π°Π»Π»Π΅Π»ΡŒΠ½ΠΎΠ³ΠΎ.

    РСшим нСсколько Π·Π°Π΄Π°Ρ‡ Π½Π° ΠΏΠΎΡΠ»Π΅Π΄ΠΎΠ²Π°Ρ‚Π΅Π»ΡŒΠ½ΠΎΠ΅ ΠΈ ΠΏΠ°Ρ€Π°Π»Π»Π΅Π»ΡŒΠ½ΠΎΠ΅ соСдинСниС.

    Π—Π°Π΄Π°Ρ‡ΠΊΠ° Ρ€Π°Π·

    Найти ΠΎΠ±Ρ‰Π΅Π΅ сопротивлСниС Ρ†Π΅ΠΏΠΈ.

    R1 = 1 Ом, R2 = 2 Ом, R3 = 3 Ом, R4 = 4 Ом.


    РСшСниС:

    ΠžΠ±Ρ‰Π΅Π΅ сопротивлСниС ΠΏΡ€ΠΈ ΠΏΠΎΡΠ»Π΅Π΄ΠΎΠ²Π°Ρ‚Π΅Π»ΡŒΠ½ΠΎΠΌ соСдинСнии рассчитываСтся ΠΏΠΎ Ρ„ΠΎΡ€ΠΌΡƒΠ»Π΅:

    R = R1 + R2 + R3 + R4 = 1 + 2 + 3 + 4 = 10 Ом

    ΠžΡ‚Π²Π΅Ρ‚: ΠΎΠ±Ρ‰Π΅Π΅ сопротивлСниС Ρ†Π΅ΠΏΠΈ Ρ€Π°Π²Π½ΠΎ 10 Ом

    Π—Π°Π΄Π°Ρ‡ΠΊΠ° Π΄Π²Π°

    Найти ΠΎΠ±Ρ‰Π΅Π΅ сопротивлСниС Ρ†Π΅ΠΏΠΈ.

    R1 = 4 Ом, R2 = 2 Ом


    РСшСниС:

    ΠžΠ±Ρ‰Π΅Π΅ сопротивлСниС ΠΏΡ€ΠΈ ΠΏΠ°Ρ€Π°Π»Π»Π΅Π»ΡŒΠ½ΠΎΠΌ соСдинСнии рассчитываСтся ΠΏΠΎ Ρ„ΠΎΡ€ΠΌΡƒΠ»Π΅:

    R = (R1 * R2)/R1 + R2 = 4*2/4+2 = 4/3 = 1 β…“ Ом

    ΠžΡ‚Π²Π΅Ρ‚: ΠΎΠ±Ρ‰Π΅Π΅ сопротивлСниС Ρ†Π΅ΠΏΠΈ Ρ€Π°Π²Π½ΠΎ 1 β…“ Ом

    Π—Π°Π΄Π°Ρ‡ΠΊΠ° Ρ‚Ρ€ΠΈ

    Найти ΠΎΠ±Ρ‰Π΅Π΅ сопротивлСниС Ρ†Π΅ΠΏΠΈ, состоящСй ΠΈΠ· рСзистора ΠΈ Π΄Π²ΡƒΡ… Π»Π°ΠΌΠΏ.

    R1 = 1 Ом, R2 = 2 Ом, R3 = 3 Ом


    РСшСниС:

    Π‘Π½Π°Ρ‡Π°Π»Π° ΠΎΠ±ΠΎΠ·Π½Π°Ρ‡ΠΈΠΌ, Ρ‡Ρ‚ΠΎ Π»Π°ΠΌΠΏΡ‹ с Ρ‚ΠΎΡ‡ΠΊΠΈ зрСния элСмСнта элСктричСской Ρ†Π΅ΠΏΠΈ Π½Π΅ ΠΎΡ‚Π»ΠΈΡ‡Π°ΡŽΡ‚ΡΡ ΠΎΡ‚ рСзисторов. Π’ΠΎ Π΅ΡΡ‚ΡŒ Ρƒ Π½ΠΈΡ… Ρ‚ΠΎΠΆΠ΅ Π΅ΡΡ‚ΡŒ сопротивлСниС, ΠΈ ΠΎΠ½ΠΈ Ρ‚Π°ΠΊΠΆΠ΅ Π²Π»ΠΈΡΡŽΡ‚ Π½Π° Ρ†Π΅ΠΏΡŒ.

    Π’ Π΄Π°Π½Π½ΠΎΠΌ случаС соСдинСниС являСтся ΡΠΌΠ΅ΡˆΠ°Π½Π½Ρ‹ΠΌ. Π›Π°ΠΌΠΏΡ‹ соСдСнСны ΠΏΠ°Ρ€Π°Π»Π»Π΅Π»ΡŒΠ½ΠΎ, Π° ΠΏΠΎΡΠ»Π΅Π΄ΠΎΠ²Π°Ρ‚Π΅Π»ΡŒΠ½ΠΎ ΠΊ Π½ΠΈΠΌ ΠΏΠΎΠ΄ΠΊΠ»ΡŽΡ‡Π΅Π½ рСзистор.

    Π‘Π½Π°Ρ‡Π°Π»Π° посчитаСм ΠΎΠ±Ρ‰Π΅Π΅ сопротивлСниС для Π»Π°ΠΌΠΏ. ΠžΠ±Ρ‰Π΅Π΅ сопротивлСниС ΠΏΡ€ΠΈ ΠΏΠ°Ρ€Π°Π»Π»Π΅Π»ΡŒΠ½ΠΎΠΌ соСдинСнии рассчитываСтся ΠΏΠΎ Ρ„ΠΎΡ€ΠΌΡƒΠ»Π΅:

    Rламп = (R2 * R3)/R2 + R3 = 2*3/2+3 = 6/5 = 1,2 Ом

    ΠžΠ±Ρ‰Π΅Π΅ сопротивлСниС ΠΏΡ€ΠΈ ΠΏΠΎΡΠ»Π΅Π΄ΠΎΠ²Π°Ρ‚Π΅Π»ΡŒΠ½ΠΎΠΌ соСдинСнии рассчитываСтся ΠΏΠΎ Ρ„ΠΎΡ€ΠΌΡƒΠ»Π΅:

    R = R1 + Rламп = 1 + 1,2 = 2,2 Ом

    ΠžΡ‚Π²Π΅Ρ‚: ΠΎΠ±Ρ‰Π΅Π΅ сопротивлСниС Ρ†Π΅ΠΏΠΈ Ρ€Π°Π²Π½ΠΎ 2,2 Ом.

    НаконСц-Ρ‚ΠΎ, послСдняя ΠΈ самая слоТная Π·Π°Π΄Π°Ρ‡Π°! Π’ Π½Π΅ΠΉ собрали всС самоС ΡΠ΅Ρ€ΡŒΠ΅Π·Π½ΠΎΠ΅ ΠΈΠ· этой ΡΡ‚Π°Ρ‚ΡŒΠΈ πŸ’ͺ.

    Π—Π°Π΄Π°Ρ‡ΠΊΠ° Ρ‡Π΅Ρ‚Ρ‹Ρ€Π΅ со Π·Π²Π΅Π·Π΄ΠΎΡ‡ΠΊΠΎΠΉ

    К аккумулятору с Π­Π”Π‘ 12 Π’, ΠΏΠΎΠ΄ΠΊΠ»ΡŽΡ‡Π΅Π½Π° Π»Π°ΠΌΠΏΠΎΡ‡ΠΊΠ° ΠΈ Π΄Π²Π° ΠΏΠ°Ρ€Π°Π»Π»Π΅Π»ΡŒΠ½ΠΎ соСдинСнных рСзистора сопротивлСниСм ΠΊΠ°ΠΆΠ΄Ρ‹ΠΉ ΠΏΠΎ 10 Ом. Π˜Π·Π²Π΅ΡΡ‚Π½ΠΎ, Ρ‡Ρ‚ΠΎ Ρ‚ΠΎΠΊ Π² Ρ†Π΅ΠΏΠΈ 0,5 А, Π° сопротивлСниС Π»Π°ΠΌΠΏΠΎΡ‡ΠΊΠΈ R/2.2)/2R = R/2 = 10/2 = 5 Ом

    И ΠΎΠ±Ρ‰Π΅Π΅ сопротивлСниС Ρ†Π΅ΠΏΠΈ Ρ€Π°Π²Π½ΠΎ:

    R = RΠ»Π°ΠΌΠΏΡ‹ + RрСзисторов = 5 + 5 = 10 Ом

    Π’Ρ‹Ρ€Π°Π·ΠΈΠΌ Π²Π½ΡƒΡ‚Ρ€Π΅Π½Π½Π΅Π΅ сопротивлСниС источника ΠΈΠ· Π·Π°ΠΊΠΎΠ½Π° Ома для ΠΏΠΎΠ»Π½ΠΎΠΉ Ρ†Π΅ΠΏΠΈ.

    I = Ξ΅/(R + r)

    R + r = Ξ΅/I

    r = Ξ΅/I β€” R

    ΠŸΠΎΠ΄ΡΡ‚Π°Π²ΠΈΠΌ значСния:

    r = 12/0,5 β€” 10 = 14 Ом

    ΠžΡ‚Π²Π΅Ρ‚: Π²Π½ΡƒΡ‚Ρ€Π΅Π½Π½Π΅Π΅ сопротивлСниС источника Ρ€Π°Π²Π½ΠΎ 14 Ом.

    Π§Ρ‚ΠΎΠ±Ρ‹ Ρ€Π΅Π±Π΅Π½ΠΎΠΊ научился Ρ€Π΅ΡˆΠ°Ρ‚ΡŒ самыС слоТныС Π·Π°Π΄Π°Ρ‡ΠΈ ΠΈ чувствовал сСбя ΡƒΠ²Π΅Ρ€Π΅Π½Π½ΠΎ Π½Π° ΠΎΠ»ΠΈΠΌΠΏΠΈΠ°Π΄Π°Ρ… ΠΈ экзамСнах, Π·Π°ΠΏΠΈΡˆΠΈΡ‚Π΅ Π΅Π³ΠΎ Π½Π° бСсплатный Π²Π²ΠΎΠ΄Π½Ρ‹ΠΉ ΡƒΡ€ΠΎΠΊ Π² Skysmart.

    ΠŸΡ€ΠΎΡ„Π΅ΡΡΠΈΠΎΠ½Π°Π»ΡŒΠ½Ρ‹Π΅ учитСля Ρ„ΠΈΠ·ΠΈΠΊΠΈ Π½Π΅ Ρ‚ΠΎΠ»ΡŒΠΊΠΎ Π½Π°ΡƒΡ‡Π°Ρ‚ Ρ€Π΅ΡˆΠ°Ρ‚ΡŒ Π·Π°Π΄Π°Ρ‡ΠΈ ΠΈ подготовят ΠΊ экзамСну, Π½ΠΎ ΠΈ ΠΎΠ±ΡŠΡΡΠ½ΡΡ‚, ΠΊΠ°ΠΊ это всС устроСно: Π»Π΅Π³ΠΊΠΎ, ΠΈΠ½Ρ‚Π΅Ρ€Π°ΠΊΡ‚ΠΈΠ²Π½ΠΎ ΠΈ с ΠΏΡ€ΠΈΠΌΠ΅Ρ€Π°ΠΌΠΈ ΠΈΠ· Ρ€Π΅Π°Π»ΡŒΠ½ΠΎΠΉ ΠΆΠΈΠ·Π½ΠΈ соврСмСнных подростков.

    Π—Π°ΠΊΠΎΠ½ ΠΎΠΌΠ° для ΠΏΠΎΠ»Π½ΠΎΠΉ Ρ†Π΅ΠΏΠΈ

      1. Π—Π°ΠΊΠΎΠ½ Ома для ΠΏΠΎΠ»Π½ΠΎΠΉ Ρ†Π΅ΠΏΠΈ

    2.3.1. ЭлСктричСская Ρ†Π΅ΠΏΡŒ состоит ΠΈΠ· источника Ρ‚ΠΎΠΊΠ° ΠΈ Π΄Π²ΡƒΡ… сопротивлСний, ΠΎΠ΄Π½ΠΎ ΠΈΠ· ΠΊΠΎΡ‚ΠΎΡ€Ρ‹Ρ… ΠΌΠΎΠΆΠ΅Ρ‚ Ρ‡Π΅Ρ€Π΅Π· ΠΊΠ»ΡŽΡ‡ ΡΠΎΠ΅Π΄ΠΈΠ½ΡΡ‚ΡŒΡΡ ΠΏΠ°Ρ€Π°Π»Π»Π΅Π»ΡŒΠ½ΠΎ со Π²Ρ‚ΠΎΡ€Ρ‹ΠΌ сопротивлСниСм. Π‘ΠΎΠΏΡ€ΠΎΡ‚ΠΈΠ²Π»Π΅Π½ΠΈΠ΅ ΠΏ R1 Π²Π΄Π²ΠΎΠ΅ большС сопротивлСния R2. Π’Π½ΡƒΡ‚Ρ€Π΅Π½Π½Π΅Π΅ сопротивлСниС источника Ρ‚ΠΎΠΊΠ° r = 0,1 R1. ΠžΠΏΡ€Π΅Π΄Π΅Π»ΠΈΡ‚ΡŒ, Π²ΠΎ сколько Ρ€Π°Π· измСнятся показания Π°ΠΌΠΏΠ΅Ρ€ΠΌΠ΅Ρ‚Ρ€Π° ΠΈ напряТСниС Π½Π° ΠΊΠ»Π΅ΠΌΠΌΠ°Ρ… источника ΠΏΡ€ΠΈ Π·Π°ΠΌΡ‹ΠΊΠ°Π½ΠΈΠΈ ΠΊΠ»ΡŽΡ‡Π° К?

    РСшСниС

    1. ΠŸΡ€ΠΈ Ρ€Π°Π·ΠΎΠΌΠΊΠ½ΡƒΡ‚ΠΎΠΌ ΠΊΠ»ΡŽΡ‡Π΅ К Π·Π°ΠΊΠΎΠ½ Ома для ΠΏΠΎΠ»Π½ΠΎΠΉ Ρ†Π΅ΠΏΠΈ записываСтся ΡΠ»Π΅Π΄ΡƒΡŽΡ‰ΠΈΠΌ ΠΎΠ±Ρ€Π°Π·ΠΎΠΌ

    . (1)

    2. ΠŸΡ€ΠΈ Π·Π°ΠΌΡ‹ΠΊΠ°Π½ΠΈΠΈ ΠΊΠ»ΡŽΡ‡Π° сопротивлСниС Π½Π°Π³Ρ€ΡƒΠ·ΠΊΠΈ измСнится

    . (2)

    3. Π—Π°ΠΊΠΎΠ½ Ома Π² этом случаС ΠΏΡ€ΠΈΠΌΠ΅Ρ‚ Π²ΠΈΠ΄

    . (3)

    4. ΠžΡ‚Π½ΠΎΡˆΠ΅Π½ΠΈΠ΅ Ρ‚ΠΎΠΊΠΎΠ² опрСдСлится ΠΊΠ°ΠΊ

    . (4)

    5. ПадСниС напряТСния Π½Π° ΠΊΠ»Π΅ΠΌΠΌΠ°Ρ… источника ΠΏΡ€ΠΈ Ρ€Π°Π·ΠΎΠΌΠΊΠ½ΡƒΡ‚ΠΎΠΌ ΠΊΠ»ΡŽΡ‡Π΅

    . (5)

    6. ПадСниС напряТСния послС замыкания ΠΊΠ»ΡŽΡ‡Π°

    . (6)

    7. ΠžΡ‚Π½ΠΎΡˆΠ΅Π½ΠΈΠ΅ напряТСний Π½Π° ΠΊΠ»Π΅ΠΌΠΌΠ°Ρ… источника

    . (7)

    2.3.2. БатарСя замкнутая Π½Π° сопротивлСниСR1 = 10 Ом, Π΄Π°Ρ‘Ρ‚ Ρ‚ΠΎΠΊ силой I1 = 3 А; замкнутая Π½Π° сопротивлСниСR2 = 20 Ом, ΠΎΠ½Π° Π΄Π°Ρ‘Ρ‚ Ρ‚ΠΎΠΊ силой I2 = 1,6 А. ΠžΠΏΡ€Π΅Π΄Π΅Π»ΠΈΡ‚Π΅ Π­Π”Π‘ источника ο₯ ΠΈ Π΅Ρ‘ Π²Π½ΡƒΡ‚Ρ€Π΅Π½Π½Π΅Π΅ сопротивлСниС r.

    РСшСниС

    1. Π—Π°ΠΏΠΈΡˆΠ΅ΠΌ Π΄Π²Π°ΠΆΠ΄Ρ‹ ΡƒΡ€Π°Π²Π½Π΅Π½ΠΈΠ΅ Π·Π°ΠΊΠΎΠ½Π° Ома для ΠΏΠΎΠ»Π½ΠΎΠΉ Ρ†Π΅ΠΏΠΈ

    (1)

    2. Π’Ρ‹Ρ€Π°Π·ΠΈΠΌ ΠΈΠ· ΠΏΠ΅Ρ€Π²ΠΎΠ³ΠΎ уравнСния систСмы (1) Π²Π΅Π»ΠΈΡ‡ΠΈΠ½Ρƒ ο₯ ΠΈ подставим Π²ΠΎ Π²Ρ‚ΠΎΡ€ΠΎΠ΅ ΡƒΡ€Π°Π²Π½Π΅Π½ΠΈΠ΅

    , (2)

    3. Π Π°Π·Ρ€Π΅ΡˆΠΈΠΌ ΠΏΠΎΠ»ΡƒΡ‡Π΅Π½Π½ΠΎΠ΅ ΡƒΡ€Π°Π²Π½Π΅Π½ΠΈΠ΅ ΠΎΡ‚Π½ΠΎΡΠΈΡ‚Π΅Π»ΡŒΠ½ΠΎ Π²Π½ΡƒΡ‚Ρ€Π΅Π½Π½Π΅Π³ΠΎ сопротивлСния источника r

    . (3)

    4. Π—Π½Π°Ρ‡Π΅Π½ΠΈΠ΅ Π²Π΅Π»ΠΈΡ‡ΠΈΠ½Ρ‹ ο₯ ΠΌΠΎΠΆΠ½ΠΎ ΠΏΠΎΠ»ΡƒΡ‡ΠΈΡ‚ΡŒ ΠΈΠ· любого уравнСния систСмы (1) ΠΏΡ€ΠΈ подстановки Π² Π½Π΅Π³ΠΎ r ΠΈΠ· уравнСния (3)

    .

    2.3.3. Π‘Π°Ρ‚Π°Ρ€Π΅ΠΈ с Π­Π”Π‘ ο₯1 = 20 Π’, ο₯2 = 30 Π’ ΠΈ Π²Π½ΡƒΡ‚Ρ€Π΅Π½Π½ΠΈΠΌΠΈ сопротивлСниями соотвСтствСнно r1 = 4 Ом, r2 = 6 Ом соСдинСны ΠΏΠ°Ρ€Π°Π»Π»Π΅Π»ΡŒΠ½ΠΎ ΠΈ согласно. ΠšΠ°ΠΊΠΎΠ²Ρ‹ Π΄ΠΎΠ»ΠΆΠ½Ρ‹ Π±Ρ‹Ρ‚ΡŒ ΠΏΠ°Ρ€Π°ΠΌΠ΅Ρ‚Ρ€Ρ‹ ο₯ ΠΈ r эквивалСнтного источника, ΠΊΠΎΡ‚ΠΎΡ€Ρ‹ΠΌ ΠΌΠΎΠΆΠ½ΠΎ Π·Π°ΠΌΠ΅Π½ΠΈΡ‚ΡŒ соСдинСниС?

    РСшСниС

    1. ΠžΠΏΡ€Π΅Π΄Π΅Π»ΠΈΠΌ силу Ρ‚ΠΎΠΊΠ°, ΠΏΡ€ΠΎΡ‚Π΅ΠΊΠ°ΡŽΡ‰Π΅Π³ΠΎ Ρ‡Π΅Ρ€Π΅Π· источники ΠΏΡ€ΠΈ ΠΈΡ… совмСстном Π²ΠΊΠ»ΡŽΡ‡Π΅Π½ΠΈΠΈ

    . (1)

    2. Π‘ΠΈΠ»Π° Ρ‚ΠΎΠΊΠ°, ΠΊΠΎΡ‚ΠΎΡ€Ρ‹ΠΉ ΠΌΠΎΠΆΠ΅Ρ‚ Π±Ρ‹Ρ‚ΡŒ ΠΏΠΎΠ»ΡƒΡ‡Π΅Π½ ΠΎΡ‚ Π΄Π²ΡƒΡ… источников ΠΏΡ€ΠΈ ΠΈΡ… совмСстной Ρ€Π°Π±ΠΎΡ‚Π΅ I0 = I1 + I2 = 5 A

    3. ΠžΠ±Ρ‰Π΅Π΅ Π²Π½ΡƒΡ‚Ρ€Π΅Π½Π½Π΅Π΅ сопротивлСниС

    . (2)

    4. ΠžΠΏΡ€Π΅Π΄Π΅Π»ΠΈΠΌ Π΄Π°Π»Π΅Π΅ ΡΠΊΠ²ΠΈΠ²Π°Π»Π΅Π½Ρ‚Π½ΡƒΡŽ Π­Π”Π‘

    . (3)

    Π’Π°ΠΊΠΈΠΌ ΠΎΠ±Ρ€Π°Π·ΠΎΠΌ, эквивалСнтный источник Π΄ΠΎΠ»ΠΆΠ΅Π½ ΠΈΠΌΠ΅Ρ‚ΡŒ Π­Π”Π‘ ο₯ = 12 Π’ ΠΈ Π²Π½ΡƒΡ‚Ρ€Π΅Π½Π½Π΅Π΅ сопротивлСниС r = 2,4 Ом.

    2.3.4. Π”Π²Π΅ Π±Π°Ρ‚Π°Ρ€Π΅ΠΈ с ΠΎΠ΄ΠΈΠ½Π°ΠΊΠΎΠ²Ρ‹ΠΌ Π²Π½ΡƒΡ‚Ρ€Π΅Π½Π½ΠΈΠΌ сопротивлСниСм соСдинСны Ρ‚Π°ΠΊ, Ρ‡Ρ‚ΠΎ Π­Π”Π‘ ΠΎΠ±Ρ€Π°Π·ΠΎΠ²Π°Π²ΡˆΠ΅Π³ΠΎΡΡ источника напряТСния Ρ€Π°Π²Π½Π° ο₯. Π­Π”Π‘ ΠΎΠ΄Π½ΠΎΠΉ ΠΈΠ· Π±Π°Ρ‚Π°Ρ€Π΅ΠΉ 3/2ο₯. НарисуйтС всС Π²ΠΎΠ·ΠΌΠΎΠΆΠ½Ρ‹Π΅ схСмы соСдинСний. Для ΠΊΠ°ΠΆΠ΄ΠΎΠ³ΠΎ Π²Π°Ρ€ΠΈΠ°Π½Ρ‚Π° соСдинСний ΠΎΠΏΡ€Π΅Π΄Π΅Π»ΠΈΡ‚Π΅ Π­Π”Π‘ Π²Ρ‚ΠΎΡ€ΠΎΠΉ Π±Π°Ρ‚Π°Ρ€Π΅ΠΈ.

    РСшСниС

    1. Один ΠΈΠ· Π²Π°Ρ€ΠΈΠ°Π½Ρ‚ΠΎΠ² Π²ΠΊΠ»ΡŽΡ‡Π΅Π½ΠΈΠ΅ источников ΠΏΠΎΡΠ»Π΅Π΄ΠΎΠ²Π°Ρ‚Π΅Π»ΡŒΠ½ΠΎ ΠΈ встрСчно, ΠΊΠΎΠ³Π΄Π° Π­Π”Π‘ Π²Ρ‚ΠΎΡ€ΠΎΠ³ΠΎ источника Ρ€Π°Π²Π½Π° ο₯2 = 0,5ο₯, Π° ο₯1 = ο₯. Π’ этом случаС общая Π­Π”Π‘ ο₯ опрСдСлится ΠΊΠ°ΠΊ . Π’Π½ΡƒΡ‚Ρ€Π΅Π½Π½Π΅ сопротивлСниС Ρ‚Π°ΠΊΠΎΠ³ΠΎ Π²ΠΊΠ»ΡŽΡ‡Π΅Π½ΠΈΡ источников Π±ΡƒΠ΄Π΅Ρ‚ Ρ€Π°Π²Π½ΠΎ 2r.

    2. Π’ΠΎΠ·ΠΌΠΎΠΆΠ½ΠΎ ΠΈ ΠΏΠ°Ρ€Π°Π»Π»Π΅Π»ΡŒΠ½ΠΎΠ΅ согласноС Π²ΠΊΠ»ΡŽΡ‡Π΅Π½ΠΈΠ΅ источников, ΠΎΠ±Ρ‰Π΅Π΅ сопротивлСниС ΠΊΠΎΡ‚ΠΎΡ€Ρ‹Ρ… Π±ΡƒΠ΄Π΅Ρ‚ Ρ€Π°Π²Π½ΠΎ r/2. ПадСниС напряТСния Π½Π° источниках Π±ΡƒΠ΄Π΅Ρ‚ ΠΎΠ΄ΠΈΠ½Π°ΠΊΠΎΠ²Ρ‹ΠΌ ΠΈ Ρ€Π°Π²Π½Ρ‹ΠΌ ο₯. Π‘ΠΈΠ»Π° Ρ‚ΠΎΠΊΠ° Ρ‡Π΅Ρ€Π΅Π· ΠΎΠ±Ρ‰ΡƒΡŽ ΡˆΠΈΠ½Ρƒ опрСдСлится ΠΊΠ°ΠΊ

    . (1)

    Π‘ΠΈΠ»Π° Ρ‚ΠΎΠΊΠ° Ρ‡Π΅Ρ€Π΅Π· ΠΏΠ΅Ρ€Π²Ρ‹ΠΉ источник

    . (2)

    Π‘ΠΈΠ»Π° Ρ‚ΠΎΠΊΠ° Ρ‡Π΅Ρ€Π΅Π· Π²Ρ‚ΠΎΡ€ΠΎΠΉ источник

    . (3)

    ЭлСктродвиТущая сила Π²Ρ‚ΠΎΡ€ΠΎΠ³ΠΎ источника

    . (4)

    3. Π‘Π»Π΅Π΄ΡƒΡŽΡ‰ΠΈΠΉ способ отличаСтся ΠΎΡ‚ ΠΏΡ€Π΅Π΄Ρ‹Π΄ΡƒΡ‰Π΅Π³ΠΎ Ρ‚Π΅ΠΌ, Ρ‡Ρ‚ΠΎ источники Π²ΠΊΠ»ΡŽΡ‡Π΅Π½Ρ‹ встрСчно. Π§Ρ‚ΠΎΠ±Ρ‹ ΠΏΠΎΠ»ΡƒΡ‡ΠΈΡ‚ΡŒ Π² Ρ€Π΅Π·ΡƒΠ»ΡŒΡ‚Π°Ρ‚Π΅ Π±Π°Ρ‚Π°Ρ€Π΅ΡŽ с Π­Π”Π‘, Ρ€Π°Π²Π½ΠΎΠΉ ο₯, Π½Π΅ΠΎΠ±Ρ…ΠΎΠ΄ΠΈΠΌΠΎ, Ρ‡Ρ‚ΠΎΠ±Ρ‹ Ρƒ Π²Ρ‚ΠΎΡ€ΠΎΠ³ΠΎ элСмСнта Π­Π”Π‘ Π±Ρ‹Π»Π° Ρ€Π°Π²Π½Π° ο₯/2. Как ΠΈ Π² ΠΏΡ€Π΅Π΄Ρ‹Π΄ΡƒΡ‰Π΅ΠΌ случаС сила Ρ‚ΠΎΠΊΠ° Π±ΡƒΠ΄Π΅Ρ‚ ΠΎΠΏΡ€Π΅Π΄Π΅Π»ΡΡ‚ΡŒΡΡ ΡƒΡ€Π°Π²Π½Π΅Π½ΠΈΠ΅ΠΌ (1), ΠΏΠΎΡ‚ΠΎΠΌΡƒ Ρ‡Ρ‚ΠΎ Π²Π½ΡƒΡ‚Ρ€Π΅Π½Π½ΠΈΠ΅ сопротивлСния Π²ΠΊΠ»ΡŽΡ‡Π΅Π½Ρ‹ ΠΏΠ°Ρ€Π°Π»Π»Π΅Π»ΡŒΠ½ΠΎ. Π‘ΠΈΠ»Π° Ρ‚ΠΎΠΊΠ° Ρ‡Π΅Ρ€Π΅Π· ΠΏΠ΅Ρ€Π²Ρ‹ΠΉ источник Π±ΡƒΠ΄Π΅Ρ‚ ΠΎΠΏΡ€Π΅Π΄Π΅Π»ΡΡ‚ΡŒΡΡ ΠΊΠ°ΠΊ

    . (5)

    Π’ΠΎΠΊ Ρ‡Π΅Ρ€Π΅Π· Π²Ρ‚ΠΎΡ€ΠΎΠΉ источник

    . (6)

    ЭлСктродвиТущая сила Π²Ρ‚ΠΎΡ€ΠΎΠ³ΠΎ элСмСнта Π΄ΠΎΠ»ΠΆΠ½Π° ΡΠΎΡΡ‚Π°Π²Π»ΡΡ‚ΡŒ

    . (7)

    2.3.5. Π’Ρ€ΠΈ ΠΎΠ΄ΠΈΠ½Π°ΠΊΠΎΠ²Ρ‹Π΅ Π±Π°Ρ‚Π°Ρ€Π΅ΠΈ соСдинСны ΠΏΠ°Ρ€Π°Π»Π»Π΅Π»ΡŒΠ½ΠΎ ΠΈ ΠΏΠΎΠ΄ΠΊΠ»ΡŽΡ‡Π΅Π½Ρ‹ ΠΊ Π²Π½Π΅ΡˆΠ½Π΅ΠΌΡƒ ΡΠΎΠΏΡ€ΠΎΡ‚ΠΈΠ²Π»Π΅Π½ΠΈΡŽ. Как измСнится сила Ρ‚ΠΎΠΊΠ° Ρ‡Π΅Ρ€Π΅Π· это сопротивлСниС, Ссли ΠΏΠΎΠ»ΡΡ€Π½ΠΎΡΡ‚ΡŒ ΠΎΠ΄Π½ΠΎΠΉ ΠΈΠ· Π±Π°Ρ‚Π°Ρ€Π΅ΠΉ ΠΏΠΎΠΌΠ΅Π½ΡΡ‚ΡŒ Π½Π° ΠΎΠ±Ρ€Π°Ρ‚Π½ΡƒΡŽ?

    РСшСниС

    1. ΠžΡ‚ΠΌΠ΅Ρ‚ΠΈΠΌ сразу Ρ‡Ρ‚ΠΎ, Π² связи с ΠΈΠ΄Π΅Π½Ρ‚ΠΈΡ‡Π½ΠΎΡΡ‚ΡŒΡŽ элСмСнтов Π² ΠΎΠ±ΠΎΠΈΡ… случаях ΠΈΡ… ΠΏΠ°Ρ€Π°Π»Π»Π΅Π»ΡŒΠ½ΠΎΠ³ΠΎ Π²ΠΊΠ»ΡŽΡ‡Π΅Π½ΠΈΡ суммарноС Π²Π½ΡƒΡ‚Ρ€Π΅Π½Π½Π΅Π΅ сопротивлСниС Π±ΡƒΠ΄Π΅Ρ‚ Π² Ρ‚Ρ€ΠΈ Ρ€Π°Π·Π° мСньшС, Ρ‡Π΅ΠΌ Ρƒ ΠΎΠ΄Π½ΠΎΠ³ΠΎ источника, ΠΏΡ€ΠΈ этом ΠΏΡ€ΠΈ согласном Π²ΠΊΠ»ΡŽΡ‡Π΅Π½ΠΈΠΈ сила Ρ‚ΠΎΠΊΠ° Ρ‡Π΅Ρ€Π΅Π· внСшнСС сопротивлСниС R опрСдСлится ΡƒΡ€Π°Π²Π½Π΅Π½ΠΈΠ΅ΠΌ

    . (1)

    2. ΠŸΡ€ΠΎΠ°Π½Π°Π»ΠΈΠ·ΠΈΡ€ΡƒΠ΅ΠΌ ΡΠΈΡ‚ΡƒΠ°Ρ†ΠΈΡŽ ΠΏΡ€ΠΈ встрСчном Π²ΠΊΠ»ΡŽΡ‡Π΅Π½ΠΈΠΈ ΠΎΠ΄Π½ΠΎΠ³ΠΎ ΠΈΠ· источников Ρ‚ΠΎΠΊΠ°. Π Π΅Π·ΡƒΠ»ΡŒΡ‚ΠΈΡ€ΡƒΡŽΡ‰ΠΈΠΉ Ρ‚ΠΎΠΊ опрСдСлится ΠΊΠ°ΠΊ

    . (2)

    3. ΠžΡ‚Π½ΠΎΡˆΠ΅Π½ΠΈΠ΅ сил Ρ‚ΠΎΠΊΠΎΠ²

    . (3)

    2.3.6. Π§Ρ‚ΠΎ ΠΏΠΎΠΊΠ°ΠΆΠ΅Ρ‚ Π²ΠΎΠ»ΡŒΡ‚ΠΌΠ΅Ρ‚Ρ€, Ссли Π² Ρ†Π΅ΠΏΠΈ, ΠΈΠ·ΠΎΠ±Ρ€Π°ΠΆΡ‘Π½Π½ΠΎΠΉ Π½Π° рисункС, Ссли источники ΠΎΠ΄ΠΈΠ½Π°ΠΊΠΎΠ²Ρ‹, Π­Π”Π‘ ΠΊΠ°ΠΆΠ΄ΠΎΠ³ΠΎ ΠΈΠ· Π½ΠΈΡ… ο₯ =1,5 Π’, Π²Π½ΡƒΡ‚Ρ€Π΅Π½Π½Π΅Π΅ сопротивлСниС r = 2 Ом? Π§Π΅ΠΌΡƒ Π±ΡƒΠ΄Π΅Ρ‚ Ρ€Π°Π²Π½Π° сила Ρ‚ΠΎΠΊΠ° Π² Ρ†Π΅ΠΏΠΈ?

    РСшСниС

    1. Π‘ΡƒΠ΄Π΅ΠΌ ΡΡ‡ΠΈΡ‚Π°Ρ‚ΡŒ, Ρ‡Ρ‚ΠΎ Π²ΠΎΠ»ΡŒΡ‚ΠΌΠ΅Ρ‚Ρ€ ΠΎΠ±Π»Π°Π΄Π°Π΅Ρ‚ бСсконСчно большим сопротивлСниСм, Π² этом случаС сила Ρ‚ΠΎΠΊΠ° Π² Ρ†Π΅ΠΏΠΈ опрСдСлится ΡΠΎΠΎΡ‚Π½ΠΎΡˆΠ΅Π½ΠΈΠ΅ΠΌ

    . (1)

    2. ΠŸΠΎΡΠΊΠΎΠ»ΡŒΠΊΡƒ всС Ρ‚Ρ€ΠΈ элСмСнта Π² Π΄Π°Π½Π½ΠΎΠΉ схСмС Π²ΠΊΠ»ΡŽΡ‡Π΅Π½ΠΈΡ Ρ€Π°Π±ΠΎΡ‚Π°ΡŽΡ‚ Π² Ρ€Π΅ΠΆΠΈΠΌΠ΅ ΠΊΠΎΡ€ΠΎΡ‚ΠΊΠΎΠ³ΠΎ замыкания, ΠΈ Ρ‚ΠΎΠΊ I0, ΠΏΠΎ сути являСтся Ρ‚ΠΎΠΊΠΎΠΌ ΠΊΠΎΡ€ΠΎΡ‚ΠΊΠΎΠ³ΠΎ замыкания, Ρ‚ΠΎ Π² ΡƒΠΊΠ°Π·Π°Π½Π½Ρ‹Ρ… Π½Π° схСмС Ρ‚ΠΎΡ‡ΠΊΠ°Ρ… Ρ€Π°Π·Π½ΠΎΡΡ‚ΡŒ ΠΏΠΎΡ‚Π΅Π½Ρ†ΠΈΠ°Π»ΠΎΠ² Π±ΡƒΠ΄Π΅Ρ‚ Ρ€Π°Π²Π½Π° Π½ΡƒΠ»ΡŽ, Ρ‚.Π΅. UV =0.

    2.3.7. ΠžΠΏΡ€Π΅Π΄Π΅Π»ΠΈΡ‚Π΅ заряд кондСнсатора Π‘ Ρ‘ΠΌΠΊΠΎΡΡ‚ΡŒΡŽ Π‘ = 4 ΠΌΠΊΠ€ Π² стационарном Ρ€Π΅ΠΆΠΈΠΌΠ΅, Ссли R1 = R2 = R3 = R= 100 Ом. Π˜ΡΡ‚ΠΎΡ‡Π½ΠΈΠΊ Ρ‚ΠΎΠΊΠ° ΠΎΠ±Π»Π°Π΄Π°Π΅Ρ‚ Π­Π”Π‘ ο₯ = 300 Π’ ΠΈ Π½ΡƒΠ»Π΅Π²Ρ‹ΠΌ Π²Π½ΡƒΡ‚Ρ€Π΅Π½Π½ΠΈΠΌ сопротивлСниСм.

    РСшСниС

    1. БопротивлСния R2 ΠΈ R3 Π²ΠΊΠ»ΡŽΡ‡Π΅Π½Ρ‹ ΠΏΠ°Ρ€Π°Π»Π»Π΅Π»ΡŒΠ½ΠΎ, поэтому ΠΈΡ… ΠΌΠΎΠΆΠ½ΠΎ ΠΏΡ€Π΅Π΄ΡΡ‚Π°Π²ΠΈΡ‚ΡŒ эквивалСнтным ΠΎΠ΄Π½ΠΈΠΌ сопротивлСниСм Π²Π΅Π»ΠΈΡ‡ΠΈΠ½ΠΎΠΉ

    . (1)

    2. ΠžΠΏΡ€Π΅Π΄Π΅Π»ΠΈΠΌ силу Ρ‚ΠΎΠΊΠ° Π² Ρ†Π΅ΠΏΠΈ

    . (2)

    3. ПадСниС напряТСния Π½Π° сопротивлСнии R1 Π±ΡƒΠ΄Π΅Ρ‚ Ρ€Π°Π²Π½ΠΎ разности ΠΏΠΎΡ‚Π΅Π½Ρ†ΠΈΠ°Π»ΠΎΠ² Π½Π° ΠΎΠ±ΠΊΠ»Π°Π΄ΠΊΠ°Ρ… кондСнсатора, ΠΊΠΎΡ‚ΠΎΡ€Ρ‹ΠΉ для постоянного Ρ‚ΠΎΠΊΠ° ΠΎΠ±Π»Π°Π΄Π°Π΅Ρ‚ бСсконСчным сопротивлСниСм

    . (3)

    4. Заряд кондСнсатора ΠΎΠΏΡ€Π΅Π΄Π΅Π»ΠΈΠΌ ΠΈΠ· уравнСния энСргии

    . (4)

    2.3.8. Π”Π²Π° Π²Π΅Ρ€Ρ‚ΠΈΠΊΠ°Π»ΡŒΠ½ΠΎ располоТСнных стСрТня, ΠΈΠΌΠ΅ΡŽΡ‰ΠΈΠ΅ Π΄Π»ΠΈΠ½Ρƒ L = 1 ΠΌ ΠΈ Π΄ΠΈΠ°ΠΌΠ΅Ρ‚Ρ€ d = 1 см сопротивлСниС Π½Π° Π΅Π΄ΠΈΠ½ΠΈΡ†Ρƒ Π΄Π»ΠΈΠ½Ρ‹  = 1οƒ—10 ο€­ 5 ΠžΠΌοƒ—ΠΌ, подсоСдинСны Ρ‡Π΅Ρ€Π΅Π· ΠΈΠ΄Π΅Π°Π»ΡŒΠ½Ρ‹ΠΉ Π°ΠΌΠΏΠ΅Ρ€ΠΌΠ΅Ρ‚Ρ€ ΠΊ источнику Π­Π”Π‘ ο₯ = 1,5 Π’ ΠΈ Π²Π½ΡƒΡ‚Ρ€Π΅Π½Π½ΠΈΠΌ сопротивлСниСм r0 = 0,05 Ом. Полосок касаСтся сопротивлСниС R = 0,1 Ом, ΠΊΠΎΡ‚ΠΎΡ€ΠΎΠ΅ Π² ΠΏΠΎΠ»Π΅ тяТСсти g Π½Π°Ρ‡ΠΈΠ½Π°Π΅Ρ‚ ΡΠΎΡΠΊΠ°Π»ΡŒΠ·Ρ‹Π²Π°Ρ‚ΡŒ вдоль Π½ΠΈΡ… ΠΈΠ· Π²Π΅Ρ€Ρ…Π½Π΅ΠΉ Ρ‚ΠΎΡ‡ΠΊΠΈ Π²Π½ΠΈΠ· Π±Π΅Π· Π½Π°Ρ€ΡƒΡˆΠ΅Π½ΠΈΡ ΠΊΠΎΠ½Ρ‚Π°ΠΊΡ‚Π°, ΠΊΠ°ΠΊ ΠΏΠΎΠΊΠ°Π·Π°Π½ΠΎ Π½Π° рисункС. Π’ ΠΏΡ€Π΅Π½Π΅Π±Ρ€Π΅ΠΆΠ΅Π½ΠΈΠΈ эффСктами, связанными с ΠΌΠ°Π³Π½ΠΈΡ‚Π½Ρ‹ΠΌ ΠΏΠΎΠ»Π΅ΠΌ, ΠΎΠΏΡ€Π΅Π΄Π΅Π»ΠΈΡ‚ΡŒ ΠΊΠ°ΠΊΠΎΠ΅ Π·Π½Π°Ρ‡Π΅Π½ΠΈΠ΅ Ρ‚ΠΎΠΊΠ° I ΠΏΠΎΠΊΠ°ΠΆΠ΅Ρ‚ Π°ΠΌΠΏΠ΅Ρ€ΠΌΠ΅Ρ‚Ρ€ Ρ‡Π΅Ρ€Π΅Π· врСмя  = 0,5 с послС Π½Π°Ρ‡Π°Π»Π° двиТСния? Π‘ΠΈΠ»Ρƒ трСния Π½Π΅ ΡƒΡ‡ΠΈΡ‚Ρ‹Π²Π°Ρ‚ΡŒ

    РСшСниС

    1. Π—Π°ΠΏΠΈΡˆΠ΅ΠΌ кинСматичСскиС уравнСния двиТСния сопротивлСния, считая, Ρ‡Ρ‚ΠΎ Π½Π° Π½Π΅Π³ΠΎ дСйствуСт Ρ‚ΠΎΠ»ΡŒΠΊΠΎ сила тяТСсти ΠΈ Π΄Π²ΠΈΠΆΠ΅Π½ΠΈΠ΅ происходит ΠΏΠΎ Π²Π΅Ρ€Ρ‚ΠΈΠΊΠ°Π»ΡŒΠ½ΠΎΠΉ оси с Π½ΡƒΠ»Π΅Π²ΠΎΠΉ Π½Π°Ρ‡Π°Π»ΡŒΠ½ΠΎΠΉ ΡΠΊΠΎΡ€ΠΎΡΡ‚ΡŒΡŽ

    , (1)

    ΠΈ ΠΎΠΏΡ€Π΅Π΄Π΅Π»ΠΈΠΌ расстояниС ΠΊΠΎΡ‚ΠΎΡ€ΠΎΠ΅ ΠΏΡ€ΠΎΠΉΠ΄Ρ‘Ρ‚ сопротивлСниС Π·Π° врСмя 

    . (2)

    2. ΠžΠΏΡ€Π΅Π΄Π΅Π»ΠΈΠΌ элСктричСскоС сопротивлСниС ΠΎΠ΄Π½ΠΎΠ³ΠΎ ΠΎΡ‚Ρ€Π΅Π·ΠΊΠ° стСрТня Π΄Π»ΠΈΠ½ΠΎΠΉ

    . (3)

    3. ЭлСктричСская схСма установки, Ρ‚Π°ΠΊΠΈΠΌ ΠΎΠ±Ρ€Π°Π·ΠΎΠΌ прСдставит собой Ρ‚Ρ€ΠΈ ΠΏΠΎΡΠ»Π΅Π΄ΠΎΠ²Π°Ρ‚Π΅Π»ΡŒΠ½ΠΎ Π²ΠΊΠ»ΡŽΡ‡Π΅Π½Π½Ρ‹Ρ… Π²Π½Π΅ΡˆΠ½ΠΈΡ… сопротивлСния: R0 = R + 2r

    ΠΈ Π²Π½ΡƒΡ‚Ρ€Π΅Π½Π½Π΅Π΅ сопротивлСниС источника r0. Π—Π°ΠΊΠΎΠ½ Ома для ΠΏΠΎΠ»Π½ΠΎΠΉ Ρ†Π΅ΠΏΠΈ Π² этом случаС Π·Π°ΠΏΠΈΡˆΠ΅Ρ‚ΡΡ Ρ‚Π°ΠΊ

    . (4)

    2.3.9. Π”Π²Π° Π³Π°Π»ΡŒΠ²Π°Π½ΠΈΡ‡Π΅ΡΠΊΠΈΡ… элСмСнта с ο₯1 =1,5 Π’ ΠΈ ο₯2 = 4,5 Π’ соСдинСны ΠΎΠ΄Π½ΠΎΠΈΠΌΡ‘Π½Π½Ρ‹ΠΌΠΈ полюсами. Π’Π½ΡƒΡ‚Ρ€Π΅Π½Π½Π΅Π΅ сопротивлСниС ΠΏΠ΅Ρ€Π²ΠΎΠ³ΠΎ источника r1 Π² Π΄Π²Π° Ρ€Π°Π·Π° мСньшС Π²Π½ΡƒΡ‚Ρ€Π΅Π½Π½Π΅Π³ΠΎ сопротивлСния Π²Ρ‚ΠΎΡ€ΠΎΠ³ΠΎ элСмСнта r2, Ρ‚.Π΅. r2 = 2 r1. ΠšΠ°ΠΊΠΎΠ²Ρ‹ ΠΏΡ€ΠΈ этом Π²ΠΊΠ»ΡŽΡ‡Π΅Π½ΠΈΠΈ элСмСнтов Π±ΡƒΠ΄ΡƒΡ‚ показания Π²ΠΎΠ»ΡŒΡ‚ΠΌΠ΅Ρ‚Ρ€Π°?

    РСшСниС

    1. Если ΡΡ‡ΠΈΡ‚Π°Ρ‚ΡŒ, Ρ‡Ρ‚ΠΎ Π²ΠΎΠ»ΡŒΡ‚ΠΌΠ΅Ρ‚Ρ€ ΠΎΠ±Π»Π°Π΄Π°Π΅Ρ‚ бСсконСчным сопротивлСниСм, Ρ‚ΠΎ Ρ€Π°Π·Π½ΠΎΡΡ‚ΡŒ элСктродвиТущих сил источников Ρ‚ΠΎΠΊΠ° Π±ΡƒΠ΄Π΅Ρ‚ Ρ€Π°Π²Π½Π° суммС ΠΏΠ°Π΄Π΅Π½ΠΈΠΉ напряТСния Π½Π° ΠΈΡ… Π²Π½ΡƒΡ‚Ρ€Π΅Π½Π½ΠΈΡ… сопротивлСниях

    . (1)

    2. Π‘ Π΄Ρ€ΡƒΠ³ΠΎΠΉ стороны Π²Ρ‚ΠΎΡ€ΠΎΠΉ элСмСнт являСтся внСшнСй Π½Π°Π³Ρ€ΡƒΠ·ΠΊΠΎΠΉ для ΠΏΠ΅Ρ€Π²ΠΎΠ³ΠΎ элСмСнта

    , (2)

    Π³Π΄Π΅ U ο€­ показания Π²ΠΎΠ»ΡŒΡ‚ΠΌΠ΅Ρ‚Ρ€Π°.

    3. Π’Ρ‹Ρ€Π°Π·ΠΈΠΌ ΠΈΠ· послСднСго уравнСния силу Ρ‚ΠΎΠΊΠ° Π² Ρ†Π΅ΠΏΠΈ

    . (3)

    4. ΠŸΠΎΠ΄ΡΡ‚Π°Π²ΠΈΠΌ Π·Π½Π°Ρ‡Π΅Π½ΠΈΠ΅ силы Ρ‚ΠΎΠΊΠ° Π² ΡƒΡ€Π°Π²Π½Π΅Π½ΠΈΠ΅ (1)

    , (4)

    ΠΎΡ‚ΠΊΡƒΠ΄Π°

    . (5)

    2.3.10. Π˜ΡΡ‚ΠΎΡ‡Π½ΠΈΠΊ Ρ‚ΠΎΠΊΠ° ΠΎΠ±Π»Π°Π΄Π°Π΅Ρ‚ Π²Π½ΡƒΡ‚Ρ€Π΅Π½Π½ΠΈΠΌ сопротивлСниСм r = 1 Ом, Ρ‘ΠΌΠΊΠΎΡΡ‚ΡŒ кондСнсатора Π‘ = 10 ΠΌΠΊΠ€, R1 = 5 Ом, R2 = 10 Ом. Π”ΠΎ замыкания ΠΊΠ»ΡŽΡ‡Π° Π²ΠΎΠ»ΡŒΡ‚ΠΌΠ΅Ρ‚Ρ€ ΠΏΠΎΠΊΠ°Π·Ρ‹Π²Π°Π΅Ρ‚ напряТСниС U1 = 10 Π’, Π° послС замыкания ο€­ U2 = 8 Π’. ΠžΠΏΡ€Π΅Π΄Π΅Π»ΠΈΡ‚ΡŒ заряд кондСнсатора ΠΈ Π²Π΅Π»ΠΈΡ‡ΠΈΠ½Ρƒ сопротивлСния R3.

    РСшСниС

    1. ΠŸΡ€ΠΈ Ρ€Π°Π·ΠΎΠΌΠΊΠ½ΡƒΡ‚ΠΎΠΌ ΠΊΠ»ΡŽΡ‡Π΅ Ρ‚ΠΎΠΊ Π² Ρ†Π΅ΠΏΠΈ отсутствуСт, поэтому Π²ΠΎΠ»ΡŒΡ‚ΠΌΠ΅Ρ‚Ρ€ Π±ΡƒΠ΄Π΅Ρ‚ Π΄Π΅ΠΌΠΎΠ½ΡΡ‚Ρ€ΠΈΡ€ΠΎΠ²Π°Ρ‚ΡŒ Π²Π΅Π»ΠΈΡ‡ΠΈΠ½Ρƒ Π­Π”Π‘, U1 = ο₯ = 10 Π’.

    2. Π—Π°ΠΏΠΈΡˆΠ΅ΠΌ Π΄Π°Π»Π΅Π΅ ΡƒΡ€Π°Π²Π½Π΅Π½ΠΈΠ΅ ΠΎΠ±Ρ‰Π΅Π³ΠΎ сопротивлСния Ρ†Π΅ΠΏΠΈ, считая Ρ‡Ρ‚ΠΎ кондСнсатор для постоянного Ρ‚ΠΎΠΊΠ° Π² стационарном Ρ€Π΅ΠΆΠΈΠΌΠ΅ прСдставляСт бСсконСчноС сопротивлСниС

    , (1)

    с Π΄Ρ€ΡƒΠ³ΠΎΠΉ стороны

    . (2)

    3. ΠžΠΏΡ€Π΅Π΄Π΅Π»ΠΈΠΌ Π²Π΅Π»ΠΈΡ‡ΠΈΠ½Ρƒ сопротивлСния R3

    . (3)

    4. ΠžΠΏΡ€Π΅Π΄Π΅Π»ΠΈΠΌ ΠΏΠ°Π΄Π΅Π½ΠΈΠ΅ напряТСния Π½Π° сопротивлСнии R3, ΠΊΠΎΡ‚ΠΎΡ€ΠΎΠ΅ Π²ΠΊΠ»ΡŽΡ‡Π΅Π½ΠΎ ΠΏΠ°Ρ€Π°Π»Π»Π΅Π»ΡŒΠ½ΠΎ кондСнсатору

    . (4)

    5. Заряд, ΠΏΡ€ΠΎΡˆΠ΅Π΄ΡˆΠΈΠΉ Ρ‡Π΅Ρ€Π΅Π· кондСнсатор

    . (5)

    2.3.11. Π˜Π΄Π΅Π°Π»ΡŒΠ½Ρ‹ΠΉ источник Ρ‚ΠΎΠΊΠ° с ο₯ = 100 Π’ Π²ΠΊΠ»ΡŽΡ‡Π΅Π½ Π² Ρ†Π΅ΠΏΡŒ, ΡΠΎΡΡ‚ΠΎΡΡ‰ΡƒΡŽ ΠΈΠ· кондСнсаторов Π‘3 = Π‘4 = 1 ΠΌΠΊΠ€, Π‘1 = 2 ΠΌΠΊΠ€, Π‘2 =4 ΠΌΠΊΠ€ ΠΈ сопротивлСния R. ΠžΠΏΡ€Π΅Π΄Π΅Π»ΠΈΡ‚ΡŒ ΠΏΠ°Π΄Π΅Π½ΠΈΠ΅ напряТСния Π½Π° кондСнсаторах Π‘1 ΠΈ Π‘2.

    РСшСниС

    1. ΠŸΡ€ΠΈ ΠΏΠΎΠ΄ΠΊΠ»ΡŽΡ‡Π΅Π½ΠΈΠΈ схСмы ΠΊ источнику Π² Ρ†Π΅ΠΏΠΈ ΠΏΠΎΡ‚Π΅Ρ‡Ρ‘Ρ‚ Ρ‚ΠΎΠΊ Π΄ΠΎ ΠΌΠΎΠΌΠ΅Π½Ρ‚Π° ΠΏΠΎΠ»Π½ΠΎΠΉ зарядки всСх кондСнсаторов. ПослС Ρ‚ΠΎΠ³ΠΎ ΠΊΠ°ΠΊ кондСнсаторы зарядятся Ρ‚ΠΎΠΊ прСкращаСтся, Ρ‚.ΠΊ. элСктричСскиС ёмкости ΠΏΡ€Π΅Π΄ΡΡ‚Π°Π²Π»ΡΡŽΡ‚ для постоянного Ρ‚ΠΎΠΊΠ° Ρ€Π°Π·Ρ€Ρ‹Π² Ρ†Π΅ΠΏΠΈ.

    2. ВсС ΠΎΠ±ΠΊΠ»Π°Π΄ΠΊΠΈ кондСнсаторов, соСдинённыС с сопротивлСниСм Π±ΡƒΠ΄ΡƒΡ‚ ΠΈΠΌΠ΅Ρ‚ΡŒ ΠΎΠ΄ΠΈΠ½Π°ΠΊΠΎΠ²Ρ‹ΠΉ ΠΏΠΎΡ‚Π΅Π½Ρ†ΠΈΠ°Π», ΠΏΡ€ΠΈ этом ΠΏΠ°Ρ€Ρ‹ кондСнсаторов Π‘1 + Π‘3 ΠΈ Π‘2 + Π‘4 Π²ΠΊΠ»ΡŽΡ‡Π΅Π½Ρ‹ с источником Ρ‚ΠΎΠΊΠ° ΠΏΠΎΡΠ»Π΅Π΄ΠΎΠ²Π°Ρ‚Π΅Π»ΡŒΠ½ΠΎ.

    3. ПадСниС напряТСния Π½Π° кондСнсаторах опрСдСлится ΡƒΡ€Π°Π²Π½Π΅Π½ΠΈΠ΅ΠΌ

    . (1)

    4. Заряд кондСнсаторов опрСдСлится ΠΊΠ°ΠΊ

    . (2)

    5. Π’Ρ‹Ρ€Π°Π·ΠΈΠΌ ΠΈΠ· послСднСго уравнСния Π²Π΅Π»ΠΈΡ‡ΠΈΠ½Ρƒ U2, подставим Π΅Ρ‘ Π² ΡƒΡ€Π°Π²Π½Π΅Π½ΠΈΠ΅ (1) ΠΈ Ρ€Π°Π·Ρ€Π΅ΡˆΠΈΠΌ Π΅Π³ΠΎ ΠΎΡ‚Π½ΠΎΡΠΈΡ‚Π΅Π»ΡŒΠ½ΠΎ U1

    , (3)

    , (4)

    . (5)

    6. ΠžΠΏΡ€Π΅Π΄Π΅Π»ΠΈΠΌ Π΄Π°Π»Π΅Π΅ Π²Π΅Π»ΠΈΡ‡ΠΈΠ½Ρƒ U2 ΠΈΠ· уравнСния (1)

    . (6)

    2.3.12. ЭлСктричСская схСма состоит ΠΈΠ· Π΄Π²ΡƒΡ… кондСнсаторов Π‘1 = 2 ΠΌΠΊΠ€ ΠΈ Π‘2 = 4 ΠΌΠΊΠ€ ΠΈ Ρ‚Ρ€Ρ‘Ρ… сопротивлСний R1 = 200 Ом, R2 = R3 = 100 Ом. Π’ Ρ†Π΅ΠΏΡŒ Π²ΠΊΠ»ΡŽΡ‡Ρ‘Π½ ΠΈΠ΄Π΅Π°Π»ΡŒΠ½Ρ‹ΠΉ источник Ρ‚ΠΎΠΊΠ° с ο₯ = 100 Π’. ΠžΠΏΡ€Π΅Π΄Π΅Π»ΠΈΡ‚ΡŒ ΠΏΠ°Π΄Π΅Π½ΠΈΠ΅ напряТСния Π½Π° кондСнсаторах U1, U2 ΠΈ ΠΈΡ… заряд Q1, Q2.

    РСшСниС

    1. ПадСниС напряТСния U1 Π½Π° кондСнсаторС Π‘1 Ρ€Π°Π²Π½ΠΎ разности ΠΏΠΎΡ‚Π΅Π½Ρ†ΠΈΠ°Π»ΠΎΠ² ΠΌΠ΅ΠΆΠ΄Ρƒ Ρ‚ΠΎΡ‡ΠΊΠ°ΠΌΠΈ Ρ†Π΅ΠΏΠΈ 1 ΠΈ 3, Π° напряТСниС Π½Π° Π‘2 опрСдСляСтся Ρ€Π°Π·Π½ΠΎΡΡ‚ΡŒΡŽ ΠΏΠΎΡ‚Π΅Π½Ρ†ΠΈΠ°Π»ΠΎΠ² ΠΌΠ΅ΠΆΠ΄Ρƒ Ρ‚ΠΎΡ‡ΠΊΠ°ΠΌΠΈ 2 ΠΈ 4

    , . (1)

    2. ПослС зарядки кондСнсаторов Ρ†Π΅ΠΏΡŒ Π±ΡƒΠ΄Π΅Ρ‚ ΠΏΡ€Π΅Π΄ΡΡ‚Π°Π²Π»ΡΡ‚ΡŒ собой Ρ‚Ρ€ΠΈ ΠΏΠΎΡΠ»Π΅Π΄ΠΎΠ²Π°Ρ‚Π΅Π»ΡŒΠ½ΠΎ соСдинённых сопротивлСния

    =400 Ом. (2)

    3. ΠžΠΏΡ€Π΅Π΄Π΅Π»ΠΈΠΌ силу Ρ‚ΠΎΠΊΠ° Π² Ρ†Π΅ΠΏΠΈ

    . (3)

    4. ΠžΠΏΡ€Π΅Π΄Π΅Π»ΠΈΠΌ Π²Π΅Π»ΠΈΡ‡ΠΈΠ½Ρƒ напряТСний U1, U2 ΠΊΠΎΡ‚ΠΎΡ€Ρ‹Π΅, ΠΊΠ°ΠΊ слСдуСт ΠΈΠ· ΡƒΡ€Π°Π²Π½Π΅Π½ΠΈΠΉ (1) Π±ΡƒΠ΄ΡƒΡ‚ Ρ€Π°Π²Π½Ρ‹ суммС ΠΏΠ°Π΄Π΅Π½ΠΈΠΉ напряТСния Π½Π° сопротивлСниях U1 = UR1 + UR2, U2 = UR3 + UR4

    , (4)

    , (5)

    5. Заряд кондСнсаторов ΠΎΠΏΡ€Π΅Π΄Π΅Π»ΠΈΠΌ, ΠΈΡΠΏΠΎΠ»ΡŒΠ·ΡƒΡ взаимосвязь падСния напряТСния заряда ΠΈ ёмкости

    (6)

    2.3.13. Π”Π²Π° ΠΏΠΎΡΠ»Π΅Π΄ΠΎΠ²Π°Ρ‚Π΅Π»ΡŒΠ½ΠΎ соСдинённых кондСнсатора Π‘1 = 2 ΠΌΠΊΠ€ ΠΈ Π‘2 = 4 ΠΌΠΊΠ€ Π·Π°ΠΌΠΊΠ½ΡƒΡ‚Ρ‹ Π½Π° источник Ρ‚ΠΎΠΊΠ° с ο₯ = 20 Π’, ΠΏΠ°Ρ€Π°Π»Π»Π΅Π»ΡŒΠ½ΠΎ ΠΊΠΎΡ‚ΠΎΡ€ΠΎΠΌΡƒ Π²ΠΊΠ»ΡŽΡ‡Π΅Π½ΠΎ сопротивлСниС R = 20 Ом. Π’ΠΎΠΊ ΠΊΠΎΡ€ΠΎΡ‚ΠΊΠΎΠ³ΠΎ замыкания источника IΠšΠ— Π² Ρ‚Ρ€ΠΈ Ρ€Π°Π·Π° ΠΏΡ€Π΅Π²Ρ‹ΡˆΠ°Π΅Ρ‚ Ρ€Π°Π±ΠΎΡ‡ΠΈΠΉ стационарный Ρ‚ΠΎΠΊ Π² Ρ†Π΅ΠΏΠΈ I. ΠžΠΏΡ€Π΅Π΄Π΅Π»ΠΈΡ‚ΡŒ ΠΏΠ°Π΄Π΅Π½ΠΈΠ΅ напряТСния Π½Π° ΠΊΠ°ΠΆΠ΄ΠΎΠΌ ΠΈΠ· кондСнсаторов.

    РСшСниС

    1. ΠŸΡ€ΠΈ ΠΏΠΎΡΠ»Π΅Π΄ΠΎΠ²Π°Ρ‚Π΅Π»ΡŒΠ½ΠΎΠΌ соСдинСнии кондСнсаторов Ρ‡Π΅Ρ€Π΅Π· Π½ΠΈΡ… ΠΏΡ€ΠΎΡ‚Π΅ΠΊΠ°Π΅Ρ‚ ΠΎΠ΄ΠΈΠ½Π°ΠΊΠΎΠ²Ρ‹ΠΉ зарядный Ρ‚ΠΎΠΊ, поэтому заряд Π½Π° ΠΈΡ… ΠΎΠ±ΠΊΠ»Π°Π΄ΠΊΠ°Ρ… Π±ΡƒΠ΄Π΅Ρ‚ ΠΎΠ΄ΠΈΠ½Π°ΠΊΠΎΠ²Ρ‹ΠΌ, Ρ‚.Π΅. Q1 = Q2

    . (1)

    2. ПадСниС напряТСния Π½Π° кондСнсаторах ΠΌΠΎΠΆΠ½ΠΎ ΠΏΡ€Π΅Π΄ΡΡ‚Π°Π²ΠΈΡ‚ΡŒ Π² Π²ΠΈΠ΄Π΅ суммы

    . (2)

    3. Π’Ρ‹Ρ€Π°Π·ΠΈΠΌ Π΄Π°Π»Π΅Π΅ Π²Π΅Π»ΠΈΡ‡ΠΈΠ½Ρƒ U2 ΠΈΠ· уравнСния (1) подставим Π΅Ρ‘ Π² ΡƒΡ€Π°Π²Π½Π΅Π½ΠΈΠ΅ (2) ΠΈ ΠΎΠΏΡ€Π΅Π΄Π΅Π»ΠΈΠΌ ΠΏΠ°Π΄Π΅Π½ΠΈΠ΅ напряТСния Π½Π° Π‘1 ΠΈ Π‘2

    , (3)

    5. ΠžΠΏΡ€Π΅Π΄Π΅Π»ΠΈΠΌ Π΄Π°Π»Π΅Π΅ Π²Π½ΡƒΡ‚Ρ€Π΅Π½Π½Π΅Π΅ сопротивлСниС источника Ρ‚ΠΎΠΊΠ° ΠΈ Π²Π΅Π»ΠΈΡ‡ΠΈΠ½Ρƒ U0

    . (4)

    6. ΠŸΠΎΠ΄ΡΡ‚Π°Π²ΠΈΠΌ Π΄Π°Π»Π΅Π΅ Π²Π΅Π»ΠΈΡ‡ΠΈΠ½Ρƒ U0 Π² уравнСния (3)

    . (5)

      1. ΠŸΡ€Π°Π²ΠΈΠ»Π° ΠšΠΈΡ€Ρ…Π³ΠΎΡ„Π°

    2.4.1. ΠžΠΏΡ€Π΅Π΄Π΅Π»ΠΈΡ‚ΡŒ силу Ρ‚ΠΎΠΊΠΎΠ² Π²ΠΎ всСх участках Ρ†Π΅ΠΏΠΈ, Ссли источники Ρ‚ΠΎΠΊΠ° ΠΎΠ±Π»Π°Π΄Π°ΡŽΡ‚ Π­Π”Π‘: ο₯1 = 10 B, ο₯2 = 20 Π’, ΠΈΡ… Π²Π½ΡƒΡ‚Ρ€Π΅Π½Π½ΠΈΠ΅ сопротивлСния соотвСтствСнно Ρ€Π°Π²Π½Ρ‹: r1 = 2 Ом, r2 = 3 Ом. Π˜ΡΡ‚ΠΎΡ‡Π½ΠΈΠΊΠΈ Π½Π°Π³Ρ€ΡƒΠΆΠ΅Π½Ρ‹ Π½Π° внСшнСС сопротивлСниС R = 100 Ом.

    РСшСниС

    1. Π—Π°Π΄Π°Ρ‡Ρƒ цСлСсообразно Ρ€Π΅ΡˆΠ°Ρ‚ΡŒ, ΠΈΡΠΏΠΎΠ»ΡŒΠ·ΡƒΡ ΠΏΡ€Π°Π²ΠΈΠ»Π° ΠšΠΈΡ€Ρ…Π³ΠΎΡ„Π°, ΠΊΠΎΡ‚ΠΎΡ€Ρ‹Π΅ ΡƒΠ΄ΠΎΠ±Π½Ρ‹ ΠΏΡ€ΠΈ расчСтах ΠΏΠ°Ρ€Π°ΠΌΠ΅Ρ‚Ρ€ΠΎΠ² Ρ€Π°Π·Π²Π΅Ρ‚Π²Π»Ρ‘Π½Π½Ρ‹Ρ… Ρ†Π΅ΠΏΠ΅ΠΉ. Π’ ΠΎΠ±Ρ‰Π΅ΠΌ Π²ΠΈΠ΄Π΅ матСматичСскиС выраТСния ΠΏΡ€Π°Π²ΠΈΠ» ΠΈΠΌΠ΅ΡŽΡ‚ Π²ΠΈΠ΄:

    . (1)

    2. Π’ соотвСтствиС с ΠΏΠ΅Ρ€Π²Ρ‹ΠΌ ΠΏΡ€Π°Π²ΠΈΠ»ΠΎΠΌ алгСбраичСская сумма сил Ρ‚ΠΎΠΊΠΎΠ² Π² любом ΠΈΠ· ΡƒΠ·Π»ΠΎΠ² Π΄ΠΎΠ»ΠΆΠ½Π° Π±Ρ‹Ρ‚ΡŒ Ρ€Π°Π²Π½Π° Π½ΡƒΠ»ΡŽ

    . (2)

    3. Π’Ρ‹Π΄Π΅Π»ΠΈΠΌ Π΄Π²Π° Π·Π°ΠΌΠΊΠ½ΡƒΡ‚Ρ‹Ρ… ΠΊΠΎΠ½Ρ‚ΡƒΡ€Π°, содСрТащих источники Ρ‚ΠΎΠΊΠ° (Π½Π°ΠΏΡ€Π°Π²Π»Π΅Π½ΠΈΠ΅ ΠΎΠ±Ρ…ΠΎΠ΄Π° ΠΊΠΎΠ½Ρ‚ΡƒΡ€ΠΎΠ² ΠΏΠΎΠΊΠ°Π·Π°Π½ΠΎ ΠΏΡƒΠ½ΠΊΡ‚ΠΈΡ€ΠΎΠΌ) ΠΈ запишСм для Π½ΠΈΡ… Π²Ρ‚ΠΎΡ€ΠΎΠ΅ ΠΏΡ€Π°Π²ΠΈΠ»ΠΎ ΠšΠΈΡ€Ρ…Π³ΠΎΡ„Π°

    . (3)

    4. Π’Π°ΠΊΠΈΠΌ ΠΎΠ±Ρ€Π°Π·ΠΎΠΌ, ΠΏΡ€ΠΈΡ…ΠΎΠ΄ΠΈΠΌ ΠΊ систСмС Ρ‚Ρ€Ρ‘Ρ… алгСбраичСских ΡƒΡ€Π°Π²Π½Π΅Π½ΠΈΠΉ с трСмя нСизвСстными Π²Π΅Π»ΠΈΡ‡ΠΈΠ½Π°ΠΌΠΈ

    (4)

    5. Π’Ρ‹Ρ€Π°Π·ΠΈΠΌ ΠΈΠ· Π²Ρ‚ΠΎΡ€ΠΎΠ³ΠΎ ΠΈ Ρ‚Ρ€Π΅Ρ‚ΡŒΠ΅Π³ΠΎ ΡƒΡ€Π°Π²Π½Π΅Π½ΠΈΠΉ систСмы (4) силы Ρ‚ΠΎΠΊΠ° I1 ΠΈ I2

    , (5)

    ΠΈ подставим эти значСния Π² ΠΏΠ΅Ρ€Π²ΠΎΠ΅ ΡƒΡ€Π°Π²Π½Π΅Π½ΠΈΠ΅ систСмы с Ρ†Π΅Π»ΡŒΡŽ Π΅Π³ΠΎ Ρ€Π΅ΡˆΠ΅Π½ΠΈΡ ΠΎΡ‚Π½ΠΎΡΠΈΡ‚Π΅Π»ΡŒΠ½ΠΎ силы Ρ‚ΠΎΠΊΠ° I

    , (6)

    , (7)

    , (8)

    . (9)

    6. ΠžΠΏΡ€Π΅Π΄Π΅Π»ΠΈΠΌ Π΄Π°Π»Π΅Π΅ Π·Π½Π°Ρ‡Π΅Π½ΠΈΠ΅ сил Ρ‚ΠΎΠΊΠΎΠ² I1 ΠΈ I2

    (10)

    7. Π—Π½Π°ΠΊ минус для Ρ‚ΠΎΠΊΠ° I1 ΠΏΠΎΠΊΠ°Π·Ρ‹Π²Π°Π΅Ρ‚, Ρ‡Ρ‚ΠΎ Π½Π°ΠΏΡ€Π°Π²Π»Π΅Π½ΠΈΠ΅ Ρ‚ΠΎΠΊΠ° Π²Ρ‹Π±Ρ€Π°Π½ΠΎ Π½Π΅ΠΏΡ€Π°Π²ΠΈΠ»ΡŒΠ½ΠΎ, Ρ‚ΠΎΠΊ Π±ΡƒΠ΄Π΅Ρ‚ Ρ‚Π΅Ρ‡ΡŒ Π² ΠΎΠ±Ρ€Π°Ρ‚Π½ΠΎΠΌ Π½Π°ΠΏΡ€Π°Π²Π»Π΅Π½ΠΈΠΈ.

    8. ΠŸΡ€ΠΎΠ²Π΅Ρ€ΠΈΠΌ ΠΏΡ€Π°Π²ΠΈΠ»ΡŒΠ½ΠΎΡΡ‚ΡŒ Ρ€Π΅ΡˆΠ΅Π½ΠΈΡ ΠΏΡƒΡ‚Ρ‘ΠΌ Π°Π½Π°Π»ΠΈΠ·Π° баланса Ρ‚ΠΎΠΊΠΎΠ² ΠΏΠΎ ΡƒΡ€Π°Π²Π½Π΅Π½ΠΈΡŽ (1)

    . (11)

    2.4.2. ЭлСктричСская Ρ†Π΅ΠΏΡŒ состоит ΠΈΠ· рСзисторов R1 = R2 = 10 Ом ΠΈ Ρ‚Ρ€Ρ‘Ρ… ΠΈΠ΄Π΅Π°Π»ΡŒΠ½Ρ‹Ρ… источников Ρ‚ΠΎΠΊΠ°, ΠΏΡ€ΠΈΡ‡Ρ‘ΠΌ ο₯1 = 10 Π’, ο₯2 = 14 Π’. ΠŸΡ€ΠΈ ΠΊΠ°ΠΊΠΎΠΌ Π·Π½Π°Ρ‡Π΅Π½ΠΈΠΈ Π­Π”Π‘ Ρ‚Ρ€Π΅Ρ‚ΡŒΠ΅Π³ΠΎ источника ο₯3 Ρ‚ΠΎΠΊ Ρ‡Π΅Ρ€Π΅Π· сопротивлСниС R3 Π½Π΅ ΠΏΠΎΡ‚Π΅Ρ‡Ρ‘Ρ‚?

    РСшСниС

    1. Π’Ρ‹Π±Π΅Ρ€Π΅ΠΌ Π½Π°ΠΏΡ€Π°Π²Π»Π΅Π½ΠΈΠ΅ Ρ‚ΠΎΠΊΠΎΠ², Π²Ρ‹Π΄Π΅Π»ΠΈΠΌ Π΄Π²Π° ΠΊΠΎΠ½Ρ‚ΡƒΡ€Π° ΠΈ запишСм уравнСния ΠΏΡ€Π°Π²ΠΈΠ» ΠšΠΈΡ€Ρ…Π³ΠΎΡ„Π° Π² соотвСтствии с уравнСниями (1) ΠΏΡ€Π΅Π΄Ρ‹Π΄ΡƒΡ‰Π΅ΠΉ Π·Π°Π΄Π°Ρ‡ΠΈ

    (1)

    2. Π’Π°ΠΊ ΠΊΠ°ΠΊ ΠΏΠΎ ΡƒΡΠ»ΠΎΠ²ΠΈΡŽ Π·Π°Π΄Π°Ρ‡ΠΈ I3 = 0, Ρ‚ΠΎ I1 = ο€­ I2, уравнСния (1) ΠΏΡ€ΠΈ этом ΠΏΡ€ΠΈΠΌΡƒΡ‚ Π²ΠΈΠ΄

    (2)

    3. ПодСлим ΠΏΠΎΡ‡Π»Π΅Π½Π½ΠΎ послСдниС уравнСния Π΄Ρ€ΡƒΠ³ Π½Π° Π΄Ρ€ΡƒΠ³Π° ΠΈ ΠΏΠΎΠ»ΡƒΡ‡Π΅Π½Π½ΠΎΠ΅ ΡΠΎΠΎΡ‚Π½ΠΎΡˆΠ΅Π½ΠΈΠ΅ Ρ€Π°Π·Ρ€Π΅ΡˆΠΈΠΌ ΠΎΡ‚Π½ΠΎΡΠΈΡ‚Π΅Π»ΡŒΠ½ΠΎ ο₯3

    ,

    ,

    . (3)

    2.4.3. Π‘Ρ…Π΅ΠΌΠ° состоит ΠΈΠ· Ρ‚Ρ€Ρ‘Ρ… ΠΈΠ΄Π΅Π°Π»ΡŒΠ½Ρ‹Ρ… источников Π­Π”Π‘, Π΄Π²Π° ΠΈΠ· ΠΊΠΎΡ‚ΠΎΡ€Ρ‹Ρ… Π·Π°Π΄Π°Π½Ρ‹: ο₯1 = 10 Π’, ο₯2 = 8 Π’, ΠΈ Ρ‚Ρ€Ρ‘Ρ… сопротивлСний Π΄Π²Π° ΠΈΠ· ΠΊΠΎΡ‚ΠΎΡ€Ρ‹Ρ… Ρ‚ΠΎΠΆΠ΅ извСстны: R1 = 100 Ом, R2 = 80 Ом. ΠžΠΏΡ€Π΅Π΄Π΅Π»ΠΈΡ‚ΡŒ ΠΏΡ€ΠΈ ΠΊΠ°ΠΊΠΎΠΌ Π·Π½Π°Ρ‡Π΅Π½ΠΈΠΈ ο₯3 Ρ‚ΠΎΠΊ Ρ‡Π΅Ρ€Π΅Π· сопротивлСниС R3 Ρ‚ΠΎΠΊ Ρ‚Π΅Ρ‡ΡŒ Π½Π΅ Π±ΡƒΠ΄Π΅Ρ‚.

    РСшСниС

    1.Π’Ρ‹Π±Π΅Ρ€Π΅ΠΌ ΡƒΠ·Π΅Π» схСмы для ΠΊΠΎΡ‚ΠΎΡ€ΠΎΠ³ΠΎ запишСм ΡƒΡ€Π°Π²Π½Π΅Π½ΠΈΠ΅ ΠΏΠ΅Ρ€Π²ΠΎΠ³ΠΎ ΠΏΡ€Π°Π²ΠΈΠ»Π° ΠšΠΈΡ€Ρ…Π³ΠΎΡ„Π°

    . (1)

    2. Π’Ρ‹Π΄Π΅Π»ΠΈΠΌ Π΄Π²Π° Π·Π°ΠΌΠΊΠ½ΡƒΡ‚Ρ‹Ρ… ΠΊΠΎΠ½Ρ‚ΡƒΡ€Π° ΠΈ ΡΠΎΠ²Π΅Ρ€ΡˆΠΈΠΌ ΠΈΡ… ΠΎΠ±Ρ…ΠΎΠ΄ Π² ΡƒΠΊΠ°Π·Π°Π½Π½Ρ‹Ρ… ΠΏΡƒΠ½ΠΊΡ‚ΠΈΡ€Π½ΠΎΠΉ Π»ΠΈΠ½ΠΈΠ΅ΠΉ направлСниях ΠΏΠΎ Π²Ρ‚ΠΎΡ€ΠΎΠΌΡƒ ΠΏΡ€Π°Π²ΠΈΠ»Ρƒ ΠšΠΈΡ€Ρ…Π³ΠΎΡ„Π°

    . (2)

    3. По ΡƒΡΠ»ΠΎΠ²ΠΈΡŽ Π·Π°Π΄Π°Ρ‡ΠΈ I3 =0, поэтому уравнСния (1) ΠΈ (2) ΠΌΠΎΠΆΠ½ΠΎ ΠΏΠ΅Ρ€Π΅ΠΏΠΈΡΠ°Ρ‚ΡŒ ΡΠ»Π΅Π΄ΡƒΡŽΡ‰ΠΈΠΌ ΠΎΠ±Ρ€Π°Π·ΠΎΠΌ

    . (3)

    4. ПодСлим ΠΏΠΎΡ‡Π»Π΅Π½Π½ΠΎ послСдниС Π΄Π²Π° уравнСния систСмы (3) Π΄Ρ€ΡƒΠ³ Π½Π° Π΄Ρ€ΡƒΠ³Π°

    ,

    . (4)

    5. ΠžΠΏΡ€Π΅Π΄Π΅Π»ΠΈΠΌ ΠΈΠ· уравнСния (4) Π·Π½Π°Ρ‡Π΅Π½ΠΈΠ΅ ο₯3

    ,

    . (5)

    2.4.4. Π”Π²Π΅ аккумуляторныС Π±Π°Ρ‚Π°Ρ€Π΅ΠΈ (ο₯1 = 8 Π’, r1 = 2 Ом; ο₯2 = 6 Π’οƒ—, r2 = 1,5 Ом) Π²ΠΊΠ»ΡŽΡ‡Π΅Π½Ρ‹ ΠΏΠ°Ρ€Π°Π»Π»Π΅Π»ΡŒΠ½ΠΎ ΠΈ согласно. ΠŸΠ°Ρ€Π°Π»Π»Π΅Π»ΡŒΠ½ΠΎ источникам Ρ‚ΠΎΠΊΠ° подсоСдинСно сопротивлСниС R = 10 Ом. ΠžΠΏΡ€Π΅Π΄Π΅Π»ΠΈΡ‚ΡŒ силу Ρ‚ΠΎΠΊΠ° Ρ‚Π΅ΠΊΡƒΡ‰Π΅Π³ΠΎ Ρ‡Π΅Ρ€Π΅Π· сопротивлСниС.

    РСшСниС

    1. Π’Ρ‹Π±Π΅Ρ€Π΅ΠΌ ΡƒΠ·Π΅Π», для ΠΊΠΎΡ‚ΠΎΡ€ΠΎΠ³ΠΎ запишСм ΡƒΡ€Π°Π²Π½Π΅Π½ΠΈΠ΅ ΠΏΠ΅Ρ€Π²ΠΎΠ³ΠΎ ΠΏΡ€Π°Π²ΠΈΠ»Π° ΠšΠΈΡ€Ρ…Π³ΠΎΡ„Π°

    . (1)

    2. Π’Ρ‹Π΄Π΅Π»ΠΈΠΌ Π΄Π²Π° ΠΊΠΎΠ½Ρ‚ΡƒΡ€Π°, ΠΏΠΎΠΊΠ°Π·Π°Π½Π½Ρ‹Π΅ Π½Π° схСмС ΠΏΡƒΠ½ΠΊΡ‚ΠΈΡ€Π½Ρ‹ΠΌΠΈ линиями ΠΈ составим для Π½ΠΈΡ… уравнСния Π²Ρ‚ΠΎΡ€ΠΎΠ³ΠΎ ΠΏΡ€Π°Π²ΠΈΠ»Π° ΠšΠΈΡ€Ρ…Π³ΠΎΡ„Π°

    . (2)

    3. Из ΡƒΡ€Π°Π²Π½Π΅Π½ΠΈΠΉ (2) Π²Ρ‹Ρ€Π°Π·ΠΈΠΌ Ρ‚ΠΎΠΊΠΈ I1 ΠΈ I2 ΠΈ подставим ΠΏΠΎΠ»ΡƒΡ‡Π΅Π½Π½Ρ‹Π΅ значСния Π² ΡƒΡ€Π°Π²Π½Π΅Π½ΠΈΠ΅ (1)

    , (3)

    , (4)

    . (5)

    4. ΠžΠΏΡ€Π΅Π΄Π΅Π»ΠΈΠΌ ΠΈΠ· уравнСния (5) силу Ρ‚ΠΎΠΊΠ°, ΠΏΡ€ΠΎΡ‚Π΅ΠΊΠ°ΡŽΡ‰Π΅Π³ΠΎ Ρ‡Π΅Ρ€Π΅Π· сопротивлСниС R

    , (6)

    . (7)

    5. ΠžΠΏΡ€Π΅Π΄Π΅Π»ΠΈΠΌ Π΄Π°Π»Π΅Π΅ Ρ‚ΠΎΠΊΠΈ Ρ‡Π΅Ρ€Π΅Π· источники Ρ‚ΠΎΠΊΠ°

    . (8)

    Π—Π½Π°ΠΊ «минус» ΠΏΠΎΠΊΠ°Π·Ρ‹Π²Π°Π΅Ρ‚, Ρ‡Ρ‚ΠΎ Π½Π°ΠΏΡ€Π°Π²Π»Π΅Π½ΠΈΠ΅ Ρ‚ΠΎΠΊΠ° I1 Π²Ρ‹Π±Ρ€Π°Π½ΠΎ Π½Π΅Π²Π΅Ρ€Π½ΠΎ.

    2.4.5. ΠžΠΏΡ€Π΅Π΄Π΅Π»ΠΈΡ‚ΡŒ силу Ρ‚ΠΎΠΊΠ° I3 Π² рСзисторС R3 ΠΈ ΠΏΠ°Π΄Π΅Π½ΠΈΠ΅ напряТСния U3, Ссли: ο₯1 = 4 Π’, ο₯2 = 3 Π’, R1 = 2 Ом, R2 = 6 Ом, R3 = 1 Ом. Π˜ΡΡ‚ΠΎΡ‡Π½ΠΈΠΊΠΈ ΡΡ‡ΠΈΡ‚Π°Ρ‚ΡŒ ΠΈΠ΄Π΅Π°Π»ΡŒΠ½Ρ‹ΠΌΠΈ, ΠΈΡ… Π²Π½ΡƒΡ‚Ρ€Π΅Π½Π½ΠΈΠΌ сопротивлСниСм ΠΏΡ€Π΅Π½Π΅Π±Ρ€Π΅Ρ‡ΡŒ.

    РСшСниС

    1. Π—Π°ΠΏΠΈΡˆΠ΅ΠΌ Ρ‚Ρ€ΠΈ уравнСния Π² соотвСтствии с ΠΏΡ€Π°Π²ΠΈΠ»Π°ΠΌΠΈ ΠšΠΈΡ€Ρ…Π³ΠΎΡ„Π°

    . (1)

    2. Π’Ρ‹Ρ€Π°Π·ΠΈΠΌ ΠΈΠ· ΠΏΠ΅Ρ€Π²ΠΎΠ³ΠΎ уравнСния систСмы (1) силу Ρ‚ΠΎΠΊΠ° I1

    , (2)

    ΠΈ подставим ΠΏΠΎΠ»ΡƒΡ‡Π΅Π½Π½ΠΎΠ΅ Π·Π½Π°Ρ‡Π΅Π½ΠΈΠ΅ Π²ΠΎ Π²Ρ‚ΠΎΡ€ΠΎΠ΅ ΡƒΡ€Π°Π²Π½Π΅Π½ΠΈΠ΅

    , (3)

    . (4)

    3. Π Π°Π·Ρ€Π΅ΡˆΠΈΠΌ Ρ‚Ρ€Π΅Ρ‚ΡŒΠ΅ ΡƒΡ€Π°Π²Π½Π΅Π½ΠΈΠ΅ систСмы (1) ΠΎΡ‚Π½ΠΎΡΠΈΡ‚Π΅Π»ΡŒΠ½ΠΎ силы Ρ‚ΠΎΠΊΠ° I2

    . (5)

    4. ΠŸΠΎΠ΄ΡΡ‚Π°Π²ΠΈΠΌ Π·Π½Π°Ρ‡Π΅Π½ΠΈΠ΅ I2 ΠΈΠ· уравнСния (5) Π² ΡƒΡ€Π°Π²Π½Π΅Π½ΠΈΠ΅ (4)

    . (6)

    5. Π£Ρ€Π°Π²Π½Π΅Π½ΠΈΠ΅ (6) содСрТит ΠΎΠ΄Π½Ρƒ Π½Π΅ΠΈΠ·Π²Π΅ΡΡ‚Π½ΡƒΡŽ ΠΈΡΠΊΠΎΠΌΡƒΡŽ Π²Π΅Π»ΠΈΡ‡ΠΈΠ½Ρƒ I3

    . (7)

    . (8)

    Π’Π°ΠΊΠΈΠΌ ΠΎΠ±Ρ€Π°Π·ΠΎΠΌ, Ρ‚ΠΎΠΊ Ρ‡Π΅Ρ€Π΅Π· сопротивлСниС R3 Ρ€Π°Π²Π΅Π½ Π½ΡƒΠ»ΡŽ, это Π·Π½Π°Ρ‡ΠΈΡ‚, Ρ‡Ρ‚ΠΎ ΠΏΠ°Π΄Π΅Π½ΠΈΠ΅ напряТСния Π½Π° этом рСзисторС Ρ‚ΠΎΠΆΠ΅ Ρ€Π°Π²Π½ΠΎ Π½ΡƒΠ»ΡŽ.

    2.4.6. Π’Ρ€ΠΈ источника с Π­Π”Π‘ ο₯1 = 12 Π’, ο₯2 = 5 Π’ ΠΈ ο₯3 = 10 Π’ с ΠΎΠ΄ΠΈΠ½Π°ΠΊΠΎΠ²Ρ‹ΠΌ Π²Π½ΡƒΡ‚Ρ€Π΅Π½Π½ΠΈΠΌ сопротивлСниСм r = 1 Ом соСдинСны ΠΌΠ΅ΠΆΠ΄Ρƒ собой ΠΎΠ΄Π½ΠΎΠΈΠΌΡ‘Π½Π½Ρ‹ΠΌΠΈ полюсами. ΠŸΡ€Π΅Π½Π΅Π±Ρ€Π΅Π³Π°Ρ сопротивлСниСм ΡΠΎΠ΅Π΄ΠΈΠ½ΠΈΡ‚Π΅Π»ΡŒΠ½Ρ‹Ρ… ΠΏΡ€ΠΎΠ²ΠΎΠ΄ΠΎΠ², ΠΎΠΏΡ€Π΅Π΄Π΅Π»ΠΈΡ‚ΡŒ силы Ρ‚ΠΎΠΊΠΎΠ², ΠΏΡ€ΠΎΡ‚Π΅ΠΊΠ°ΡŽΡ‰ΠΈΡ… Ρ‡Π΅Ρ€Π΅Π· источники.

    РСшСниС

    1. Π’Ρ‹Π±Π΅Ρ€Π΅ΠΌ ΠΎΠ΄ΠΈΠ½ ΠΈΠ· ΡƒΠ·Π»ΠΎΠ² ΠΈ Π²Ρ‹Π΄Π΅Π»ΠΈΠΌ Π΄Π²Π° Π·Π°ΠΌΠΊΠ½ΡƒΡ‚Ρ‹Ρ… ΠΊΠΎΠ½Ρ‚ΡƒΡ€Π°, для ΠΊΠΎΡ‚ΠΎΡ€Ρ‹Ρ… запишСм Ρ‚Ρ€ΠΈ уравнСния ΠΏΠ΅Ρ€Π²ΠΎΠ³ΠΎ ΠΈ Π²Ρ‚ΠΎΡ€ΠΎΠ³ΠΎ ΠΏΡ€Π°Π²ΠΈΠ»Π° ΠšΠΈΡ€Ρ…Π³ΠΎΡ„Π°

    . (1)

    2. ΠŸΠΎΠ΄ΡΡ‚Π°Π²ΠΈΠΌ Π² послСдниС Π΄Π²Π° уравнСния систСмы (1) Π·Π°Π΄Π°Π½Π½Ρ‹Π΅ числовыС значСния ΠΈ свСдём Π΅Ρ‘ ΠΊ Π²ΠΈΠ΄Ρƒ

    . (2)

    3. Π’Ρ‹Ρ€Π°Π·ΠΈΠΌ значСния сил Ρ‚ΠΎΠΊΠΎΠ² I1 ΠΈ I3

    , (3)

    ΠΈ подставим эти значСния Π² ΠΏΠ΅Ρ€Π²ΠΎΠ΅ ΡƒΡ€Π°Π²Π½Π΅Π½ΠΈΠ΅ систСмы (2)

    , (4)

    ΡΠ»Π΅Π΄ΠΎΠ²Π°Ρ‚Π΅Π»ΡŒΠ½ΠΎ

    . (5)

    2.4.7. Для Π·Π°Π΄Π°Π½Π½ΠΎΠΉ Ρ†Π΅ΠΏΠΈ ΠΎΠΏΡ€Π΅Π΄Π΅Π»ΠΈΡ‚ΡŒ Π²Π΅Π»ΠΈΡ‡ΠΈΠ½Ρ‹ сил Ρ‚ΠΎΠΊΠΎΠ² Ρ‡Π΅Ρ€Π΅Π· рСзисторы, Ссли извСстно, Ρ‡Ρ‚ΠΎ: ο₯1 = ο₯2 = 4 Π’; ο₯3 = 2 Π’; R1 = 1 Ом; R2 = 4 Ом; R3 = 2 Ом. Π’Π½ΡƒΡ‚Ρ€Π΅Π½Π½ΠΈΠΌ сопротивлСниСм источников Ρ‚ΠΎΠΊΠ° ΠΈ сопротивлСниСм ΡΠΎΠ΅Π΄ΠΈΠ½ΠΈΡ‚Π΅Π»ΡŒΠ½Ρ‹Ρ… ΠΏΡ€ΠΎΠ²ΠΎΠ΄ΠΎΠ² ΠΏΡ€Π΅Π½Π΅Π±Ρ€Π΅Ρ‡ΡŒ.

    РСшСниС

    1. Π—Π°ΠΏΠΈΡˆΠ΅ΠΌ для Π΄Π°Π½Π½ΠΎΠΉ Ρ†Π΅ΠΏΠΈ уравнСния ΠšΠΈΡ€Ρ…Π³ΠΎΡ„Π°, рассматривая баланс Ρ‚ΠΎΠΊΠΎΠ² Π² ΡƒΠ·Π»Π΅ А ΠΈ баланс напряТСний для Π²Ρ‹Π±Ρ€Π°Π½Π½Ρ‹Ρ… ΠΊΠΎΠ½Ρ‚ΡƒΡ€ΠΎΠ²

    (1)

    2. ΠŸΠΎΠ΄ΡΡ‚Π°Π²ΠΈΠΌ числСнныС значСния Π·Π°Π΄Π°Π½Π½Ρ‹Ρ… ΠΏΠΎ ΡƒΡΠ»ΠΎΠ²ΠΈΡŽ Π·Π°Π΄Π°Ρ‡ΠΈ Π²Π΅Π»ΠΈΡ‡ΠΈΠ½

    (2)

    3. Π’Ρ‹Ρ€Π°Π·ΠΈΠΌ ΠΈΠ· ΠΏΠ΅Ρ€Π²ΠΎΠ³ΠΎ уравнСния систСмы (2) силу Ρ‚ΠΎΠΊΠ° I3 ΠΈ подставим это Π·Π½Π°Ρ‡Π΅Π½ΠΈΠ΅ Π² Ρ‚Ρ€Π΅Ρ‚ΡŒΠ΅ ΡƒΡ€Π°Π²Π½Π΅Π½ΠΈΠ΅

    , (3)

    , (4)

    . (5)

    4. ΠžΠ±Ρ€Π°Π·ΡƒΠ΅ΠΌ Π½ΠΎΠ²ΡƒΡŽ систСму алгСбраичСских ΡƒΡ€Π°Π²Π½Π΅Π½ΠΈΠΉ ΠΈΠ· Π²Ρ‚ΠΎΡ€ΠΎΠ³ΠΎ уравнСния систСмы (2) ΠΈ уравнСния (5)

    . (6)

    5. Π’Ρ‹Ρ€Π°Π·ΠΈΠΌ Π΄Π°Π»Π΅Π΅ ΠΈΠ· Π²Ρ‚ΠΎΡ€ΠΎΠ³ΠΎ уравнСния систСмы (6) силу Ρ‚ΠΎΠΊ I1 ΠΈ подставим Π² ΠΏΠ΅Ρ€Π²ΠΎΠ΅ ΡƒΡ€Π°Π²Π½Π΅Π½ΠΈΠ΅

    . (7)

    6. ΠžΠΏΡ€Π΅Π΄Π΅Π»ΠΈΠΌ ΠΎΡΡ‚Π°Π»ΡŒΠ½Ρ‹Π΅ Π΄Π²Π΅ силы Ρ‚ΠΎΠΊΠ°, воспользовавшись Ρ€Π°Π½Π΅Π΅ записанными ΡΠΎΠΎΡ‚Π½ΠΎΡˆΠ΅Π½ΠΈΡΠΌΠΈ ΠΌΠ΅ΠΆΠ΄Ρƒ Π½ΠΈΠΌΠΈ

    . (8)

    . (9)

    2.4.8.ΠžΠΏΡ€Π΅Π΄Π΅Π»ΠΈΡ‚ΡŒ силы Ρ‚ΠΎΠΊΠΎΠ², Ρ‚Π΅ΠΊΡƒΡ‰ΠΈΡ… Π² ΠΊΠ°ΠΆΠ΄ΠΎΠΉ Π²Π΅Ρ‚Π²ΠΈ Ρ†Π΅ΠΏΠΈ, Ссли: ο₯1 = 6,5 Π’, ο₯2 = 3,9 Π’; R1 = R2 = R3 = R4 = R5 = 10 Ом.

    РСшСниС

    1. Для опрСдСлСния искомых Π²Π΅Π»ΠΈΡ‡ΠΈΠ½ Ρ‚ΠΎΠΊΠΎΠ² Π½Π΅ΠΎΠ±Ρ…ΠΎΠ΄ΠΈΠΌΠΎ ΡΠΎΡΡ‚Π°Π²ΠΈΡ‚ΡŒ ΡˆΠ΅ΡΡ‚ΡŒ ΡƒΡ€Π°Π²Π½Π΅Π½ΠΈΠΉ: Ρ‚Ρ€ΠΈ уравнСния баланса Ρ‚ΠΎΠΊΠΎΠ² ΠΈ Ρ‚Ρ€ΠΈ уравнСния баланса напряТСний. Π’Ρ‹Π±Π΅Ρ€Π΅ΠΌ для баланса Ρ‚ΠΎΠΊΠΎΠ² Ρ‚Ρ€ΠΈ ΡƒΠ·Π»Π°, Π° для баланса напряТСний Π²Ρ‹Π΄Π΅Π»ΠΈΠΌ Ρ‚Ρ€ΠΈ Π·Π°ΠΌΠΊΠ½ΡƒΡ‚Ρ‹Ρ… ΠΊΠΎΠ½Ρ‚ΡƒΡ€Π°.

    2. Боставим уравнСния баланса Ρ‚ΠΎΠΊΠΎΠ² для ΡƒΠ·Π»ΠΎΠ² a, b ΠΈ с

    . (1)

    3. Для ΠΎΠ±ΠΎΠ·Π½Π°Ρ‡Π΅Π½Π½Ρ‹Ρ… Π½Π° схСмС Ρ†Π΅ΠΏΠΈ ΠΏΡƒΠ½ΠΊΡ‚ΠΈΡ€Π½Ρ‹ΠΌΠΈ линиями Π·Π°ΠΌΠΊΠ½ΡƒΡ‚Ρ‹Ρ… ΠΊΠΎΠ½Ρ‚ΡƒΡ€ΠΎΠ² 1, 2 ΠΈ 3 составим уравнСния баланса напряТСний, Π½Π°ΠΏΡ€Π°Π²Π»Π΅Π½ΠΈΠ΅ ΠΎΠ±Ρ…ΠΎΠ΄Π° ΠΏΠΎΠΊΠ°Π·Π°Π½Ρ‹ стрСлками

    . (2)

    4. Π‘ ΡƒΡ‡Ρ‘Ρ‚ΠΎΠΌ ΠΎΠ΄ΠΈΠ½Π°ΠΊΠΎΠ²ΠΎΠΉ Π²Π΅Π»ΠΈΡ‡ΠΈΠ½Ρ‹ всСх сопротивлСний R = 10 Ом послСднюю систСму ΡƒΡ€Π°Π²Π½Π΅Π½ΠΈΠΉ ΠΌΠΎΠΆΠ½ΠΎ ΠΏΠ΅Ρ€Π΅ΠΏΠΈΡΠ°Ρ‚ΡŒ ΡΠ»Π΅Π΄ΡƒΡŽΡ‰ΠΈΠΌ ΠΎΠ±Ρ€Π°Π·ΠΎΠΌ

    (3)

    5. БовмСстноС Ρ€Π΅ΡˆΠ΅Π½ΠΈΠ΅ систСмы алгСбраичСских ΡƒΡ€Π°Π²Π½Π΅Π½ΠΈΠΉ (4)

    (4)

    ΠΌΠ΅Ρ‚ΠΎΠ΄ΠΎΠΌ подстановки позволяСт ΠΏΡ€ΠΈΠΉΡ‚ΠΈ ΠΊ ΡΠ»Π΅Π΄ΡƒΡŽΡ‰ΠΈΠΌ значСниям сил Ρ‚ΠΎΠΊΠΎΠ²

    (5)

    ΠžΡ‚Ρ€ΠΈΡ†Π°Ρ‚Π΅Π»ΡŒΠ½Ρ‹Π΅ значСния сил Ρ‚ΠΎΠΊΠΎΠ², ΠΏΠΎΠ»ΡƒΡ‡Π΅Π½Π½Ρ‹Π΅ Π² Ρ€Π΅Π·ΡƒΠ»ΡŒΡ‚Π°Ρ‚Π΅ Ρ€Π΅ΡˆΠ΅Π½ΠΈΡ, ΠΏΠΎΠΊΠ°Π·Ρ‹Π²Π°ΡŽΡ‚, Ρ‡Ρ‚ΠΎ ΠΈΡ… Π½Π°ΠΏΡ€Π°Π²Π»Π΅Π½ΠΈΠ΅ Π±Ρ‹Π»ΠΎ ΠΈΠ·Π½Π°Ρ‡Π°Π»ΡŒΠ½ΠΎ Π²Ρ‹Π±Ρ€Π°Π½ΠΎ Π½Π΅Π²Π΅Ρ€Π½ΠΎ ΠΈ слСдуСт ΠΏΠΎΠΌΠ΅Π½ΡΡ‚ΡŒ Π½Π° ΠΎΠ±Ρ€Π°Ρ‚Π½ΠΎΠ΅.

    2.5. НСлинСйныС элСмСнты Π² цСпях постоянного Ρ‚ΠΎΠΊΠ°

    2.5.1. ΠžΠΏΡ€Π΅Π΄Π΅Π»ΠΈΡ‚ΡŒ Π²Π΅Π»ΠΈΡ‡ΠΈΠ½Ρƒ силы Ρ‚ΠΎΠΊΠ° Ρ‡Π΅Ρ€Π΅Π· ΠΈΠ΄Π΅Π°Π»ΡŒΠ½Ρ‹ΠΉ источник (r = 0, ο₯ = 10 Π’) ΠΏΡ€ΠΈ Π²ΠΊΠ»ΡŽΡ‡Π΅Π½ΠΈΠΈ Π΅Π³ΠΎ Π² схСму двумя способами, Ссли R1 = R2 = R3 = R4 = 10 Ом, Π° Π΄ΠΈΠΎΠ΄ ΠΈΠ΄Π΅Π°Π»ΡŒΠ½Ρ‹ΠΉ, Ρ‚.Π΅. ΠΎΠ±Π»Π°Π΄Π°Π΅Ρ‚ Π² прямом Π½Π°ΠΏΡ€Π°Π²Π»Π΅Π½ΠΈΠΈ Π½ΡƒΠ»Π΅Π²Ρ‹ΠΌ сопротивлСниСм, Π° Π² ΠΎΠ±Ρ€Π°Ρ‚Π½ΠΎΠΌ Π½Π°ΠΏΡ€Π°Π²Π»Π΅Π½ΠΈΠΈ бСсконСчно большим сопротивлСниСм.

    РСшСниС

    1. Π’ ΠΏΠ΅Ρ€Π²ΠΎΠΌ случаС (лСвая схСма) Π΄ΠΈΠΎΠ΄ Π±ΡƒΠ΄Π΅Ρ‚ ΠΏΡ€Π΅Π΄ΡΡ‚Π°Π²Π»ΡΡ‚ΡŒ собой бСсконСчно большоС сопротивлСниС, Ρ‚.Π΅., ΠΏΠΎ сути, Ρ€Π°Π·Ρ€Ρ‹Π² Ρ†Π΅ΠΏΠΈ. Π’ΠΎ Π²Ρ‚ΠΎΡ€ΠΎΠΌ случаС (правая схСма) сопротивлСниС Π΄ΠΈΠΎΠ΄Π° Π±ΡƒΠ΄Π΅Ρ‚ ΠΌΠ°Π»ΠΎ. Π’Π°ΠΊΠΈΠΌ ΠΎΠ±Ρ€Π°Π·ΠΎΠΌ эквивалСнтныС схСмы Ρ†Π΅ΠΏΠ΅ΠΉ ΠΌΠΎΠΆΠ½ΠΎ ΠΏΡ€Π΅ΠΎΠ±Ρ€Π°Π·ΠΎΠ²Π°Ρ‚ΡŒ ΡΠ»Π΅Π΄ΡƒΡŽΡ‰ΠΈΠΌ ΠΎΠ±Ρ€Π°Π·ΠΎΠΌ.

    2. Π’ случаС большого сопротивлСния Ρ†Π΅ΠΏΠΈ рСзисторы R3 ΠΈ R4 ΠΎΠΊΠ°Π·Ρ‹Π²Π°ΡŽΡ‚ΡΡ Π²ΠΊΠ»ΡŽΡ‡Π΅Π½Π½Ρ‹ΠΌΠΈ ΠΏΠΎΡΠ»Π΅Π΄ΠΎΠ²Π°Ρ‚Π΅Π»ΡŒΠ½ΠΎ, ΠΈΡ… ΠΎΠ±Ρ‰Π΅Π΅ сопротивлСниС ο€­ R3,4 = 20 Ом, ΠΊΠΎΡ‚ΠΎΡ€ΠΎΠ΅, Π² свою ΠΎΡ‡Π΅Ρ€Π΅Π΄ΡŒ Π²ΠΊΠ»ΡŽΡ‡Π΅Π½ΠΎ ΠΏΠ°Ρ€Π°Π»Π»Π΅Π»ΡŒΠ½ΠΎ рСзистору R2

    . (1)

    3. ΠžΠΏΡ€Π΅Π΄Π΅Π»ΠΈΠΌ эквивалСнтноС сопротивлСниС ΠΏΡ€Π°Π²ΠΎΠΉ Ρ†Π΅ΠΏΠΈ

    . (2)

    4. Π‘ΠΈΠ»Π° Ρ‚ΠΎΠΊΠ° Π² ΠΏΠ΅Ρ€Π²ΠΎΠΌ случаС Π²ΠΊΠ»ΡŽΡ‡Π΅Π½ΠΈΡ источника Ρ‚ΠΎΠΊΠ°

    . (3)

    5. ΠŸΡ€ΠΈ ΠΎΡ‚ΠΊΡ€Ρ‹Ρ‚ΠΎΠΌ Π΄ΠΈΠΎΠ΄Π΅, ΠΊΠΎΠ³Π΄Π° ΠΎΠ½ ΠΎΠ±Π»Π°Π΄Π°Π΅Ρ‚ вСсьма ΠΌΠ°Π»Ρ‹ΠΌ сопротивлСниСм схСму Ρ‚ΠΎΠΆΠ΅ ΠΌΠΎΠΆΠ½ΠΎ ΠΏΠΎΡΠ»Π΅Π΄ΠΎΠ²Π°Ρ‚Π΅Π»ΡŒΠ½ΠΎ ΠΏΡ€Π΅ΠΎΠ±Ρ€Π°Π·ΠΎΠ²Π°Ρ‚ΡŒ, ΠΏΡ€ΠΈ этом

    , (4)

    , (5)

    . (6)

    6. Π‘ΠΈΠ»Π° Ρ‚ΠΎΠΊΠ° ΠΏΡ€ΠΈ ΠΎΡ‚ΠΊΡ€Ρ‹Ρ‚ΠΎΠΌ Π΄ΠΈΠΎΠ΄Π΅ составит

    . (7)

    2.5.2. ΠžΠΏΡ€Π΅Π΄Π΅Π»ΠΈΡ‚ΡŒ силу Ρ‚ΠΎΠΊΠ°, ΠΏΡ€ΠΎΡ‚Π΅ΠΊΠ°ΡŽΡ‰Π΅Π³ΠΎ Ρ‡Π΅Ρ€Π΅Π· ΠΈΠ΄Π΅Π°Π»ΡŒΠ½Ρ‹ΠΉ Π΄ΠΈΠΎΠ΄, Ссли ΠΎΠ½ Π²ΠΊΠ»ΡŽΡ‡Π΅Π½ Π² диагональ симмСтричного моста, составлСнного ΠΈΠ· сопротивлСний R1 = 10 кОм, R2 = 15 кОм, R3 = 30 кОм R4 = 25 кОм. ΠœΠΎΡΡ‚ΠΈΠΊ ΠΏΠΎΠ΄ΠΊΠ»ΡŽΡ‡Π΅Π½ ΠΊ ΠΈΠ΄Π΅Π°Π»ΡŒΠ½ΠΎΠΌΡƒ источнику Ρ‚ΠΎΠΊΠ° с ο₯ = 200 B.

    РСшСниС

    1. ΠŸΡ€Π΅Π΄ΠΏΠΎΠ»ΠΎΠΆΠΈΠΌ, Ρ‡Ρ‚ΠΎ Π΄ΠΈΠΎΠ΄ Π·Π°ΠΏΠ΅Ρ€Ρ‚, Ρ‚.Π΅. ΠΌΠ΅ΠΆΠ΄Ρƒ Ρ‚ΠΎΡ‡ΠΊΠ°ΠΌΠΈ Π° ΠΈ b бСсконСчно большоС сопротивлСниС. Π’ этом случаС ΠΎΠ±Ρ‰Π΅Π΅ сопротивлСниС схСмы опрСдСлится ΡƒΡ€Π°Π²Π½Π΅Π½ΠΈΠ΅ΠΌ

    . (1)

    2. Π‘ΠΈΠ»Π° Ρ‚ΠΎΠΊΠ° Ρ‡Π΅Ρ€Π΅Π· источник опрСдСлится ΠΊΠ°ΠΊ

    . (2)

    3. ЭквивалСнтная схСма Ρ†Π΅ΠΏΠΈ Π² этом случаС ΠΌΠΎΠΆΠ΅Ρ‚ Π±Ρ‹Ρ‚ΡŒ прСдставлСна Π² Π²ΠΈΠ΄Π΅ ΠΏΠΎΡΠ»Π΅Π΄ΠΎΠ²Π°Ρ‚Π΅Π»ΡŒΠ½ΠΎΠ³ΠΎ соСдинСния сопротивлСний R1,4 ΠΈ R2,3, ΠΊΠΎΡ‚ΠΎΡ€Ρ‹Π΅, Π² свою ΠΎΡ‡Π΅Ρ€Π΅Π΄ΡŒ, Π²ΠΊΠ»ΡŽΡ‡Π΅Π½Ρ‹ ΠΏΠ°Ρ€Π°Π»Π»Π΅Π»ΡŒΠ½ΠΎ источнику Ρ‚ΠΎΠΊΠ°

    , (1)

    . (2)

    4. ПадСниС напряТСния Π½Π° элСмСнтах эквивалСнтной схСмы

    , (3)

    , (4)

    , (5)

    . (6)

    5. Π Π°Π·Π½ΠΎΡΡ‚ΡŒ ΠΏΠΎΡ‚Π΅Π½Ρ†ΠΈΠ°Π»ΠΎΠ² ΠΌΠ΅ΠΆΠ΄Ρƒ Ρ‚ΠΎΡ‡ΠΊΠ°ΠΌΠΈ Π²ΠΊΠ»ΡŽΡ‡Π΅Π½ΠΈΡ Π΄ΠΈΠΎΠ΄Π° составляСт U = 12 Π’, ΠΏΡ€ΠΈ Ρ‚Π°ΠΊΠΎΠΉ полярности Π² ΡƒΠ·Π»ΠΎΠ²Ρ‹Ρ… Ρ‚ΠΎΡ‡ΠΊΠ°Ρ… Π΄ΠΈΠΎΠ΄ Π΄ΠΎΠ»ΠΆΠ΅Π½ Π±Ρ‹Ρ‚ΡŒ ΠΎΡ‚ΠΊΡ€Ρ‹Ρ‚ ΠΈ Π΄ΠΎΠ»ΠΆΠ΅Π½ ΠΏΡ€Π΅Π΄ΡΡ‚Π°Π²Π»ΡΡ‚ΡŒ собой вСсьма ΠΌΠ°Π»ΠΎΠ΅ сопротивлСниС. Π”Ρ€ΡƒΠ³ΠΈΠΌΠΈ словами эквивалСнтная схСма Ρ†Π΅ΠΏΠΈ Π±ΡƒΠ΄Π΅Ρ‚ ΠΏΡ€Π΅Π΄ΡΡ‚Π°Π²Π»ΡΡ‚ΡŒ собой ΠΏΠ°Ρ€Π°Π»Π»Π΅Π»ΡŒΠ½ΠΎΠ΅ Π²ΠΊΠ»ΡŽΡ‡Π΅Π½ΠΈΠ΅ сопротивлСний R1, R2 ΠΈ R3,R4, ΠΊΠΎΡ‚ΠΎΡ€Ρ‹Π΅ ΠΎΠ±Ρ€Π°Π·ΡƒΡŽΡ‚ ΠΏΠΎΡΠ»Π΅Π΄ΠΎΠ²Π°Ρ‚Π΅Π»ΡŒΠ½ΡƒΡŽ Ρ†Π΅ΠΏΡŒ. ΠžΠ±Ρ‰Π΅Π΅ сопротивлСниС Ρ†Π΅ΠΏΠΈ Π² этом случаС опрСдСлится ΠΊΠ°ΠΊ

    . (7)

    6. Π‘ΠΈΠ»Π° Ρ‚ΠΎΠΊΠ° Ρ‡Π΅Ρ€Π΅Π· источник

    . (8)

    7. Боставим систСму ΡƒΡ€Π°Π²Π½Π΅Π½ΠΈΠΉ ΠšΠΈΡ€Ρ…Π³ΠΎΡ„Π° для баланса Ρ‚ΠΎΠΊΠΎΠ² Π² ΡƒΠ·Π»Π°Ρ… a ΠΈ b, Π΄ΠΎΠΏΠΎΠ»Π½ΠΈΠ² ΠΈΡ… двумя уравнСниями Π·Π°ΠΊΠΎΠ½Π° Ома для участка Ρ†Π΅ΠΏΠΈ

    (9)

    8. ΠŸΠΎΠ΄ΡΡ‚Π°Π²ΠΈΠ² Π² уравнСния (5) ΠΈ (6) Π·Π°Π΄Π°Π½Π½Ρ‹Π΅ значСния сопротивлСний, ΠΏΡ€Π΅ΠΎΠ±Ρ€Π°Π·ΡƒΠ΅ΠΌ ΠΈΡ… ΠΊ Π²ΠΈΠ΄Ρƒ

    . (10)

    9. ΠŸΠΎΠ΄ΡΡ‚Π°Π²ΠΈΠΌ Π·Π½Π°Ρ‡Π΅Π½ΠΈΠ΅ силы Ρ‚ΠΎΠΊΠ° I1 ΠΈΠ· уравнСния (10) Π² ΡƒΡ€Π°Π²Π½Π΅Π½ΠΈΠ΅ (1) систСмы (9)

    . (11)

    10. Π‘ΠΈΠ»Π° Ρ‚ΠΎΠΊΠ° I1 ΠΈΠ· ΡƒΡ€Π°Π²Π½Π΅Π½ΠΈΠΉ (10) опрСдСлится ΠΊΠ°ΠΊ

    . (12)

    11. Π”Π°Π»Π΅Π΅ подставим Π·Π½Π°Ρ‡Π΅Π½ΠΈΠ΅ силы Ρ‚ΠΎΠΊΠ° I2 ΠΈΠ· уравнСния (10) Π² ΡƒΡ€Π°Π²Π½Π΅Π½ΠΈΠ΅ (4) систСмы (9)

    . (13)

    12. ΠžΠΏΡ€Π΅Π΄Π΅Π»ΠΈΠΌ Π΄Π°Π»Π΅Π΅ силу Ρ‚ΠΎΠΊΠ° I2, воспользовавшись для этого уравнСниями (10)

    . (14)

    13. Из уравнСния (4) систСмы (9) Π½Π°ΠΉΠ΄Ρ‘ΠΌ ΠΈΡΠΊΠΎΠΌΡƒΡŽ Π²Π΅Π»ΠΈΡ‡ΠΈΠ½Ρƒ силы Ρ‚ΠΎΠΊΠ° Ρ‡Π΅Ρ€Π΅Π· Π΄ΠΈΠΎΠ΄

    . (15)

    2.5.3. ЀотоэлСмСнт Π²ΠΊΠ»ΡŽΡ‡Ρ‘Π½ Π² диагональ моста, составлСнного ΠΈΠ· Ρ‡Π΅Ρ‚Ρ‹Ρ€Ρ‘Ρ… рСзисторов R1 = 100 кОм, R2 = 400 кОм, R3 = 200 кОм, R4 = 300 кОм. Π˜Π΄Π΅Π°Π»ΡŒΠ½Ρ‹ΠΉ источник Ρ‚ΠΎΠΊΠ° с Π­Π”Π‘ ο₯ = 1 ΠΊΠ’ Π²ΠΊΠ»ΡŽΡ‡Π΅Π½ Π² Π΄Ρ€ΡƒΠ³ΡƒΡŽ диагональ моста. ΠžΠΏΡ€Π΅Π΄Π΅Π»ΠΈΡ‚ΡŒ напряТСниС Π½Π° фотоэлСмСнтС, Ссли Ρ‡Π΅Ρ€Π΅Π· Π½Π΅Π³ΠΎ Ρ‚Π΅Ρ‡Ρ‘Ρ‚ Ρ‚ΠΎΠΊ силой ID = 10 мА.

    РСшСниС

    1.ΠŸΠΎΡΠΊΠΎΠ»ΡŒΠΊΡƒ Ρ‡Π΅Ρ€Π΅Π· фотоэлСмСнт ΠΎΡ‚ Π°Π½ΠΎΠ΄Π° ΠΊ ΠΊΠ°Ρ‚ΠΎΠ΄Ρƒ Ρ‚Π΅Ρ‡Ρ‘Ρ‚, Π·Π°Π΄Π°Π½Π½Ρ‹ΠΉ ΠΏΠΎ ΡƒΡΠ»ΠΎΠ²ΠΈΡŽ Π·Π°Π΄Π°Ρ‡ΠΈ Ρ‚ΠΎΠΊ силой ID = 10 мА, Ρ‚ΠΎ ΠΎΠ½ ΠΎΡ‚ΠΊΡ€Ρ‹Ρ‚ ΠΈ прСдставляСт собой ΠΌΠ°Π»ΠΎΠ΅ сопротивлСниС. ЭквивалСнтная схСма Ρ†Π΅ΠΏΠΈ Π² этом случаС ΠΌΠΎΠΆΠ΅Ρ‚ Π±Ρ‹Ρ‚ΡŒ прСдставлСна Π² Π²ΠΈΠ΄Π΅ ΠΏΠ°Ρ€Π°Π»Π»Π΅Π»ΡŒΠ½ΠΎΠ³ΠΎ Π²ΠΊΠ»ΡŽΡ‡Π΅Π½ΠΈΡ сопротивлСний R1, R2, ΠΈ R3, R4, ΠΊΠΎΡ‚ΠΎΡ€Ρ‹Π΅ Π² свою ΠΎΡ‡Π΅Ρ€Π΅Π΄ΡŒ соСдинСны ΠΏΠΎΡΠ»Π΅Π΄ΠΎΠ²Π°Ρ‚Π΅Π»ΡŒΠ½ΠΎ.

    2. ΠžΠΏΡ€Π΅Π΄Π΅Π»ΠΈΠΌ эквивалСнтноС сопротивлСниС всСй Ρ†Π΅ΠΏΠΈ

    . (1)

    3. Найдём Π²Π΅Π»ΠΈΡ‡ΠΈΠ½Ρƒ силы Ρ‚ΠΎΠΊΠ° Ρ‡Π΅Ρ€Π΅Π· источник I0

    . (2)

    4. Боставим систСму ΠΈΠ· пяти (ΠΏΠΎ количСству нСизвСстных Π²Π΅Π»ΠΈΡ‡ΠΈΠ½) алгСбраичСских ΡƒΡ€Π°Π²Π½Π΅Π½ΠΈΠΉ Π½Π° основС ΠΏΠ΅Ρ€Π²ΠΎΠ³ΠΎ ΠΏΡ€Π°Π²ΠΈΠ»Π° ΠšΠΈΡ€Ρ…Π³ΠΎΡ„Π° ΠΈ условий равСнства ΠΏΠΎΡ‚Π΅Π½Ρ†ΠΈΠ°Π»ΠΎΠ² ΡƒΠ·Π»ΠΎΠ² a ΠΈ b

    (3)

    5. Π—Π°ΠΏΠΈΡˆΠ΅ΠΌ уравнСния (4), (5) с ΡƒΡ‡Ρ‘Ρ‚ΠΎΠΌ Π·Π°Π΄Π°Π½Π½Ρ‹Ρ… Π²Π΅Π»ΠΈΡ‡ΠΈΠ½ рСзисторов

    . (4)

    6. ΠŸΠ΅Ρ€Π΅ΠΏΠΈΡˆΠ΅ΠΌ ΡƒΡ€Π°Π²Π½Π΅Π½ΠΈΠ΅ (2) систСмы (3) с ΡƒΡ‡Ρ‘Ρ‚ΠΎΠΌ ΡƒΡ€Π°Π²Π½Π΅Π½ΠΈΠΉ (4)

    . (5)

    7. ΠžΠΏΡ€Π΅Π΄Π΅Π»ΠΈΠΌ ΠΈΠ· уравнСния (2) систСмы (3) Π·Π½Π°Ρ‡Π΅Π½ΠΈΠ΅ силы Ρ‚ΠΎΠΊΠ° I1

    . (6)

    8. Найдём ΠΏΠ°Π΄Π΅Π½ΠΈΠ΅ напряТСний Π½Π° рСзисторах R1 ΠΈ R2

    . (7)

    . (8)

    9. НапряТСниС Π½Π° фотоэлСмСнтС: .

    Богласно Π·Π°ΠΊΠΎΠ½Ρƒ Ома, напряТСниС V, Ρ‚ΠΎΠΊ I ΠΈ сопротивлСниС R Π² Ρ†Π΅ΠΏΠΈ связаны ΡƒΡ€Π°Π²Π½Π΅Π½ΠΈΠ΅ΠΌ V = IR, Π³Π΄Π΅ Π΅Π΄ΠΈΠ½ΠΈΡ†Ρ‹ измСрСния — Π²ΠΎΠ»ΡŒΡ‚Ρ‹, Π°ΠΌΠΏΠ΅Ρ€Ρ‹ ΠΈ ΠΎΠΌΡ‹. ΠŸΡ€Π΅Π΄ΠΏΠΎΠ»ΠΎΠΆΠΈΠΌ, Ρ‡Ρ‚ΠΎ напряТСниС являСтся постоянным ΠΈ составляСт V = 18 Π’. РассчитайтС ΡΡ€Π΅Π΄Π½ΡŽΡŽ ΡΠΊΠΎΡ€ΠΎΡΡ‚ΡŒ измСнСния I

    .


    Π˜ΡΡ…ΠΎΠ΄Ρ ΠΈΠ· Π·Π°ΠΊΠΎΠ½Π° Ома,

    $$ V = ИК, $$

    ΠΌΡ‹ ΠΌΠΎΠΆΠ΅ΠΌ Π²Ρ‹Ρ€Π°Π·ΠΈΡ‚ΡŒ Ρ‚ΠΎΠΊ ΠΊΠ°ΠΊ Ρ„ΡƒΠ½ΠΊΡ†ΠΈΡŽ напряТСния ΠΈ сопротивлСния,

    $$ I = \ dfrac {V} {R} $$

    БрСдняя ΡΠΊΠΎΡ€ΠΎΡΡ‚ΡŒ измСнСния интСнсивности ΠΏΠΎ ΠΎΡ‚Π½ΠΎΡˆΠ΅Π½ΠΈΡŽ ΠΊ ΡΠΎΠΏΡ€ΠΎΡ‚ΠΈΠ²Π»Π΅Π½ΠΈΡŽ опрСдСляСтся ΠΏΠΎ Ρ„ΠΎΡ€ΠΌΡƒΠ»Π΅, ΠΏΡ€ΠΈΠ²Π΅Π΄Π΅Π½Π½ΠΎΠΉ Π½ΠΈΠΆΠ΅.

    $$ \ eta = \ dfrac {\ Delta I} {\ Delta R} = \ dfrac {I (R + \ Delta R) -I (R)} {\ Delta R} $$

    ΠŸΠΎΠ΄ΡΡ‚Π°Π²Π»ΡΡ Ρ„ΡƒΠ½ΠΊΡ†ΠΈΡŽ,

    $$ \ begin {align} \ bar {\ eta} & = \ dfrac {V / (R + \ Delta R) -V / R} {\ Delta R} \\ [0,3 см] & = V \ dfrac {R-R- \ Delta R} {R (R + \ Delta R) \ Delta R} \\ [0,3 см] & = — \ dfrac {V \ Delta R} {R (R + \ Delta R) \ Delta R} \\ [0,3 см] & = — \ dfrac {V} {R (R + \ Delta R)} \ end {align} \\ $$

    ΠžΡ†Π΅Π½ΠΊΠ° Ρ€Π΅Π·ΡƒΠ»ΡŒΡ‚Π°Ρ‚Π°, ΠΊΠΎΠ³Π΄Π° {eq} V = 18 \; \ rm V {/ eq}, Π° сопротивлСниС измСнится с {eq} 8.0 \; \ rm \ ОмСга {/ eq} ΠΊ {eq} 8.1 \; \ rm \ Omega {/ eq}, Ρ‚.Π΅. {eq} R = 8.0 \; \ rm \ Omega {/ eq} ΠΈ {eq} \ Delta R = 0.1 \; \ rm Omega {/ eq},

    $$ \ eta = — \ dfrac {18 \; \ rm V} {8.0 \; \ rm \ Omega \ cdot (8.0 \; \ rm \ Omega +0.1 \; \ rm Omega)} \ ΠΏΡ€ΠΈΠ±Π»ΠΈΠ·ΠΈΡ‚Π΅Π»ΡŒΠ½ΠΎ \ Π² ΡˆΡ‚ΡƒΡ‡Π½ΠΎΠΉ ΡƒΠΏΠ°ΠΊΠΎΠ²ΠΊΠ΅ {- 0,28 \; \ rm A / \ Omega} $$

    ΠžΠ±Ρ€Π°Ρ‚ΠΈΡ‚Π΅ Π²Π½ΠΈΠΌΠ°Π½ΠΈΠ΅, Ρ‡Ρ‚ΠΎ ΡΠΊΠΎΡ€ΠΎΡΡ‚ΡŒ измСнСния выраТаСтся Π² {eq} \ rm A / \ Omega {/ eq} — Π΅Π΄ΠΈΠ½ΠΈΡ†Ρ‹ Ρ‚ΠΎΠΊΠ°, Ρ€Π°Π·Π΄Π΅Π»Π΅Π½Π½Ρ‹Π΅ Π½Π° Π΅Π΄ΠΈΠ½ΠΈΡ†Ρ‹ сопротивлСния. ΠžΡ‚Ρ€ΠΈΡ†Π°Ρ‚Π΅Π»ΡŒΠ½Π°Ρ срСдняя ΡΠΊΠΎΡ€ΠΎΡΡ‚ΡŒ измСнСния ΠΎΠ·Π½Π°Ρ‡Π°Π΅Ρ‚, Ρ‡Ρ‚ΠΎ Ρ‚ΠΎΠΊ ΡƒΠΌΠ΅Π½ΡŒΡˆΠΈΡ‚ΡΡ Π² срСднСм Π½Π° {eq} 0.2} \ ΠΏΡ€ΠΈΠ±Π»ΠΈΠ·ΠΈΡ‚Π΅Π»ΡŒΠ½ΠΎ \ Π² ΡˆΡ‚ΡƒΡ‡Π½ΠΎΠΉ ΡƒΠΏΠ°ΠΊΠΎΠ²ΠΊΠ΅ {-0,28 \; \ rm A / \ Omega} $$

    МгновСнная ΡΠΊΠΎΡ€ΠΎΡΡ‚ΡŒ измСнСния интСнсивности составляСт {eq} 0,28 \; \ rm A / \ Omega {/ экв}. ΠžΠΏΡΡ‚ΡŒ ΠΆΠ΅, Π·Π½Π°ΠΊ минус Π² скорости измСнСния ΡƒΠΊΠ°Π·Ρ‹Π²Π°Π΅Ρ‚ Π½Π° ΡƒΠΌΠ΅Π½ΡŒΡˆΠ΅Π½ΠΈΠ΅ Ρ‚ΠΎΠΊΠ° ΠΏΡ€ΠΈ ΡƒΠ²Π΅Π»ΠΈΡ‡Π΅Π½ΠΈΠΈ сопротивлСния.

    Π§Ρ‚ΠΎ Ρ‚Π°ΠΊΠΎΠ΅ Π·Π°ΠΊΠΎΠ½ Ома ΠΈ ΠΎ Ρ‡Π΅ΠΌ ΠΎΠ½ Π½Π°ΠΌ Π³ΠΎΠ²ΠΎΡ€ΠΈΡ‚?

    Π—Π°ΠΊΠΎΠ½ Ома гласит, Ρ‡Ρ‚ΠΎ элСктричСский Ρ‚ΠΎΠΊ, проходящий Ρ‡Π΅Ρ€Π΅Π· ΠΏΡ€ΠΎΠ²ΠΎΠ΄Π½ΠΈΠΊ, прямо ΠΏΡ€ΠΎΠΏΠΎΡ€Ρ†ΠΈΠΎΠ½Π°Π»Π΅Π½ разности ΠΏΠΎΡ‚Π΅Π½Ρ†ΠΈΠ°Π»ΠΎΠ² Π½Π° Π½Π΅ΠΌ. Π”Ρ€ΡƒΠ³ΠΈΠΌΠΈ словами, постоянная ΠΏΡ€ΠΎΠΏΠΎΡ€Ρ†ΠΈΠΎΠ½Π°Π»ΡŒΠ½ΠΎΡΡ‚ΡŒ ΠΏΡ€ΠΈΠ²ΠΎΠ΄ΠΈΡ‚ ΠΊ ΡΠΎΠΏΡ€ΠΎΡ‚ΠΈΠ²Π»Π΅Π½ΠΈΡŽ ΠΏΡ€ΠΎΠ²ΠΎΠ΄Π½ΠΈΠΊΠ°.Π—Π°ΠΊΠΎΠ½ Ома гласит, Ρ‡Ρ‚ΠΎ постоянный Ρ‚ΠΎΠΊ, ΠΏΡ€ΠΎΡ‚Π΅ΠΊΠ°ΡŽΡ‰ΠΈΠΉ Π² ΠΏΡ€ΠΎΠ²ΠΎΠ΄Π½ΠΈΠΊΠ΅, Ρ‚Π°ΠΊΠΆΠ΅ прямо ΠΏΡ€ΠΎΠΏΠΎΡ€Ρ†ΠΈΠΎΠ½Π°Π»Π΅Π½ Ρ€Π°Π·Π½ΠΈΡ†Π΅ ΠΌΠ΅ΠΆΠ΄Ρƒ Π΅Π³ΠΎ ΠΊΠΎΠ½Ρ†Π°ΠΌΠΈ. Π—Π°ΠΊΠΎΠ½ Ома сформулирован ΠΊΠ°ΠΊ V = IR, Π³Π΄Π΅ V — напряТСниС, I — Ρ‚ΠΎΠΊ, Π° R — сопротивлСниС ΠΏΡ€ΠΎΠ²ΠΎΠ΄Π½ΠΈΠΊΠ°. Π—Π°ΠΊΠΎΠ½ Ома прСдставляСт собой ваТнСйшСС матСматичСскоС ΡΠΎΠΎΡ‚Π½ΠΎΡˆΠ΅Π½ΠΈΠ΅ ΠΌΠ΅ΠΆΠ΄Ρƒ напряТСниСм, сопротивлСниСм ΠΈ Ρ‚ΠΎΠΊΠΎΠΌ.

    Π’ΠΎΠΊ

    Богласно Π·Π°ΠΊΠΎΠ½Ρƒ Ома, Ρ‚ΠΎΠΊ Ρ‚Π΅Ρ‡Π΅Ρ‚ ΠΏΠΎ ΠΏΡ€ΠΎΠ²ΠΎΠ΄Π½ΠΈΠΊΡƒ, ΠΊΠ°ΠΊ Π²ΠΎΠ΄Π° Ρ‚Π΅Ρ‡Π΅Ρ‚ ΠΏΠΎ Ρ€Π΅ΠΊΠ΅. По повСрхности ΠΏΡ€ΠΎΠ²ΠΎΠ΄Π½ΠΈΠΊΠ° Ρ‚ΠΎΠΊ Ρ‚Π΅Ρ‡Π΅Ρ‚ ΠΎΡ‚ ΠΎΡ‚Ρ€ΠΈΡ†Π°Ρ‚Π΅Π»ΡŒΠ½ΠΎΠ³ΠΎ ΠΊ ΠΏΠΎΠ»ΠΎΠΆΠΈΡ‚Π΅Π»ΡŒΠ½ΠΎΠΌΡƒ.ЭлСктричСский Ρ‚ΠΎΠΊ, содСрТащийся Π² Ρ†Π΅ΠΏΠΈ, ΠΌΠΎΠΆΠ½ΠΎ Ρ€Π°ΡΡΡ‡ΠΈΡ‚Π°Ρ‚ΡŒ, Ρ€Π°Π·Π΄Π΅Π»ΠΈΠ² напряТСниС Π½Π° сопротивлСниС. Π’ΠΎΠΊ ΠΏΡ€ΠΎΠΏΠΎΡ€Ρ†ΠΈΠΎΠ½Π°Π»Π΅Π½ Π½Π°ΠΏΡ€ΡΠΆΠ΅Π½ΠΈΡŽ ΠΈ ΠΎΠ±Ρ€Π°Ρ‚Π½ΠΎ ΠΏΡ€ΠΎΠΏΠΎΡ€Ρ†ΠΈΠΎΠ½Π°Π»Π΅Π½ ΡΠΎΠΏΡ€ΠΎΡ‚ΠΈΠ²Π»Π΅Π½ΠΈΡŽ. Π’Π°ΠΊΠΈΠΌ ΠΎΠ±Ρ€Π°Π·ΠΎΠΌ, ΡƒΠ²Π΅Π»ΠΈΡ‡Π΅Π½ΠΈΠ΅ напряТСния ΠΏΡ€ΠΈΠ²Π΅Π΄Π΅Ρ‚ ΠΊ ΡƒΠ²Π΅Π»ΠΈΡ‡Π΅Π½ΠΈΡŽ Ρ‚ΠΎΠΊΠ°. Π­Ρ‚ΠΎ ΠΌΠΎΠΆΠ΅Ρ‚ ΠΏΡ€ΠΎΠΈΠ·ΠΎΠΉΡ‚ΠΈ, Ρ‚ΠΎΠ»ΡŒΠΊΠΎ Ссли сопротивлСниС остаСтся постоянным. Если сопротивлСниС увСличиваСтся, Π° напряТСниС Π½Π΅Ρ‚, Ρ‚ΠΎΠΊ Π±ΡƒΠ΄Π΅Ρ‚ ΡƒΠΌΠ΅Π½ΡŒΡˆΠ°Ρ‚ΡŒΡΡ.

    НапряТСниС

    НапряТСниС ΠΌΠΎΠΆΠ½ΠΎ ΠΎΠΏΠΈΡΠ°Ρ‚ΡŒ ΠΊΠ°ΠΊ Ρ€Π°Π·Π½ΠΎΡΡ‚ΡŒ элСктричСских ΠΏΠΎΡ‚Π΅Π½Ρ†ΠΈΠ°Π»ΠΎΠ² ΠΌΠ΅ΠΆΠ΄Ρƒ двумя Ρ‚ΠΎΡ‡ΠΊΠ°ΠΌΠΈ Π² Ρ†Π΅ΠΏΠΈ.Π’Ρ‹ ΠΌΠΎΠΆΠ΅Ρ‚Π΅ Ρ€Π°ΡΡΡ‡ΠΈΡ‚Π°Ρ‚ΡŒ напряТСниС, Ссли извСстны Ρ‚ΠΎΠΊ ΠΈ сопротивлСниС Π² Ρ†Π΅ΠΏΠΈ. Если Ρ‚ΠΎΠΊ ΠΈΠ»ΠΈ сопротивлСниС приводят ΠΊ ΡƒΠ²Π΅Π»ΠΈΡ‡Π΅Π½ΠΈΡŽ Π² Ρ†Π΅ΠΏΠΈ, напряТСниС автоматичСски увСличиваСтся.

    Π‘ΠΎΠΏΡ€ΠΎΡ‚ΠΈΠ²Π»Π΅Π½ΠΈΠ΅

    Π‘ΠΎΠΏΡ€ΠΎΡ‚ΠΈΠ²Π»Π΅Π½ΠΈΠ΅ опрСдСляСт, сколько Ρ‚ΠΎΠΊΠ° Π±ΡƒΠ΄Π΅Ρ‚ ΠΏΡ€ΠΎΡ…ΠΎΠ΄ΠΈΡ‚ΡŒ Ρ‡Π΅Ρ€Π΅Π· ΠΊΠΎΠΌΠΏΠΎΠ½Π΅Π½Ρ‚. РСзисторы ΠΌΠΎΠΆΠ½ΠΎ ΠΈΡΠΏΠΎΠ»ΡŒΠ·ΠΎΠ²Π°Ρ‚ΡŒ для управлСния уровнями Ρ‚ΠΎΠΊΠ° ΠΈ напряТСния. ВысокоС сопротивлСниС пропускаСт Ρ‚ΠΎΠ»ΡŒΠΊΠΎ нСбольшоС количСство Ρ‚ΠΎΠΊΠ°. И Π½Π°ΠΎΠ±ΠΎΡ€ΠΎΡ‚, ΠΎΡ‡Π΅Π½ΡŒ Π½ΠΈΠ·ΠΊΠΎΠ΅ сопротивлСниС ΠΏΠΎΠ·Π²ΠΎΠ»ΠΈΡ‚ ΠΏΡ€ΠΎΠΏΡƒΡΠΊΠ°Ρ‚ΡŒ большоС количСство Ρ‚ΠΎΠΊΠ°.Π‘ΠΎΠΏΡ€ΠΎΡ‚ΠΈΠ²Π»Π΅Π½ΠΈΠ΅ измСряСтся Π² Ом.

    ΠœΠΎΡ‰Π½ΠΎΡΡ‚ΡŒ

    Богласно Π·Π°ΠΊΠΎΠ½Ρƒ Ома ΠΌΠΎΡ‰Π½ΠΎΡΡ‚ΡŒ — это количСство Ρ‚ΠΎΠΊΠ°, ΡƒΠΌΠ½ΠΎΠΆΠ΅Π½Π½ΠΎΠ΅ Π½Π° ΡƒΡ€ΠΎΠ²Π΅Π½ΡŒ напряТСния Π² Π΄Π°Π½Π½ΠΎΠΉ Ρ‚ΠΎΡ‡ΠΊΠ΅. ΠœΠΎΡ‰Π½ΠΎΡΡ‚ΡŒ измСряСтся Π² Π²Π°Ρ‚Ρ‚Π°Ρ… ΠΈΠ»ΠΈ Π²Π°Ρ‚Ρ‚Π°Ρ….

    Π—Π°ΠΊΠΎΠ½ Ом

    • Π˜Π·ΡƒΡ‡ΠΈΠ² этот Ρ€Π°Π·Π΄Π΅Π», Π²Ρ‹ Π΄ΠΎΠ»ΠΆΠ½Ρ‹ ΡƒΠΌΠ΅Ρ‚ΡŒ:
    • ΠžΠΏΠΈΡˆΠΈΡ‚Π΅ Π·Π°ΠΊΠΎΠ½ Ома для мСталличСских ΠΏΡ€ΠΎΠ²ΠΎΠ΄Π½ΠΈΠΊΠΎΠ²:
    • β€’ Π‘ΠΎΠΏΡ€ΠΎΡ‚ΠΈΠ²Π»Π΅Π½ΠΈΠ΅, напряТСниС ΠΈ Ρ‚ΠΎΠΊ.
    • ΠžΠΏΡ€Π΅Π΄Π΅Π»ΠΈΡ‚ΡŒ:
    • Ом, АмпСр ΠΈ Π’ΠΎΠ»ΡŒΡ‚.

    Ом, Π²ΠΎΠ»ΡŒΡ‚ ΠΈ Π°ΠΌΠΏΠ΅Ρ€.

    Π‘ΠΎΠΏΡ€ΠΎΡ‚ΠΈΠ²Π»Π΅Π½ΠΈΠ΅ ΠΏΡ€ΠΎΠ²ΠΎΠ΄Π½ΠΈΠΊΠ° измСряСтся Π² ΠžΠΌΠ°Ρ…, Π° Ом — это Π΅Π΄ΠΈΠ½ΠΈΡ†Π° измСрСния, названная Π² Ρ‡Π΅ΡΡ‚ΡŒ Π½Π΅ΠΌΠ΅Ρ†ΠΊΠΎΠ³ΠΎ Ρ„ΠΈΠ·ΠΈΠΊΠ° Π”ΠΆΠΎΡ€Π΄ΠΆΠ° Π‘ΠΈΠΌΠΎΠ½Π° Ома (1787–1854 Π³Π³.), ΠšΠΎΡ‚ΠΎΡ€Ρ‹ΠΉ ΠΏΠ΅Ρ€Π²Ρ‹ΠΌ ΠΏΠΎΠΊΠ°Π·Π°Π» взаимосвязь ΠΌΠ΅ΠΆΠ΄Ρƒ сопротивлСниСм, Ρ‚ΠΎΠΊΠΎΠΌ ΠΈ напряТСниСм. ΠŸΡ€ΠΈ этом ΠΎΠ½ Ρ€Π°Π·Ρ€Π°Π±ΠΎΡ‚Π°Π» свой Π·Π°ΠΊΠΎΠ½, ΠΊΠΎΡ‚ΠΎΡ€Ρ‹ΠΉ ΠΏΠΎΠΊΠ°Π·Ρ‹Π²Π°Π΅Ρ‚ взаимосвязь ΠΌΠ΅ΠΆΠ΄Ρƒ трСмя основными элСктричСскими свойствами сопротивлСния, напряТСния ΠΈ Ρ‚ΠΎΠΊΠ°.Он дСмонстрируСт ΠΎΠ΄Π½Ρƒ ΠΈΠ· самых Π²Π°ΠΆΠ½Ρ‹Ρ… взаимосвязСй Π² элСктротСхникС ΠΈ элСктронной Ρ‚Π΅Ρ…Π½ΠΈΠΊΠ΅.

    Π—Π°ΠΊΠΎΠ½ Ома гласит: Β«Π’ мСталличСских ΠΏΡ€ΠΎΠ²ΠΎΠ΄Π½ΠΈΠΊΠ°Ρ… ΠΏΡ€ΠΈ постоянной Ρ‚Π΅ΠΌΠΏΠ΅Ρ€Π°Ρ‚ΡƒΡ€Π΅ ΠΈ Π½ΡƒΠ»Π΅Π²ΠΎΠΌ ΠΌΠ°Π³Π½ΠΈΡ‚Π½ΠΎΠΌ ΠΏΠΎΠ»Π΅ ΠΏΡ€ΠΎΡ‚Π΅ΠΊΠ°ΡŽΡ‰ΠΈΠΉ Ρ‚ΠΎΠΊ ΠΏΡ€ΠΎΠΏΠΎΡ€Ρ†ΠΈΠΎΠ½Π°Π»Π΅Π½ Π½Π°ΠΏΡ€ΡΠΆΠ΅Π½ΠΈΡŽ Π½Π° ΠΊΠΎΠ½Ρ†Π°Ρ… ΠΏΡ€ΠΎΠ²ΠΎΠ΄Π½ΠΈΠΊΠ° ΠΈ ΠΎΠ±Ρ€Π°Ρ‚Π½ΠΎ ΠΏΡ€ΠΎΠΏΠΎΡ€Ρ†ΠΈΠΎΠ½Π°Π»Π΅Π½ ΡΠΎΠΏΡ€ΠΎΡ‚ΠΈΠ²Π»Π΅Π½ΠΈΡŽ ΠΏΡ€ΠΎΠ²ΠΎΠ΄Π½ΠΈΠΊΠ°. »

    ΠŸΡ€ΠΎΡ‰Π΅ говоря, ΠΏΡ€ΠΈ условии, Ρ‡Ρ‚ΠΎ Ρ‚Π΅ΠΌΠΏΠ΅Ρ€Π°Ρ‚ΡƒΡ€Π° постоянна ΠΈ элСктричСская Ρ†Π΅ΠΏΡŒ Π½Π΅ ΠΏΠΎΠ΄Π²Π΅Ρ€ΠΆΠ΅Π½Π° влиянию ΠΌΠ°Π³Π½ΠΈΡ‚Π½Ρ‹Ρ… ΠΏΠΎΠ»Π΅ΠΉ, Ρ‚ΠΎΠ³Π΄Π°:

    β€’ Π’ Ρ†Π΅ΠΏΠΈ постоянного сопротивлСния, Ρ‡Π΅ΠΌ большС напряТСниС, ΠΏΡ€ΠΈΠ»ΠΎΠΆΠ΅Π½Π½ΠΎΠ΅ ΠΊ Ρ†Π΅ΠΏΠΈ, Ρ‚Π΅ΠΌ большС Π±ΡƒΠ΄Π΅Ρ‚ ΠΏΡ€ΠΎΡ‚Π΅ΠΊΠ°Ρ‚ΡŒ Ρ‚ΠΎΠΊ.

    β€’ ΠŸΡ€ΠΈ постоянном напряТСнии, Ρ‡Π΅ΠΌ большС сопротивлСниС Ρ†Π΅ΠΏΠΈ, Ρ‚Π΅ΠΌ мСньшС Ρ‚ΠΎΠΊ.

    ΠžΠ±Ρ€Π°Ρ‚ΠΈΡ‚Π΅ Π²Π½ΠΈΠΌΠ°Π½ΠΈΠ΅, Ρ‡Ρ‚ΠΎ Π·Π°ΠΊΠΎΠ½ Ома гласит: Β«Π’ мСталличСских ΠΏΡ€ΠΎΠ²ΠΎΠ΄Π½ΠΈΠΊΠ°Ρ…Β». Π­Ρ‚ΠΎ ΠΎΠ·Π½Π°Ρ‡Π°Π΅Ρ‚, Ρ‡Ρ‚ΠΎ Π·Π°ΠΊΠΎΠ½ ΠΏΡ€ΠΈΠΌΠ΅Π½ΠΈΠΌ для Π±ΠΎΠ»ΡŒΡˆΠΈΠ½ΡΡ‚Π²Π° мСталличСских ΠΌΠ°Ρ‚Π΅Ρ€ΠΈΠ°Π»ΠΎΠ², Π½ΠΎ Π½Π΅ для всСх. НапримСр, Π²ΠΎΠ»ΡŒΡ„Ρ€Π°ΠΌ, ΠΈΡΠΏΠΎΠ»ΡŒΠ·ΡƒΠ΅ΠΌΡ‹ΠΉ для накаливания Π½Π°ΠΊΠ°Π»Π° Π»Π°ΠΌΠΏΠΎΡ‡Π΅ΠΊ, ΠΈΠΌΠ΅Π΅Ρ‚ сопротивлСниС, ΠΊΠΎΡ‚ΠΎΡ€ΠΎΠ΅ измСняСтся Π² зависимости ΠΎΡ‚ Ρ‚Π΅ΠΌΠΏΠ΅Ρ€Π°Ρ‚ΡƒΡ€Ρ‹ Π½ΠΈΡ‚ΠΈ, ΠΎΡ‚ΡΡŽΠ΄Π° Π² Π·Π°ΠΊΠΎΠ½Π΅ Ома ссылка Π½Π° Β«ΠΏΡ€ΠΈ постоянной Ρ‚Π΅ΠΌΠΏΠ΅Ρ€Π°Ρ‚ΡƒΡ€Π΅Β». Π’ элСктроникС Ρ‚Π°ΠΊΠΆΠ΅ ΠΈΡΠΏΠΎΠ»ΡŒΠ·ΡƒΡŽΡ‚ΡΡ ΠΊΠΎΠΌΠΏΠΎΠ½Π΅Π½Ρ‚Ρ‹, ΠΊΠΎΡ‚ΠΎΡ€Ρ‹Π΅ ΠΈΠΌΠ΅ΡŽΡ‚ Π½Π΅Π»ΠΈΠ½Π΅ΠΉΠ½ΡƒΡŽ Π·Π°Π²ΠΈΡΠΈΠΌΠΎΡΡ‚ΡŒ ΠΌΠ΅ΠΆΠ΄Ρƒ трСмя элСктричСскими свойствами: напряТСниСм, Ρ‚ΠΎΠΊΠΎΠΌ ΠΈ сопротивлСниСм, Π½ΠΎ ΠΈΡ… ΠΌΠΎΠΆΠ½ΠΎ ΠΎΠΏΠΈΡΠ°Ρ‚ΡŒ Ρ€Π°Π·Π½Ρ‹ΠΌΠΈ Ρ„ΠΎΡ€ΠΌΡƒΠ»Π°ΠΌΠΈ.Для Π±ΠΎΠ»ΡŒΡˆΠΈΠ½ΡΡ‚Π²Π° схСм ΠΈΠ»ΠΈ ΠΊΠΎΠΌΠΏΠΎΠ½Π΅Π½Ρ‚ΠΎΠ², ΠΊΠΎΡ‚ΠΎΡ€Ρ‹Π΅ ΠΌΠΎΠ³ΡƒΡ‚ Π±Ρ‹Ρ‚ΡŒ описаны Π·Π°ΠΊΠΎΠ½ΠΎΠΌ Ома:

    ВмСсто Ρ‚ΠΎΠ³ΠΎ, Ρ‡Ρ‚ΠΎΠ±Ρ‹ ΠΏΡ‹Ρ‚Π°Ρ‚ΡŒΡΡ Π·Π°ΠΏΠΎΠΌΠ½ΠΈΡ‚ΡŒ вСсь Π·Π°ΠΊΠΎΠ½ Ома, Ρ‚Ρ€ΠΈ элСктричСских свойства напряТСния, Ρ‚ΠΎΠΊΠ° ΠΈ сопротивлСния ΠΎΡ‚Π΄Π΅Π»ΡŒΠ½Ρ‹ΠΌΠΈ Π±ΡƒΠΊΠ²Π°ΠΌΠΈ:

    Π‘ΠΎΠΏΡ€ΠΎΡ‚ΠΈΠ²Π»Π΅Π½ΠΈΠ΅ обозначаСтся Π±ΡƒΠΊΠ²ΠΎΠΉ R ΠΈ измСряСтся Π² Π΅Π΄ΠΈΠ½ΠΈΡ†Π°Ρ… Ом, ΠΊΠΎΡ‚ΠΎΡ€Ρ‹Π΅ ΠΈΠΌΠ΅ΡŽΡ‚ символ Ξ© (грСчСская заглавная Π±ΡƒΠΊΠ²Π° O).

    НапряТСниС обозначаСтся Π±ΡƒΠΊΠ²ΠΎΠΉ V (ΠΈΠ»ΠΈ ΠΈΠ½ΠΎΠ³Π΄Π° E, сокращСниСм ΠΎΡ‚ Electromotive Force) ΠΈ измСряСтся Π² Π²ΠΎΠ»ΡŒΡ‚Π°Ρ…, ΠΊΠΎΡ‚ΠΎΡ€Ρ‹Π΅ ΠΈΠΌΠ΅ΡŽΡ‚ символ V.

    Π’ΠΎΠΊ обозначаСтся Π±ΡƒΠΊΠ²ΠΎΠΉ I (Π½Π΅ C, ΠΏΠΎΡΠΊΠΎΠ»ΡŒΠΊΡƒ ΠΎΠ½ ΠΈΡΠΏΠΎΠ»ΡŒΠ·ΡƒΠ΅Ρ‚ΡΡ для обозначСния Смкости) ΠΈ измСряСтся Π² Π΅Π΄ΠΈΠ½ΠΈΡ†Π°Ρ… Π°ΠΌΠΏΠ΅Ρ€ (часто сокращаСтся Π΄ΠΎ Π°ΠΌΠΏΠ΅Ρ€), ΠΊΠΎΡ‚ΠΎΡ€Ρ‹Π΅ ΠΈΠΌΠ΅ΡŽΡ‚ символ A.

    Π˜ΡΠΏΠΎΠ»ΡŒΠ·ΡƒΡ Π±ΡƒΠΊΠ²Ρ‹ V, I ΠΈ R для выраТСния ΠΎΡ‚Π½ΠΎΡˆΠ΅Π½ΠΈΠΉ, ΠΎΠΏΡ€Π΅Π΄Π΅Π»Π΅Π½Π½Ρ‹Ρ… Π² Π—Π°ΠΊΠΎΠ½Π΅ Ома, Π΄Π°Π΅Ρ‚ Ρ‚Ρ€ΠΈ простыС Ρ„ΠΎΡ€ΠΌΡƒΠ»Ρ‹:

    ΠšΠ°ΠΆΠ΄Ρ‹ΠΉ ΠΈΠ· Π½ΠΈΡ… ΠΏΠΎΠΊΠ°Π·Ρ‹Π²Π°Π΅Ρ‚, ΠΊΠ°ΠΊ Π½Π°ΠΉΡ‚ΠΈ Π·Π½Π°Ρ‡Π΅Π½ΠΈΠ΅ любой ΠΈΠ· этих Π²Π΅Π»ΠΈΡ‡ΠΈΠ½ Π² Ρ†Π΅ΠΏΠΈ, Ссли извСстны Π΄Π²Π΅ Π΄Ρ€ΡƒΠ³ΠΈΠ΅. НапримСр, Ρ‡Ρ‚ΠΎΠ±Ρ‹ Π½Π°ΠΉΡ‚ΠΈ напряТСниС V (Π² Π²ΠΎΠ»ΡŒΡ‚Π°Ρ…) Π½Π° рСзисторС, просто ΡƒΠΌΠ½ΠΎΠΆΡŒΡ‚Π΅ Ρ‚ΠΎΠΊ I (Π² Π°ΠΌΠΏΠ΅Ρ€Π°Ρ…) Ρ‡Π΅Ρ€Π΅Π· рСзистор Π½Π° Π·Π½Π°Ρ‡Π΅Π½ΠΈΠ΅ рСзистора R (Π² ΠΎΠΌΠ°Ρ…).

    ΠžΠ±Ρ€Π°Ρ‚ΠΈΡ‚Π΅ Π²Π½ΠΈΠΌΠ°Π½ΠΈΠ΅, Ρ‡Ρ‚ΠΎ ΠΏΡ€ΠΈ использовании этих Ρ„ΠΎΡ€ΠΌΡƒΠ» значСния V I ΠΈ R, записанныС Π² Ρ„ΠΎΡ€ΠΌΡƒΠ»Ρƒ, Π΄ΠΎΠ»ΠΆΠ½Ρ‹ Π±Ρ‹Ρ‚ΡŒ Π² Π‘ΠΠ—ΠžΠ’Π«Π₯ Π•Π”Π˜ΠΠ˜Π¦ΠΠ₯, Ρ‚ΠΎ Π΅ΡΡ‚ΡŒ Π² Π’ΠžΠ›Π¬Π’ΠΠ₯ (Π½Π΅ Π² ΠΌΠΈΠ»Π»ΠΈΠ²ΠΎΠ»ΡŒΡ‚Π°Ρ…) Π² ΠžΠΌΠ°Ρ… (Π½Π΅ Π² ΠΊΠΈΠ»ΠΎΠΌΠ°Ρ…) ΠΈ Π² ΠΠœΠŸΠ•Π ΠΠ₯ (Π½Π΅ Π² ΠΌΠΈΠΊΡ€ΠΎΠ°ΠΌΠΏΠ΅Ρ€Π°Ρ…) ΠΈ Ρ‚. Π”.

    Π’ΠΊΡ€Π°Ρ‚Ρ†Π΅ 15 кОм (ΠΊΠΈΠ»ΠΎΠΎΠΌ) вводится ΠΊΠ°ΠΊ 15 EXP 03, Π° 25 мА (ΠΌΠΈΠ»Π»ΠΈΠ°ΠΌΠΏΠ΅Ρ€) вводится ΠΊΠ°ΠΊ 25 EXP -03 ΠΈ Ρ‚. Π”. Π­Ρ‚ΠΎ ΠΏΡ€ΠΎΡ‰Π΅ всСго ΡΠ΄Π΅Π»Π°Ρ‚ΡŒ с ΠΏΠΎΠΌΠΎΡ‰ΡŒΡŽ Π½Π°ΡƒΡ‡Π½ΠΎΠ³ΠΎ ΠΊΠ°Π»ΡŒΠΊΡƒΠ»ΡΡ‚ΠΎΡ€Π°.

    Как ΠΏΠΎΠ»ΡŒΠ·ΠΎΠ²Π°Ρ‚ΡŒΡΡ ΠΊΠ°Π»ΡŒΠΊΡƒΠ»ΡΡ‚ΠΎΡ€ΠΎΠΌ с ΠΈΠ½ΠΆΠ΅Π½Π΅Ρ€Π½Ρ‹ΠΌΠΈ обозначСниями, ΡˆΠΈΡ€ΠΎΠΊΠΎ ΠΈΡΠΏΠΎΠ»ΡŒΠ·ΡƒΠ΅ΠΌΡ‹ΠΌΠΈ Π² элСктроникС, ΠΎΠ±ΡŠΡΡΠ½ΡΠ΅Ρ‚ΡΡ Π² нашСм бСсплатном Π±ΡƒΠΊΠ»Π΅Ρ‚Π΅ ΠΏΠΎΠ΄ Π½Π°Π·Π²Π°Π½ΠΈΠ΅ΠΌ «Подсказки ΠΏΠΎ ΠΌΠ°Ρ‚Π΅ΠΌΠ°Ρ‚ΠΈΠΊΠ΅Β». Π—Π°Π³Ρ€ΡƒΠ·ΠΈΡ‚Π΅ Π΅Π³ΠΎ со страницы Π·Π°Π³Ρ€ΡƒΠ·ΠΊΠΈ.

    ΠžΠΏΡ€Π΅Π΄Π΅Π»Π΅Π½ΠΈΠ΅ Ом, АмпСра ΠΈ Π’ΠΎΠ»ΡŒΡ‚

    1 Ом

    ΠœΠΎΠΆΠ΅Ρ‚ Π±Ρ‹Ρ‚ΡŒ ΠΎΠΏΡ€Π΅Π΄Π΅Π»Π΅Π½ΠΎ ΠΊΠ°ΠΊ Β«Π²Π΅Π»ΠΈΡ‡ΠΈΠ½Π° сопротивлСния, которая создаСт Ρ€Π°Π·Π½ΠΎΡΡ‚ΡŒ ΠΏΠΎΡ‚Π΅Π½Ρ†ΠΈΠ°Π»ΠΎΠ² (p.d.) ΠΈΠ»ΠΈ напряТСниС Π² 1 Π²ΠΎΠ»ΡŒΡ‚ Π½Π° Π½Π΅ΠΌ, ΠΊΠΎΠ³Π΄Π° Ρ‡Π΅Ρ€Π΅Π· Π½Π΅Π³ΠΎ ΠΏΡ€ΠΎΡ‚Π΅ΠΊΠ°Π΅Ρ‚ Ρ‚ΠΎΠΊ Π² 1 Π°ΠΌΠΏΠ΅Ρ€Β».

    1 ΠΠœΠŸΠ•Π 

    МоТно ΠΎΠΏΡ€Π΅Π΄Π΅Π»ΠΈΡ‚ΡŒ ΠΊΠ°ΠΊ Β«Π’Π΅Π»ΠΈΡ‡ΠΈΠ½Π° Ρ‚ΠΎΠΊΠ°, которая ΠΏΡ€ΠΈ ΠΏΡ€ΠΎΡ…ΠΎΠΆΠ΄Π΅Π½ΠΈΠΈ Ρ‡Π΅Ρ€Π΅Π· сопротивлСниС 1 Ом создаСт Ρ€Π°Π·Π½ΠΎΡΡ‚ΡŒ ΠΏΠΎΡ‚Π΅Π½Ρ†ΠΈΠ°Π»ΠΎΠ² Π½Π° сопротивлСнии Π² 1 Π’ΠΎΠ»ΡŒΡ‚Β».

    (Π₯отя доступны Π±ΠΎΠ»Π΅Π΅ ΠΏΠΎΠ»Π΅Π·Π½Ρ‹Π΅ опрСдСлСния Π°ΠΌΠΏΠ΅Ρ€Π°)

    1 Π’ΠžΠ›Π¬Π’

    ΠœΠΎΠΆΠ΅Ρ‚ Π±Ρ‹Ρ‚ΡŒ ΠΎΠΏΡ€Π΅Π΄Π΅Π»Π΅Π½ΠΎ ΠΊΠ°ΠΊ Β«Π Π°Π·Π½ΠΎΡΡ‚ΡŒ ΠΏΠΎΡ‚Π΅Π½Ρ†ΠΈΠ°Π»ΠΎΠ² (напряТСний), Π²ΠΎΠ·Π½ΠΈΠΊΠ°ΡŽΡ‰Π°Ρ Π½Π° сопротивлСнии 1 Ом, Ρ‡Π΅Ρ€Π΅Π· ΠΊΠΎΡ‚ΠΎΡ€ΠΎΠ΅ ΠΏΡ€ΠΎΡ‚Π΅ΠΊΠ°Π΅Ρ‚ Ρ‚ΠΎΠΊ Π² 1 АмпСр.Β«

    Π­Ρ‚ΠΈ опрСдСлСния относятся ΠΊ Π’ΠΎΠ»ΡŒΡ‚Π°ΠΌ, АмпСрам ΠΈ Ом Π² ΠΏΡ€Π΅Π΄Π΅Π»Π°Ρ… Π²Π΅Π»ΠΈΡ‡ΠΈΠ½, описанных Π² Π—Π°ΠΊΠΎΠ½Π΅ Ома, Π½ΠΎ Ρ‚Π°ΠΊΠΆΠ΅ ΠΌΠΎΠ³ΡƒΡ‚ ΠΈΡΠΏΠΎΠ»ΡŒΠ·ΠΎΠ²Π°Ρ‚ΡŒΡΡ Π°Π»ΡŒΡ‚Π΅Ρ€Π½Π°Ρ‚ΠΈΠ²Π½Ρ‹Π΅ опрСдСлСния с использованиСм Π΄Ρ€ΡƒΠ³ΠΈΡ… Π²Π΅Π»ΠΈΡ‡ΠΈΠ½.

    ΠŸΠžΠŸΠ ΠžΠ‘Π£Π™Π’Π• ΠŸΠ ΠžΠ‘Π’Π«Π• РАБЧЕВЫ, Π˜Π‘ΠŸΠžΠ›Π¬Π—Π£Π― Π—Π°ΠΊΠΎΠ½ Ома.

    % PDF-1.5 % 1 0 obj> эндобдТ 2 0 obj> эндобдТ 3 0 obj> эндобдТ 4 0 obj> ΠΏΠΎΡ‚ΠΎΠΊ ΠΊΠΎΠ½Π΅Ρ‡Π½Ρ‹ΠΉ ΠΏΠΎΡ‚ΠΎΠΊ эндобдТ xref 0 5 0000000000 65535 Ρ„ 0000000016 00000 Π½. 0000000075 00000 ΠΏ 0000000120 00000 Π½. 0000000210 00000 Π½. Ρ‚Ρ€Π΅ΠΉΠ»Π΅Ρ€ ] >> startxref 3379 %% EOF 1 0 obj> эндобдТ 2 0 obj> эндобдТ 3 0 obj> эндобдТ 5 0 obj null эндобдТ 6 0 obj> эндобдТ 7 0 obj> эндобдТ 8 0 obj> эндобдТ 9 0 obj> / Font> / XObject> / ProcSet [/ PDF / Text / ImageB] / ExtGState >>> эндобдТ 10 0 obj> эндобдТ 11 0 obj> эндобдТ 12 0 obj> эндобдТ 13 0 obj> эндобдТ 14 0 obj> / Π¨ΠΈΡ€ΠΈΠ½Π° 661 / Высота 221 / BitsPerComponent 1 / ImageMask true / Type / XObject / Subtype / Image >> stream & Mβ€šCk

    КакоС ΡƒΡ‚Π²Π΅Ρ€ΠΆΠ΄Π΅Π½ΠΈΠ΅ Π²Π΅Ρ€Π½ΠΎ Π² соотвСтствии с Π·Π°ΠΊΠΎΠ½ΠΎΠΌ Ома A Π’Π΅ΠΊΡƒΡ‰ΠΈΠΉ класс Ρ„ΠΈΠ·ΠΈΠΊΠΈ 12 JEE_Main

    Подсказка: ΠœΡ‹ Π·Π½Π°Π΅ΠΌ, Ρ‡Ρ‚ΠΎ Π·Π°ΠΊΠΎΠ½ Ома гласит, Ρ‡Ρ‚ΠΎ напряТСниС Π½Π° ΠΏΡ€ΠΎΠ²ΠΎΠ΄Π½ΠΈΠΊΠ΅ с Ρ‚ΠΎΠΊΠΎΠΌ прямо ΠΏΡ€ΠΎΠΏΠΎΡ€Ρ†ΠΈΠΎΠ½Π°Π»ΡŒΠ½ΠΎ Ρ‚ΠΎΠΊΡƒ, ΠΏΡ€ΠΎΡ‚Π΅ΠΊΠ°ΡŽΡ‰Π΅ΠΌΡƒ Π² Ρ‚ΠΎΠΊΠ΅.Π˜Ρ‚Π°ΠΊ, матСматичСскоС ΡƒΡ€Π°Π²Π½Π΅Π½ΠΈΠ΅ Π·Π°ΠΊΠΎΠ½Π° Ома ΠΈΠΌΠ΅Π΅Ρ‚ Π²ΠΈΠ΄ $ V = I \ cdot R $, Π³Π΄Π΅ $ V $ — напряТСниС, $ I $ — Ρ‚ΠΎΠΊ, Π° $ R $ — сопротивлСниС ΠΏΡ€ΠΎΠ²ΠΎΠ΄Π½ΠΈΠΊΠ°, ΠΊΠΎΡ‚ΠΎΡ€ΠΎΠ΅ фиксировано для заданная Ρ‚Π΅ΠΌΠΏΠ΅Ρ€Π°Ρ‚ΡƒΡ€Π° ΠΏΡ€ΠΎΠ²ΠΎΠ΄Π½ΠΈΠΊΠ°. Π’Π΅ΠΏΠ΅Ρ€ΡŒ ΠΌΡ‹ ΠΌΠΎΠΆΠ΅ΠΌ ΠΏΠΎΠ»ΡƒΡ‡ΠΈΡ‚ΡŒ ΠΎΡ‚Π²Π΅Ρ‚, ΠΏΡ€ΠΎΠ°Π½Π°Π»ΠΈΠ·ΠΈΡ€ΠΎΠ²Π°Π² всС Π²Π°Ρ€ΠΈΠ°Π½Ρ‚Ρ‹.

    Π˜ΡΠΏΠΎΠ»ΡŒΠ·ΡƒΠ΅ΠΌΠ°Ρ Ρ„ΠΎΡ€ΠΌΡƒΠ»Π°:
    Π—Π°ΠΊΠΎΠ½ Ома гласит, Ρ‡Ρ‚ΠΎ $ V = I \ cdot R $, Π³Π΄Π΅ $ V $ — напряТСниС, $ I $ — Ρ‚ΠΎΠΊ, Π° $ R $ — сопротивлСниС ΠΏΡ€ΠΎΠ²ΠΎΠ΄Π½ΠΈΠΊΠ°.

    ПолноС пошаговоС Ρ€Π΅ΡˆΠ΅Π½ΠΈΠ΅:
    ΠœΡ‹ Π·Π½Π°Π΅ΠΌ, Ρ‡Ρ‚ΠΎ Π·Π°ΠΊΠΎΠ½ Ома гласит, Ρ‡Ρ‚ΠΎ напряТСниС ΠΏΡ€ΠΎΠΏΠΎΡ€Ρ†ΠΈΠΎΠ½Π°Π»ΡŒΠ½ΠΎ Ρ‚ΠΎΠΊΡƒ.Π‘Π»Π΅Π΄ΠΎΠ²Π°Ρ‚Π΅Π»ΡŒΠ½ΠΎ,
    $ V = I \ cdot R $,
    Π³Π΄Π΅ $ V $ — напряТСниС, $ I $ — Ρ‚ΠΎΠΊ, Π° $ R $ — сопротивлСниС ΠΏΡ€ΠΎΠ²ΠΎΠ΄Π½ΠΈΠΊΠ°, ΠΊΠΎΡ‚ΠΎΡ€ΠΎΠ΅ фиксировано для Π΄Π°Π½Π½ΠΎΠΉ Ρ‚Π΅ΠΌΠΏΠ΅Ρ€Π°Ρ‚ΡƒΡ€Ρ‹ ΠΏΡ€ΠΎΠ²ΠΎΠ΄Π½ΠΈΠΊΠ°. Π’Π΅ΠΏΠ΅Ρ€ΡŒ, ΠΏΡ€ΠΎΠ°Π½Π°Π»ΠΈΠ·ΠΈΡ€ΠΎΠ²Π°Π² ΠΏΠΎ ΠΎΡ‡Π΅Ρ€Π΅Π΄ΠΈ всС Π²Π°Ρ€ΠΈΠ°Π½Ρ‚Ρ‹, ΡƒΠΊΠ°Π·Π°Π½Π½Ρ‹Π΅ Π² вопросС, ΠΌΡ‹ ΠΏΠΎΠ»ΡƒΡ‡ΠΈΠΌ ΠΎΡ‚Π²Π΅Ρ‚.

    Π’Π°Ρ€ΠΈΠ°Π½Ρ‚ A: Π’Π΅ΠΏΠ΅Ρ€ΡŒ Π² этом Π²Π°Ρ€ΠΈΠ°Π½Ρ‚Π΅ указываСтся, Ρ‡Ρ‚ΠΎ Ρ‚ΠΎΠΊ Π±ΡƒΠ΄Π΅Ρ‚ ΡƒΠ²Π΅Π»ΠΈΡ‡ΠΈΠ²Π°Ρ‚ΡŒΡΡ Π·Π° счСт увСличСния напряТСния Π² Ρ†Π΅ΠΏΠΈ. Π’Π΅ΠΏΠ΅Ρ€ΡŒ, ΠΊΠ°ΠΊ ΠΌΡ‹ обсуТдали Π²Ρ‹ΡˆΠ΅, согласно Π·Π°ΠΊΠΎΠ½Ρƒ Ома $ I \ propto V $, это ΠΎΠ·Π½Π°Ρ‡Π°Π΅Ρ‚, Ρ‡Ρ‚ΠΎ Ссли Ρ‚ΠΎΠΊ Π±ΡƒΠ΄Π΅Ρ‚ ΡƒΠ²Π΅Π»ΠΈΡ‡ΠΈΠ²Π°Ρ‚ΡŒΡΡ, Ρ‚ΠΎ ΠΎΠ΄Π½ΠΎΠ²Ρ€Π΅ΠΌΠ΅Π½Π½ΠΎ Π±ΡƒΠ΄Π΅Ρ‚ ΡƒΠ²Π΅Π»ΠΈΡ‡ΠΈΠ²Π°Ρ‚ΡŒΡΡ ΠΈ напряТСниС.
    Π—Π½Π°Ρ‡ΠΈΡ‚, это ΠΏΡ€Π°Π²ΠΈΠ»ΡŒΠ½Ρ‹ΠΉ Π²Π°Ρ€ΠΈΠ°Π½Ρ‚.

    Π’Π°Ρ€ΠΈΠ°Π½Ρ‚ B: Π’Π΅ΠΏΠ΅Ρ€ΡŒ Π² этом Π²Π°Ρ€ΠΈΠ°Π½Ρ‚Π΅ ΡƒΠΊΠ°Π·Π°Π½ΠΎ, Ρ‡Ρ‚ΠΎ сопротивлСниС Π±ΡƒΠ΄Π΅Ρ‚ ΡƒΠ²Π΅Π»ΠΈΡ‡ΠΈΠ²Π°Ρ‚ΡŒΡΡ ΠΏΡ€ΠΈ ΡƒΠ²Π΅Π»ΠΈΡ‡Π΅Π½ΠΈΠΈ напряТСния Π² Ρ†Π΅ΠΏΠΈ. Как ΠΎΠ±ΡΡƒΠΆΠ΄Π°Π»ΠΎΡΡŒ Π²Ρ‹ΡˆΠ΅, сопротивлСниС ΠΏΡ€ΠΎΠ²ΠΎΠ΄Π½ΠΈΠΊΠ° фиксировано для Π΄Π°Π½Π½ΠΎΠΉ Ρ‚Π΅ΠΌΠΏΠ΅Ρ€Π°Ρ‚ΡƒΡ€Ρ‹. Π—Π½Π°Ρ‡ΠΈΡ‚, сопротивлСниС Π½Π΅ увСличится, Ссли ΠΌΡ‹ ΡƒΠ²Π΅Π»ΠΈΡ‡ΠΈΠΌ напряТСниС Ρ†Π΅ΠΏΠΈ.
    Π‘Π»Π΅Π΄ΠΎΠ²Π°Ρ‚Π΅Π»ΡŒΠ½ΠΎ, этот Π²Π°Ρ€ΠΈΠ°Π½Ρ‚ Π½Π΅Π²Π΅Ρ€Π΅Π½.

    Π’Π°Ρ€ΠΈΠ°Π½Ρ‚ C: Π’Π΅ΠΏΠ΅Ρ€ΡŒ эта опция ΡƒΠΊΠ°Π·Ρ‹Π²Π°Π΅Ρ‚, Ρ‡Ρ‚ΠΎ сопротивлСниС Π±ΡƒΠ΄Π΅Ρ‚ ΡƒΠ²Π΅Π»ΠΈΡ‡ΠΈΠ²Π°Ρ‚ΡŒΡΡ Π·Π° счСт увСличСния Ρ‚ΠΎΠΊΠ° Π² Ρ†Π΅ΠΏΠΈ. Как ΠΎΠ±ΡΡƒΠΆΠ΄Π°Π»ΠΎΡΡŒ Π²Ρ‹ΡˆΠ΅, сопротивлСниС ΠΏΡ€ΠΎΠ²ΠΎΠ΄Π½ΠΈΠΊΠ° фиксировано для Π΄Π°Π½Π½ΠΎΠΉ Ρ‚Π΅ΠΌΠΏΠ΅Ρ€Π°Ρ‚ΡƒΡ€Ρ‹.Π—Π½Π°Ρ‡ΠΈΡ‚, сопротивлСниС Π½Π΅ увСличится, Ссли ΠΌΡ‹ ΡƒΠ²Π΅Π»ΠΈΡ‡ΠΈΠΌ Ρ‚ΠΎΠΊ Π² Ρ†Π΅ΠΏΠΈ.
    Π‘Π»Π΅Π΄ΠΎΠ²Π°Ρ‚Π΅Π»ΡŒΠ½ΠΎ, этот Π²Π°Ρ€ΠΈΠ°Π½Ρ‚ Π½Π΅Π²Π΅Ρ€Π΅Π½.

    Π’Π°Ρ€ΠΈΠ°Π½Ρ‚ D. Π’Π΅ΠΏΠ΅Ρ€ΡŒ Π² этом Π²Π°Ρ€ΠΈΠ°Π½Ρ‚Π΅ указываСтся, Ρ‡Ρ‚ΠΎ сопротивлСниС ΠΈ Ρ‚ΠΎΠΊ Π±ΡƒΠ΄ΡƒΡ‚ ΡƒΠ²Π΅Π»ΠΈΡ‡ΠΈΠ²Π°Ρ‚ΡŒΡΡ ΠΏΡ€ΠΈ ΡƒΠ²Π΅Π»ΠΈΡ‡Π΅Π½ΠΈΠΈ напряТСния Π² Ρ†Π΅ΠΏΠΈ. Как ΠΎΠ±ΡΡƒΠΆΠ΄Π°Π»ΠΎΡΡŒ Π²Ρ‹ΡˆΠ΅, Ρ‚ΠΎΠΊ увСличиваСтся с ΡƒΠ²Π΅Π»ΠΈΡ‡Π΅Π½ΠΈΠ΅ΠΌ напряТСния, Π½ΠΎ сопротивлСниС ΠΏΡ€ΠΎΠ²ΠΎΠ΄Π½ΠΈΠΊΠ° остаСтся Π½Π΅ΠΈΠ·ΠΌΠ΅Π½Π½Ρ‹ΠΌ для Π΄Π°Π½Π½ΠΎΠΉ Ρ‚Π΅ΠΌΠΏΠ΅Ρ€Π°Ρ‚ΡƒΡ€Ρ‹. Π—Π½Π°Ρ‡ΠΈΡ‚, сопротивлСниС Π½Π΅ увСличится, Ссли ΠΌΡ‹ ΡƒΠ²Π΅Π»ΠΈΡ‡ΠΈΠΌ напряТСниС Π² Ρ†Π΅ΠΏΠΈ.
    Π‘Π»Π΅Π΄ΠΎΠ²Π°Ρ‚Π΅Π»ΡŒΠ½ΠΎ, этот Π²Π°Ρ€ΠΈΠ°Π½Ρ‚ Π½Π΅Π²Π΅Ρ€Π΅Π½.

    Π˜Ρ‚Π°ΠΊ, ΠΏΡ€Π°Π²ΠΈΠ»ΡŒΠ½Ρ‹ΠΉ ΠΎΡ‚Π²Π΅Ρ‚ — Π²Π°Ρ€ΠΈΠ°Π½Ρ‚ (А).

    ΠŸΡ€ΠΈΠΌΠ΅Ρ‡Π°Π½ΠΈΠ΅: ΠœΡ‹ Π·Π½Π°Π΅ΠΌ, Ρ‡Ρ‚ΠΎ сопротивлСниС постоянно для Π΄Π°Π½Π½ΠΎΠΉ Ρ‚Π΅ΠΌΠΏΠ΅Ρ€Π°Ρ‚ΡƒΡ€Ρ‹ Π² соотвСтствии с Π·Π°ΠΊΠΎΠ½ΠΎΠΌ Ома. Π˜Ρ‚Π°ΠΊ, Π²Π°Ρ€ΠΈΠ°Π½Ρ‚Ρ‹ B, C ΠΈ D Π½Π΅Π²Π΅Ρ€Π½Ρ‹. Π’Π΅ΠΏΠ΅Ρ€ΡŒ, ΠΏΠΎΡΠΊΠΎΠ»ΡŒΠΊΡƒ Ρ‚ΠΎΠΊ прямо ΠΏΡ€ΠΎΠΏΠΎΡ€Ρ†ΠΈΠΎΠ½Π°Π»Π΅Π½ Π½Π°ΠΏΡ€ΡΠΆΠ΅Π½ΠΈΡŽ Π² Ρ†Π΅ΠΏΠΈ. Π—Π½Π°Ρ‡ΠΈΡ‚, Ρ‚ΠΎΠΊ Π±ΡƒΠ΄Π΅Ρ‚ ΡƒΠ²Π΅Π»ΠΈΡ‡ΠΈΠ²Π°Ρ‚ΡŒΡΡ с ΡƒΠ²Π΅Π»ΠΈΡ‡Π΅Π½ΠΈΠ΅ΠΌ напряТСния Π² Ρ†Π΅ΠΏΠΈ.

    Π§Ρ‚ΠΎ Ρ‚Π°ΠΊΠΎΠ΅ Π·Π°ΠΊΠΎΠ½ Ома? | Fluke

    Π—Π°ΠΊΠΎΠ½ Ома — это Ρ„ΠΎΡ€ΠΌΡƒΠ»Π°, ΠΈΡΠΏΠΎΠ»ΡŒΠ·ΡƒΠ΅ΠΌΠ°Ρ для расчСта взаимосвязи ΠΌΠ΅ΠΆΠ΄Ρƒ напряТСниСм, Ρ‚ΠΎΠΊΠΎΠΌ ΠΈ сопротивлСниСм Π² элСктричСской Ρ†Π΅ΠΏΠΈ.

    Для ΠΈΠ·ΡƒΡ‡Π°ΡŽΡ‰ΠΈΡ… элСктронику Π·Π°ΠΊΠΎΠ½ Ома (E = IR) ΡΡ‚ΠΎΠ»ΡŒ ΠΆΠ΅ Ρ„ΡƒΠ½Π΄Π°ΠΌΠ΅Π½Ρ‚Π°Π»ΡŒΠ½ΠΎ Π²Π°ΠΆΠ΅Π½, ΠΊΠ°ΠΊ ΡƒΡ€Π°Π²Π½Π΅Π½ΠΈΠ΅ ΠΎΡ‚Π½ΠΎΡΠΈΡ‚Π΅Π»ΡŒΠ½ΠΎΡΡ‚ΠΈ Π­ΠΉΠ½ΡˆΡ‚Π΅ΠΉΠ½Π° (E = mcΒ²) для Ρ„ΠΈΠ·ΠΈΠΊΠΎΠ².

    E = I x R

    Π’ тСкстС это ΠΎΠ·Π½Π°Ρ‡Π°Π΅Ρ‚, Ρ‡Ρ‚ΠΎ напряТСниС = Ρ‚ΠΎΠΊ x сопротивлСниС , ΠΈΠ»ΠΈ Π’ = А x Ом , ΠΈΠ»ΠΈ Π’ = A x Ом .

    Названный Π² Ρ‡Π΅ΡΡ‚ΡŒ Π½Π΅ΠΌΠ΅Ρ†ΠΊΠΎΠ³ΠΎ Ρ„ΠΈΠ·ΠΈΠΊΠ° Π“Π΅ΠΎΡ€Π³Π° Ома (1789-1854), Π·Π°ΠΊΠΎΠ½ Ома опрСдСляСт ΠΊΠ»ΡŽΡ‡Π΅Π²Ρ‹Π΅ Π²Π΅Π»ΠΈΡ‡ΠΈΠ½Ρ‹, Π΄Π΅ΠΉΡΡ‚Π²ΡƒΡŽΡ‰ΠΈΠ΅ Π² цСпях:

    ΠšΠΎΠ»ΠΈΡ‡Π΅ΡΡ‚Π²ΠΎ Π—Π°ΠΊΠΎΠ½ Ома
    символ
    Π•Π΄ΠΈΠ½ΠΈΡ†Π° измСрСния
    (Π°Π±Π±Ρ€Π΅Π²ΠΈΠ°Ρ‚ΡƒΡ€Π°)
    Роль Π² схСмы Если Π²Π°ΠΌ интСрСсно:
    НапряТСниС E Π’ΠΎΠ»ΡŒΡ‚ (Π’) Π”Π°Π²Π»Π΅Π½ΠΈΠ΅, ΠΊΠΎΡ‚ΠΎΡ€ΠΎΠ΅ запускаСт ΠΏΠΎΡ‚ΠΎΠΊ элСктронов E = элСктродвиТущая сила (старая школа)
    Π’ΠΎΠΊ I АмпСр, Π°ΠΌΠΏΠ΅Ρ€ (A) Π‘ΠΊΠΎΡ€ΠΎΡΡ‚ΡŒ ΠΏΠΎΡ‚ΠΎΠΊΠ° элСктронов I = ΠΈΠ½Ρ‚Π΅Π½ΡΠΈΠ²Π½ΠΎΡΡ‚ΡŒ
    Π‘ΠΎΠΏΡ€ΠΎΡ‚ΠΈΠ²Π»Π΅Π½ΠΈΠ΅ R Ом (Ом) Π˜Π½Π³ΠΈΠ±ΠΈΡ‚ΠΎΡ€ ΠΏΠΎΡ‚ΠΎΠΊΠ° Ом = omega

    Если извСстны Π΄Π²Π° ΠΈΠ· этих Π·Π½Π°Ρ‡Π΅Π½ΠΈΠΉ, тСхничСскиС спСциалисты ΠΌΠΎΠ³ΡƒΡ‚ ΠΏΠ΅Ρ€Π΅Π½Π°ΡΡ‚Ρ€ΠΎΠΈΡ‚ΡŒ Π·Π°ΠΊΠΎΠ½ Ома, Ρ‡Ρ‚ΠΎΠ±Ρ‹ Π²Ρ‹Ρ‡ΠΈΡΠ»ΠΈΡ‚ΡŒ Ρ‚Ρ€Π΅Ρ‚ΡŒΠ΅.ΠŸΡ€ΠΎΡΡ‚ΠΎ ΠΈΠ·ΠΌΠ΅Π½ΠΈΡ‚Π΅ ΠΏΠΈΡ€Π°ΠΌΠΈΠ΄Ρƒ ΡΠ»Π΅Π΄ΡƒΡŽΡ‰ΠΈΠΌ ΠΎΠ±Ρ€Π°Π·ΠΎΠΌ:

    Если Π²Π°ΠΌ извСстны напряТСниС (E) ΠΈ Ρ‚ΠΎΠΊ (I) ΠΈ Π²Ρ‹ Ρ…ΠΎΡ‚ΠΈΡ‚Π΅ ΡƒΠ·Π½Π°Ρ‚ΡŒ сопротивлСниС (R), вытянитС R Π² ΠΏΠΈΡ€Π°ΠΌΠΈΠ΄Π΅ ΠΈ вычислитС ΠΎΡΡ‚Π°Π²ΡˆΠ΅Π΅ΡΡ ΡƒΡ€Π°Π²Π½Π΅Π½ΠΈΠ΅ (см. ΠŸΠ΅Ρ€Π²ΠΎΠ΅ ΠΈΠ»ΠΈ дальнСС слСва, ΠΏΠΈΡ€Π°ΠΌΠΈΠ΄Π° Π²Π²Π΅Ρ€Ρ…Ρƒ).

    ΠŸΡ€ΠΈΠΌΠ΅Ρ‡Π°Π½ΠΈΠ΅: Π‘ΠΎΠΏΡ€ΠΎΡ‚ΠΈΠ²Π»Π΅Π½ΠΈΠ΅ нСльзя ΠΈΠ·ΠΌΠ΅Ρ€ΠΈΡ‚ΡŒ Π² Ρ€Π°Π±ΠΎΡ‡Π΅ΠΉ Ρ†Π΅ΠΏΠΈ, поэтому Π·Π°ΠΊΠΎΠ½ Ома особСнно ΠΏΠΎΠ»Π΅Π·Π΅Π½, ΠΊΠΎΠ³Π΄Π° Π΅Π³ΠΎ Π½ΡƒΠΆΠ½ΠΎ Π²Ρ‹Ρ‡ΠΈΡΠ»ΠΈΡ‚ΡŒ. ВмСсто Ρ‚ΠΎΠ³ΠΎ, Ρ‡Ρ‚ΠΎΠ±Ρ‹ ΠΎΡ‚ΠΊΠ»ΡŽΡ‡Π°Ρ‚ΡŒ Ρ†Π΅ΠΏΡŒ для измСрСния сопротивлСния, тСхничСский спСциалист ΠΌΠΎΠΆΠ΅Ρ‚ ΠΎΠΏΡ€Π΅Π΄Π΅Π»ΠΈΡ‚ΡŒ R, ΠΈΡΠΏΠΎΠ»ΡŒΠ·ΡƒΡ Π²Ρ‹ΡˆΠ΅ΡƒΠΊΠ°Π·Π°Π½Π½Ρ‹ΠΉ Π²Π°Ρ€ΠΈΠ°Π½Ρ‚ Π·Π°ΠΊΠΎΠ½Π° Ома.

    Π’Π΅ΠΏΠ΅Ρ€ΡŒ, Ссли Π²Ρ‹ Π·Π½Π°Π΅Ρ‚Π΅ напряТСниС (E) ΠΈ сопротивлСниС (R) ΠΈ Ρ…ΠΎΡ‚ΠΈΡ‚Π΅ Π·Π½Π°Ρ‚ΡŒ Ρ‚ΠΎΠΊ (I), вытянитС X-I ΠΈ вычислитС ΠΎΡΡ‚Π°Π²ΡˆΠΈΠ΅ΡΡ Π΄Π²Π° символа (см. Π‘Ρ€Π΅Π΄Π½ΡŽΡŽ ΠΏΠΈΡ€Π°ΠΌΠΈΠ΄Ρƒ Π²Ρ‹ΡˆΠ΅).

    И Ссли Π²Ρ‹ Π·Π½Π°Π΅Ρ‚Π΅ Ρ‚ΠΎΠΊ (I) ΠΈ сопротивлСниС (R) ΠΈ Ρ…ΠΎΡ‚ΠΈΡ‚Π΅ Π·Π½Π°Ρ‚ΡŒ напряТСниС (E), ΡƒΠΌΠ½ΠΎΠΆΡŒΡ‚Π΅ Π½ΠΈΠΆΠ½ΠΈΠ΅ ΠΏΠΎΠ»ΠΎΠ²ΠΈΠ½Ρ‹ ΠΏΠΈΡ€Π°ΠΌΠΈΠ΄Ρ‹ (см. Π’Ρ€Π΅Ρ‚ΡŒΡŽ ΠΈΠ»ΠΈ ΠΊΡ€Π°ΠΉΠ½ΡŽΡŽ ΠΏΡ€Π°Π²ΡƒΡŽ ΠΏΠΈΡ€Π°ΠΌΠΈΠ΄Ρƒ Π²Ρ‹ΡˆΠ΅).

    ΠŸΠΎΠΏΡ€ΠΎΠ±ΡƒΠΉΡ‚Π΅ нСсколько ΠΏΡ€ΠΈΠΌΠ΅Ρ€ΠΎΠ² расчСтов Π½Π° основС простой ΠΏΠΎΡΠ»Π΅Π΄ΠΎΠ²Π°Ρ‚Π΅Π»ΡŒΠ½ΠΎΠΉ схСмы, которая Π²ΠΊΠ»ΡŽΡ‡Π°Π΅Ρ‚ Ρ‚ΠΎΠ»ΡŒΠΊΠΎ ΠΎΠ΄ΠΈΠ½ источник напряТСния (аккумулятор) ΠΈ сопротивлСниС (свСт).Π’ ΠΊΠ°ΠΆΠ΄ΠΎΠΌ ΠΏΡ€ΠΈΠΌΠ΅Ρ€Π΅ извСстны Π΄Π²Π° значСния. Π˜ΡΠΏΠΎΠ»ΡŒΠ·ΡƒΠΉΡ‚Π΅ Π·Π°ΠΊΠΎΠ½ Ома для вычислСния Ρ‚Ρ€Π΅Ρ‚ΡŒΠ΅Π³ΠΎ.

    ΠŸΡ€ΠΈΠΌΠ΅Ρ€ 1: НапряТСниС (E) ΠΈ сопротивлСниС (R) извСстны.

    Какой Ρ‚ΠΎΠΊ Π² Ρ†Π΅ΠΏΠΈ?

    I = E / R = 12 Π’ / 6 Ом = 2 А

    ΠŸΡ€ΠΈΠΌΠ΅Ρ€ 2: НапряТСниС (E) ΠΈ Ρ‚ΠΎΠΊ (I) извСстны.

    КакоС сопротивлСниС создаСт Π»Π°ΠΌΠΏΠ°?

    R = E / I = 24 Π’ / 6 A = 4 Ом

    ΠŸΡ€ΠΈΠΌΠ΅Ρ€ 3: Π’ΠΎΠΊ (I) ΠΈ сопротивлСниС (R) извСстны. КакоС напряТСниС?

    КакоС напряТСниС Π² Ρ†Π΅ΠΏΠΈ?

    E = I x R = (5A) (8Ξ©) = 40 Π’

    Когда Ом ΠΎΠΏΡƒΠ±Π»ΠΈΠΊΠΎΠ²Π°Π» свою Ρ„ΠΎΡ€ΠΌΡƒΠ»Ρƒ Π² 1827 Π³ΠΎΠ΄Ρƒ, Π΅Π³ΠΎ ΠΊΠ»ΡŽΡ‡Π΅Π²Ρ‹ΠΌ Π²Ρ‹Π²ΠΎΠ΄ΠΎΠΌ Π±Ρ‹Π»ΠΎ Ρ‚ΠΎ, Ρ‡Ρ‚ΠΎ Π²Π΅Π»ΠΈΡ‡ΠΈΠ½Π° элСктричСского Ρ‚ΠΎΠΊΠ°, ΠΏΡ€ΠΎΡ‚Π΅ΠΊΠ°ΡŽΡ‰Π΅Π³ΠΎ ΠΏΠΎ ΠΏΡ€ΠΎΠ²ΠΎΠ΄Π½ΠΈΠΊΡƒ, прямо ΠΏΡ€ΠΎΠΏΠΎΡ€Ρ†ΠΈΠΎΠ½Π°Π»ΡŒΠ½Π° ΠΏΡ€ΠΈΠ»ΠΎΠΆΠ΅Π½Π½ΠΎΠΌΡƒ Π½Π°ΠΏΡ€ΡΠΆΠ΅Π½ΠΈΡŽ. Π² Ρ‚Π΅ΠΌΠ΅.Π”Ρ€ΡƒΠ³ΠΈΠΌΠΈ словами, трСбуСтся ΠΎΠ΄ΠΈΠ½ Π²ΠΎΠ»ΡŒΡ‚ давлСния, Ρ‡Ρ‚ΠΎΠ±Ρ‹ ΠΏΡ€ΠΎΡ‚ΠΎΠ»ΠΊΠ½ΡƒΡ‚ΡŒ ΠΎΠ΄ΠΈΠ½ Π°ΠΌΠΏΠ΅Ρ€ Ρ‚ΠΎΠΊΠ° Ρ‡Π΅Ρ€Π΅Π· ΠΎΠ΄ΠΈΠ½ ΠΎΠΌ сопротивлСния.

    Π§Ρ‚ΠΎ ΠΏΡ€ΠΎΠ²Π΅Ρ€ΡΡ‚ΡŒ с ΠΏΠΎΠΌΠΎΡ‰ΡŒΡŽ Π·Π°ΠΊΠΎΠ½Π° Ома

    Π—Π°ΠΊΠΎΠ½ Ома ΠΌΠΎΠΆΠ½ΠΎ ΠΈΡΠΏΠΎΠ»ΡŒΠ·ΠΎΠ²Π°Ρ‚ΡŒ для ΠΏΡ€ΠΎΠ²Π΅Ρ€ΠΊΠΈ статичСских Π·Π½Π°Ρ‡Π΅Π½ΠΈΠΉ ΠΊΠΎΠΌΠΏΠΎΠ½Π΅Π½Ρ‚ΠΎΠ² схСмы, ΡƒΡ€ΠΎΠ²Π½Π΅ΠΉ Ρ‚ΠΎΠΊΠ°, источников напряТСния ΠΈ ΠΏΠ°Π΄Π΅Π½ΠΈΠΉ напряТСния. Если, Π½Π°ΠΏΡ€ΠΈΠΌΠ΅Ρ€, ΠΈΠ·ΠΌΠ΅Ρ€ΠΈΡ‚Π΅Π»ΡŒΠ½Ρ‹ΠΉ ΠΏΡ€ΠΈΠ±ΠΎΡ€ ΠΎΠ±Π½Π°Ρ€ΡƒΠΆΠΈΠ²Π°Π΅Ρ‚ Π·Π½Π°Ρ‡Π΅Π½ΠΈΠ΅ Ρ‚ΠΎΠΊΠ°, ΠΏΡ€Π΅Π²Ρ‹ΡˆΠ°ΡŽΡ‰Π΅Π΅ Π½ΠΎΡ€ΠΌΠ°Π»ΡŒΠ½Ρ‹ΠΉ, это ΠΌΠΎΠΆΠ΅Ρ‚ ΠΎΠ·Π½Π°Ρ‡Π°Ρ‚ΡŒ, Ρ‡Ρ‚ΠΎ сопротивлСниС ΡƒΠΌΠ΅Π½ΡŒΡˆΠΈΠ»ΠΎΡΡŒ ΠΈΠ»ΠΈ Ρ‡Ρ‚ΠΎ напряТСниС ΡƒΠ²Π΅Π»ΠΈΡ‡ΠΈΠ»ΠΎΡΡŒ, вызывая ΡΠΈΡ‚ΡƒΠ°Ρ†ΠΈΡŽ высокого напряТСния. Π­Ρ‚ΠΎ ΠΌΠΎΠΆΠ΅Ρ‚ ΡƒΠΊΠ°Π·Ρ‹Π²Π°Ρ‚ΡŒ Π½Π° ΠΏΡ€ΠΎΠ±Π»Π΅ΠΌΡƒ с ΠΏΠΈΡ‚Π°Π½ΠΈΠ΅ΠΌ ΠΈΠ»ΠΈ Ρ†Π΅ΠΏΡŒΡŽ.

    Π’ цСпях постоянного Ρ‚ΠΎΠΊΠ° (dc) ΠΈΠ·ΠΌΠ΅Ρ€Π΅Π½ΠΈΠ΅ Ρ‚ΠΎΠΊΠ° Π½ΠΈΠΆΠ΅ Π½ΠΎΡ€ΠΌΠ°Π»ΡŒΠ½ΠΎΠ³ΠΎ ΠΌΠΎΠΆΠ΅Ρ‚ ΠΎΠ·Π½Π°Ρ‡Π°Ρ‚ΡŒ, Ρ‡Ρ‚ΠΎ напряТСниС снизилось ΠΈΠ»ΠΈ сопротивлСниС Ρ†Π΅ΠΏΠΈ ΡƒΠ²Π΅Π»ΠΈΡ‡ΠΈΠ»ΠΎΡΡŒ. Π’ΠΎΠ·ΠΌΠΎΠΆΠ½Ρ‹Π΅ ΠΏΡ€ΠΈΡ‡ΠΈΠ½Ρ‹ ΠΏΠΎΠ²Ρ‹ΡˆΠ΅Π½Π½ΠΎΠ³ΠΎ сопротивлСния — ΠΏΠ»ΠΎΡ…ΠΈΠ΅ ΠΈΠ»ΠΈ Π½Π΅ΠΏΠ»ΠΎΡ‚Π½Ρ‹Π΅ соСдинСния, коррозия ΠΈ / ΠΈΠ»ΠΈ ΠΏΠΎΠ²Ρ€Π΅ΠΆΠ΄Π΅Π½Π½Ρ‹Π΅ ΠΊΠΎΠΌΠΏΠΎΠ½Π΅Π½Ρ‚Ρ‹.

    Нагрузки Π² Ρ†Π΅ΠΏΠΈ ΠΏΠΎΡ‚Ρ€Π΅Π±Π»ΡΡŽΡ‚ элСктричСский Ρ‚ΠΎΠΊ. Нагрузки ΠΌΠΎΠ³ΡƒΡ‚ Π±Ρ‹Ρ‚ΡŒ Π»ΡŽΠ±Ρ‹ΠΌΠΈ ΠΊΠΎΠΌΠΏΠΎΠ½Π΅Π½Ρ‚Π°ΠΌΠΈ: нСбольшими элСктричСскими устройствами, ΠΊΠΎΠΌΠΏΡŒΡŽΡ‚Π΅Ρ€Π°ΠΌΠΈ, Π±Ρ‹Ρ‚ΠΎΠ²ΠΎΠΉ Ρ‚Π΅Ρ…Π½ΠΈΠΊΠΎΠΉ ΠΈΠ»ΠΈ большим Π΄Π²ΠΈΠ³Π°Ρ‚Π΅Π»Π΅ΠΌ. На Π±ΠΎΠ»ΡŒΡˆΠΈΠ½ΡΡ‚Π²Π΅ этих ΠΊΠΎΠΌΠΏΠΎΠ½Π΅Π½Ρ‚ΠΎΠ² (Π½Π°Π³Ρ€ΡƒΠ·ΠΎΠΊ) Π΅ΡΡ‚ΡŒ паспортная Ρ‚Π°Π±Π»ΠΈΡ‡ΠΊΠ° ΠΈΠ»ΠΈ информационная Π½Π°ΠΊΠ»Π΅ΠΉΠΊΠ°.На этих паспортных Ρ‚Π°Π±Π»ΠΈΡ‡ΠΊΠ°Ρ… ΡƒΠΊΠ°Π·Π°Π½Ρ‹ сСртификаты бСзопасности ΠΈ нСсколько ссылочных Π½ΠΎΠΌΠ΅Ρ€ΠΎΠ².

    ВСхничСскиС спСциалисты ΠΎΠ±Ρ€Π°Ρ‰Π°ΡŽΡ‚ΡΡ ΠΊ заводским Ρ‚Π°Π±Π»ΠΈΡ‡ΠΊΠ°ΠΌ Π½Π° ΠΊΠΎΠΌΠΏΠΎΠ½Π΅Π½Ρ‚Π°Ρ…, Ρ‡Ρ‚ΠΎΠ±Ρ‹ ΡƒΠ·Π½Π°Ρ‚ΡŒ стандартныС значСния напряТСния ΠΈ Ρ‚ΠΎΠΊΠ°. Π’ΠΎ врСмя тСстирования, Ссли тСхничСскиС спСциалисты ΠΎΠ±Π½Π°Ρ€ΡƒΠΆΠΈΠ²Π°ΡŽΡ‚, Ρ‡Ρ‚ΠΎ ΠΎΠ±Ρ‹Ρ‡Π½Ρ‹Π΅ значСния Π½Π΅ Ρ€Π΅Π³ΠΈΡΡ‚Ρ€ΠΈΡ€ΡƒΡŽΡ‚ΡΡ Π½Π° ΠΈΡ… Ρ†ΠΈΡ„Ρ€ΠΎΠ²Ρ‹Ρ… ΠΌΡƒΠ»ΡŒΡ‚ΠΈΠΌΠ΅Ρ‚Ρ€Π°Ρ… ΠΈΠ»ΠΈ Ρ‚ΠΎΠΊΠΎΠΈΠ·ΠΌΠ΅Ρ€ΠΈΡ‚Π΅Π»ΡŒΠ½Ρ‹Ρ… ΠΊΠ»Π΅Ρ‰Π°Ρ…, ΠΎΠ½ΠΈ ΠΌΠΎΠ³ΡƒΡ‚ ΠΈΡΠΏΠΎΠ»ΡŒΠ·ΠΎΠ²Π°Ρ‚ΡŒ Π·Π°ΠΊΠΎΠ½ Ома, Ρ‡Ρ‚ΠΎΠ±Ρ‹ ΠΎΠΏΡ€Π΅Π΄Π΅Π»ΠΈΡ‚ΡŒ, какая Ρ‡Π°ΡΡ‚ΡŒ Ρ†Π΅ΠΏΠΈ Π΄Π°Π΅Ρ‚ сбой, ΠΈ Π½Π° основании этого ΠΎΠΏΡ€Π΅Π΄Π΅Π»ΠΈΡ‚ΡŒ, Π² Ρ‡Π΅ΠΌ ΠΌΠΎΠΆΠ΅Ρ‚ Π·Π°ΠΊΠ»ΡŽΡ‡Π°Ρ‚ΡŒΡΡ ΠΏΡ€ΠΎΠ±Π»Π΅ΠΌΠ°.

    ΠžΡΠ½ΠΎΠ²Ρ‹ Π½Π°ΡƒΠΊΠΈ ΠΎ схСмах

    Π‘Ρ…Π΅ΠΌΡ‹, ΠΊΠ°ΠΊ ΠΈ вся матСрия, состоят ΠΈΠ· Π°Ρ‚ΠΎΠΌΠΎΠ².Атомы состоят ΠΈΠ· субатомных частиц:

    • ΠŸΡ€ΠΎΡ‚ΠΎΠ½Ρ‹ (с ΠΏΠΎΠ»ΠΎΠΆΠΈΡ‚Π΅Π»ΡŒΠ½Ρ‹ΠΌ элСктричСским зарядом)
    • НСйтроны (Π±Π΅Π· заряда)
    • Π­Π»Π΅ΠΊΡ‚Ρ€ΠΎΠ½Ρ‹ (с ΠΎΡ‚Ρ€ΠΈΡ†Π°Ρ‚Π΅Π»ΡŒΠ½Ρ‹ΠΌ зарядом)

    Атомы ΠΎΡΡ‚Π°ΡŽΡ‚ΡΡ связанными силами притяТСния ΠΌΠ΅ΠΆΠ΄Ρƒ ядром Π°Ρ‚ΠΎΠΌΠ° ΠΈ элСктронами Π² Π΅Π³ΠΎ внСшняя ΠΎΠ±ΠΎΠ»ΠΎΡ‡ΠΊΠ°. Под воздСйствиСм напряТСния Π°Ρ‚ΠΎΠΌΡ‹ Π² Ρ†Π΅ΠΏΠΈ Π½Π°Ρ‡ΠΈΠ½Π°ΡŽΡ‚ ΠΏΡ€Π΅ΠΎΠ±Ρ€Π°Π·ΠΎΠ²Ρ‹Π²Π°Ρ‚ΡŒΡΡ, ΠΈ ΠΈΡ… ΠΊΠΎΠΌΠΏΠΎΠ½Π΅Π½Ρ‚Ρ‹ ΠΏΡ€ΠΎΡΠ²Π»ΡΡŽΡ‚ ΠΏΠΎΡ‚Π΅Π½Ρ†ΠΈΠ°Π» притяТСния, извСстный ΠΊΠ°ΠΊ Ρ€Π°Π·Π½ΠΎΡΡ‚ΡŒ ΠΏΠΎΡ‚Π΅Π½Ρ†ΠΈΠ°Π»ΠΎΠ². Π’Π·Π°ΠΈΠΌΠ½ΠΎ ΠΏΡ€ΠΈΠ²Π»Π΅Ρ‡Π΅Π½Π½Ρ‹Π΅ свободныС элСктроны двиТутся ΠΊ ΠΏΡ€ΠΎΡ‚ΠΎΠ½Π°ΠΌ, создавая ΠΏΠΎΡ‚ΠΎΠΊ элСктронов (Ρ‚ΠΎΠΊ).Π›ΡŽΠ±ΠΎΠΉ ΠΌΠ°Ρ‚Π΅Ρ€ΠΈΠ°Π» Π² Ρ†Π΅ΠΏΠΈ, ΠΎΠ³Ρ€Π°Π½ΠΈΡ‡ΠΈΠ²Π°ΡŽΡ‰ΠΈΠΉ этот ΠΏΠΎΡ‚ΠΎΠΊ, считаСтся сопротивлСниСм.

    Бсылка: ΠŸΡ€ΠΈΠ½Ρ†ΠΈΠΏΡ‹ Ρ†ΠΈΡ„Ρ€ΠΎΠ²ΠΎΠ³ΠΎ ΠΌΡƒΠ»ΡŒΡ‚ΠΈΠΌΠ΅Ρ‚Ρ€Π° Π“Π»Π΅Π½Π° А. ΠœΠ°Π·ΡƒΡ€Π°, American Technical Publishers.

    Π‘ΠΎΠΏΡƒΡ‚ΡΡ‚Π²ΡƒΡŽΡ‰ΠΈΠ΅ ΡΡ‚Π°Ρ‚ΡŒΠΈ

    ΠΌΠΎΡ‰Π½ΠΎΡΡ‚ΡŒ — Π‘ΠΎΠΎΡ‚Π½ΠΎΡˆΠ΅Π½ΠΈΠ΅ ΠΌΠ΅ΠΆΠ΄Ρƒ Ρ‚ΠΎΠΊΠΎΠΌ, сопротивлСниСм ΠΈ напряТСниСм согласно Π·Π°ΠΊΠΎΠ½Ρƒ Ома, Π·Π°ΠΊΠΎΠ½Ρƒ Ρ‚Π΅ΠΏΠ»Π° ДТоуля ΠΈ P = IV

    Π”Π°Π²Π°ΠΉΡ‚Π΅ ΠΏΡ€ΠΈΠ²Π΅Π΄Π΅ΠΌ свои мысли ΠΏΠΎ порядку …

    Π—Π°ΠΊΠΎΠ½ Ома касаСтся Π»ΠΈΠ½Π΅ΠΉΠ½ΠΎΠ³ΠΎ (постоянного) сопротивлСния. Π—Π½Π°Ρ‡ΠΈΡ‚, ΠΌΡ‹ Π½Π΅ Π΄ΠΎΠ»ΠΆΠ½Ρ‹ Π΄ΠΎΠΏΡƒΡΠΊΠ°Ρ‚ΡŒ потСплСния. Π’ Ρ‚Π°ΠΊΠΎΠΌ располоТСнии ΠΌΡ‹ Π½Π°Ρ‡ΠΈΠ½Π°Π΅ΠΌ ΡΠΊΡΠΏΠ΅Ρ€ΠΈΠΌΠ΅Π½Ρ‚ΠΈΡ€ΠΎΠ²Π°Ρ‚ΡŒ.

    Π‘Π½Π°Ρ‡Π°Π»Π° ΠΌΡ‹ ΠΏΠΎΠ΄ΠΊΠ»ΡŽΡ‡Π°Π΅ΠΌ источник напряТСния ΠΊ рСзистору ΠΈ Π½Π°Ρ‡ΠΈΠ½Π°Π΅ΠΌ ΠΈΠ·ΠΌΠ΅Π½ΡΡ‚ΡŒ напряТСниС Π½Π° Π½Π΅ΠΌ. Π’ Ρ€Π΅Π·ΡƒΠ»ΡŒΡ‚Π°Ρ‚Π΅ ΠΏΡ€ΠΎΠΏΠΎΡ€Ρ†ΠΈΠΎΠ½Π°Π»ΡŒΠ½ΠΎ измСнится Ρ‚ΠΎΠΊ Ρ‡Π΅Ρ€Π΅Π· рСзистор — Iout = Vin / R. Если ΠΌΡ‹ ΠΈΠ·ΠΌΠ΅Π½ΠΈΠΌ сопротивлСниС, Ρ‚ΠΎΠΊ Π±ΡƒΠ΄Π΅Ρ‚ ΠΏΡ€ΠΎΠΏΠΎΡ€Ρ†ΠΈΠΎΠ½Π°Π»ΡŒΠ½ΠΎ ΠΈΠ·ΠΌΠ΅Π½ΡΡ‚ΡŒΡΡ ΠΎΠ±Ρ€Π°Ρ‚Π½ΠΎ — Iout = V / Rin.

    Π—Π°Ρ‚Π΅ΠΌ ΠΌΡ‹ ΠΏΠΎΠ΄ΠΊΠ»ΡŽΡ‡Π°Π΅ΠΌ источник Ρ‚ΠΎΠΊΠ° ΠΊ рСзистору ΠΈ Π½Π°Ρ‡ΠΈΠ½Π°Π΅ΠΌ ΠΈΠ·ΠΌΠ΅Π½ΡΡ‚ΡŒ Ρ‚ΠΎΠΊ Ρ‡Π΅Ρ€Π΅Π· Π½Π΅Π³ΠΎ. Π’Π΅ΠΏΠ΅Ρ€ΡŒ напряТСниС Π½Π° рСзисторС Π±ΡƒΠ΄Π΅Ρ‚ ΠΏΡ€ΠΎΠΏΠΎΡ€Ρ†ΠΈΠΎΠ½Π°Π»ΡŒΠ½ΠΎ ΠΈΠ·ΠΌΠ΅Π½ΡΡ‚ΡŒΡΡ — Vout = Iin.R. Если ΠΌΡ‹ ΠΈΠ·ΠΌΠ΅Π½ΠΈΠΌ сопротивлСниС, напряТСниС Π±ΡƒΠ΄Π΅Ρ‚ ΠΏΡ€ΠΎΠΏΠΎΡ€Ρ†ΠΈΠΎΠ½Π°Π»ΡŒΠ½ΠΎ ΠΈΠ·ΠΌΠ΅Π½ΡΡ‚ΡŒΡΡ — Vout = I.2.Π . Если Π²Ρ‹ Ρ…ΠΎΡ‚ΠΈΡ‚Π΅, Ρ‡Ρ‚ΠΎΠ±Ρ‹ ΠΎΠ½ΠΎ ΠΎΡΡ‚Π°Π²Π°Π»ΠΎΡΡŒ постоянным, ΠΈΠ·ΠΌΠ΅Π½ΠΈΡ‚Π΅ сопротивлСниС Π² ΡΠΎΠΎΡ‚Π²Π΅Ρ‚ΡΡ‚Π²ΡƒΡŽΡ‰Π΅ΠΌ Π½Π°ΠΏΡ€Π°Π²Π»Π΅Π½ΠΈΠΈ. Π­Ρ‚ΠΎ ΠΎΠ·Π½Π°Ρ‡Π°Π΅Ρ‚, Ρ‡Ρ‚ΠΎ Π² Π·Π°ΠΊΠΎΠ½Π΅ Ома Π΅ΡΡ‚ΡŒ Π΄Π²Π΅ Π²Ρ…ΠΎΠ΄Π½Ρ‹Π΅ ΠΏΠ΅Ρ€Π΅ΠΌΠ΅Π½Π½Ρ‹Π΅ — Iout = Vin / Rin ΠΈ Vout = Iin.Rin, ΠΈΠ»ΠΈ сопротивлСниС стало «динамичСским».

    Π’Π°ΠΊΠΈΠ΅ ΡƒΠ»ΠΎΠ²ΠΊΠΈ ΠΈΡΠΏΠΎΠ»ΡŒΠ·ΡƒΡŽΡ‚ΡΡ для создания Π½Π΅Π»ΠΈΠ½Π΅ΠΉΠ½Ρ‹Ρ… рСзисторов, ΠΊΠΎΡ‚ΠΎΡ€Ρ‹Π΅ ΠΏΠΎΠ΄Π΄Π΅Ρ€ΠΆΠΈΠ²Π°ΡŽΡ‚ постоянноС напряТСниС (Π½Π°ΠΏΡ€ΠΈΠΌΠ΅Ρ€, стабилитрон) ΠΈΠ»ΠΈ постоянный Ρ‚ΠΎΠΊ (Π½Π°ΠΏΡ€ΠΈΠΌΠ΅Ρ€, транзистор). Π­Ρ‚ΠΎ Π²ΠΈΠ΄Π½ΠΎ ΠΏΠΎ ВАΠ₯ Π΄ΠΈΠΎΠ΄Π° ΠΈ Π²Ρ‹Ρ…ΠΎΠ΄Π½ΠΎΠΉ характСристикС транзистора.

    ΠšΠΎΠ½Π΅Ρ‡Π½ΠΎ, ΠΌΡ‹ ΠΌΠΎΠΆΠ΅ΠΌ ΠΏΠΎΠ΄Π΄Π΅Ρ€ΠΆΠΈΠ²Π°Ρ‚ΡŒ ΠΏΠΎΡΡ‚ΠΎΡΠ½Π½ΡƒΡŽ ΠΌΠΎΡ‰Π½ΠΎΡΡ‚ΡŒ Π² соотвСтствии с Pout = Vin.Iin. Π­Ρ‚ΠΎ ΠΎΠ·Π½Π°Ρ‡Π°Π΅Ρ‚ ΠΏΠΎΠ΄ΠΊΠ»ΡŽΡ‡Π΅Π½ΠΈΠ΅ источника напряТСния ΠΊ источнику Ρ‚ΠΎΠΊΠ°. Π’Π°ΠΊΠΈΠΌ ΠΎΠ±Ρ€Π°Π·ΠΎΠΌ, источник напряТСния Π±ΡƒΠ΄Π΅Ρ‚ ΡƒΡΡ‚Π°Π½Π°Π²Π»ΠΈΠ²Π°Ρ‚ΡŒ напряТСниС Π½Π° источниках, Π° источник Ρ‚ΠΎΠΊΠ° Π±ΡƒΠ΄Π΅Ρ‚ ΡƒΡΡ‚Π°Π½Π°Π²Π»ΠΈΠ²Π°Ρ‚ΡŒ Ρ‚ΠΎΠΊ Ρ‡Π΅Ρ€Π΅Π· Π½ΠΈΡ…. Π’ΠΎΡ‡Π½Π΅Π΅ говоря, Π² этом располоТСнии Ρ‚ΠΎΠ»ΡŒΠΊΠΎ ΠΎΠ΄ΠΈΠ½ ΠΈΠ· элСмСнтов являСтся источником; Π΄Ρ€ΡƒΠ³ΠΎΠΉ — Π½Π°Π³Ρ€ΡƒΠ·ΠΊΠ°, рСализованная ΠΊΠ°ΠΊ Π½Π΅Π»ΠΈΠ½Π΅ΠΉΠ½Ρ‹ΠΉ рСзистор.

    Π’Π΅ΠΏΠ΅Ρ€ΡŒ, Ρ‡Ρ‚ΠΎΠ±Ρ‹ ΡΠΎΡ…Ρ€Π°Π½ΠΈΡ‚ΡŒ ΠΏΠΎΡΡ‚ΠΎΡΠ½Π½ΡƒΡŽ ΠΌΠΎΡ‰Π½ΠΎΡΡ‚ΡŒ, ΠΏΡ€ΠΈ ΡƒΠ²Π΅Π»ΠΈΡ‡Π΅Π½ΠΈΠΈ напряТСния ΠΌΡ‹ ΡƒΠΌΠ΅Π½ΡŒΡˆΠ°Π΅ΠΌ Ρ‚ΠΎΠΊ ΠΈ v.

    Π”ΠΎΠ±Π°Π²ΠΈΡ‚ΡŒ ΠΊΠΎΠΌΠΌΠ΅Π½Ρ‚Π°Ρ€ΠΈΠΉ

    Π’Π°Ρˆ адрСс email Π½Π΅ Π±ΡƒΠ΄Π΅Ρ‚ ΠΎΠΏΡƒΠ±Π»ΠΈΠΊΠΎΠ²Π°Π½. ΠžΠ±ΡΠ·Π°Ρ‚Π΅Π»ΡŒΠ½Ρ‹Π΅ поля ΠΏΠΎΠΌΠ΅Ρ‡Π΅Π½Ρ‹ *